Общая химия» (учебное пособие)

advertisement
ФЕДЕРАЛЬНОЕ АГЕНТСТВО ПО ОБРАЗОВАНИЮ
МУРМАНСКИЙ ГОСУДАРСТВЕННЫЙ ПЕДАГОГИЧЕСКИЙ УНИВЕРСИТЕТ
ОБЩАЯ ХИМИЯ
Учебное пособие
МУРМАНСК
2010
УДК 24.1я73
ББК 54 (075.8)
0-28
Печатается по решению редакционно-издательского совета Мурманского
государственного педагогического университета
Рецензенты: С.И. Овчинникова, кандидат химических наук, профессор, зав.
кафедрой биохимии МГТУ;
B.А. Крыштоп, кандидат педагогических наук, ст. преподаватель
кафедры биологии и химии МГПУ.
Общая химия: Учебное пособие / Авт.-сост. В.В. Сагайдачная. – Мурманск: МГПУ, 2010. – 116 с.
Пособие содержит теоретический материал, примеры решения типовых
задач и заданий, задачи и упражнения для самостоятельной работы, задания для
промежуточного и итогового контроля по основным разделам курса «Общая химия».
Предназначено для студентов, обучающихся по специальностям «Биология», «Экология», «География», «Математика», «Физика», «Информатика»,
«Безопасность жизнедеятельности», «Технология и предпринимательство». Модульный подход, применяемый в учебном пособии, дает возможность использовать его в соответствии с любыми учебными планами, при любой последовательности изучения тем курса общей химии. Особенностью пособия является
то, что в него включены задания применительно не к отдельному понятию или
закону, а к комплексу знаний.
Применение пособия в учебном процессе в значительной степени облегчит освоение студентами теоретического курса, позволит эффективно организовать самостоятельную работу студентов, развивать познавательную активность
обучающихся, а также оценить уровень теоретических знаний и сформированности практических умений.
Данное пособие может быть использовано студентами и преподавателями
нехимических специальностей вузов.
© Мурманский государственный
педагогический университет (МГПУ), 2010
2
ПРЕДИСЛОВИЕ
Курс общей химии в вузе выполняет следующие задачи: является
связующим звеном между довузовским и вузовским этапами химического
образования; это фундамент для изучения других естественнонаучных
дисциплин, а также важный компонент специальных дисциплин.
Потребность в подобном пособии по общей химии диктуется современными требования к подготовке специалистов и необходимостью в пополнении комплекса учебно-методических материалов по общей химии.
Настоящее учебное пособие предназначено для студентов специальностей 020201 «Биология», 050102 «Биология», 032400 «БиологияГеография», 020801 «Экология», 050103 «География», 050103 «ГеографияБиология», 050201
«Математика-физика», 050201 «Математикаинформатика», 050203 «Физика-информатика», 050202 «Информатика»,
050202 «Информатика-физика», 050104 «Безопасность жизнедеятельности», 050502 «Технология и предпринимательство».
Весь материал пособия изложен в четырех модулях, которые охватывают основные тематические блоки курса общей химии: «Основные понятия общей химии», «Важнейшие классы неорганических соединений»,
«Современная номенклатура неорганических веществ», «Химические
свойства и способы получения неорганических соединений», «Классификация и особенности протекания химических реакций», «Строение атома»,
«Периодическая система и Периодический закон Д.И. Менделеева в свете
современных представлений о строении атома», «Химическая связь»,
«Комплексные соединения», «Термодинамика химических процессов»,
«Химическая кинетика», «Обратимость химических реакций», «Химическое равновесие».
Модульный подход, применяемый в данном пособии, дает возможность использовать его в соответствии с любыми учебными планами, при
любой последовательности изучения тем курса общей химии, гибко варьируя тематические блоки. Особенностью пособия является то, что в него
включены задания применительно не к отдельному понятию или закону, а
к комплексу знаний.
В каждом модуле приведен теоретический материал, примеры решения типовых задач и заданий, задачи и упражнения для самостоятельной работы, задания для промежуточного и итогового контроля, список
рекомендуемой литературы. В пособие включены задания разной степени
сложности, что дает возможность дифференцировать работу со студентами
и разнообразить тематику заданий.
Применение данного пособия в учебном процессе в значительной
степени облегчит освоение студентами теоретического курса, позволит
развивать логическое мышление, различные интеллектуальные умения,
будет способствовать активизации познавательной активности обучаю3
щихся путем увеличения их самостоятельной работы, что отвечает требованиям Государственного образовательного стандарта высшего профессионального образования.
Учебное пособие выполняет обучающую, самоорганизующую, контролирующую и прикладную функции. Оно может быть использовано студентами при изучении курса общей химии, для самостоятельной работы,
подготовки к зачету и экзамену, а также для контроля и оценки знаний
студентов преподавателем.
4
Глава 1. Основные понятия химии. Современная номенклатура
неорганических веществ. Химические свойства и способы получения
неорганических соединений. Химические реакции
Основные понятия общей химии
Химия – это область естествознания, наука о веществах, их составе,
строении, свойствах и превращениях (рис.1).
Рис.1. Основные понятия общей химии.
Атом – электронейтральная частица, состоящая из положительно заряженного ядра и отрицательно заряженных электронов.
Молекула – отдельная электронейтральная частица, которая образуется при возникновении ковалентных связей между атомами одного или
нескольких элементов и определяет химические свойства вещества.
Химический элемент – совокупность атомов с одинаковым положительным зарядом ядра. Все элементы делят на металлы и неметаллы. К
неметаллам относят 22 элемента: водород, бор, углерод, кремний, азот,
фосфор, мышьяк, кислород, серу, селен, теллур, галогены и благородные
газы, к металлам – все остальные элементы. Химические элементы могут
иметь несколько форм существования: в виде свободных атомов, простых
и сложных веществ, а также ионов и радикалов.
Вещество – форма материи, состоящая из частиц, которые имеют
массу покоя, например, атомов, молекул, ионов (в отличие от частиц поля,
не имеющих массы покоя).
5
Вещества, образованные атомами одного химического элемента, называют простыми, а двух и более – сложными.
Один и тот же химический элемент может образовывать несколько
простых веществ. Это явление называют аллотропией, а различные простые вещества, образованные одним элементом, – аллотропными модификациями. Причиной аллотропии может быть как разное число атомов в
молекуле (например, модификации кислорода – кислород О2 и озон О3),
так и разное строение кристаллических форм (например, модификации углерода: алмаз – с тетраэдрической кристаллической решеткой, графит – с
плоскостной, карбин – с линейной).
Вещества могут иметь один и тот же качественный и количественный состав (т. е. одинаковые молекулярные формулы), но разное строение,
а, следовательно, разные свойства. Это явление называют изомерией, а
вещества – изомерами (например, роданид аммония NH4SCN и тиомочевина (NH2)2CS).
Радикалы – это атомы или группировки атомов, имеющие, по меньшей мере, один неспаренный электрон (свободную валентность). Радикалы
образуются, если молекулу разделить на атомы (или группы атомов) так,
чтобы каждый (или каждая) получил по электрону из общей электронной
пары. Радикалы в целом электронейтральны, так как имеют одинаковое
число электронов и протонов.
Ионы – частицы, у которых наблюдается дисбаланс между положительным зарядом ядра и числом электронов.
Отдавая электроны, атомы превращаются в положительные ионы –
катионы: Na0 – 1е → Na+. Это ионы металлов Меn+, водорода Н+, аммония
NН4+ и др.
Присоединяя электроны, атомы превращаются в отрицательные
ионы – анионы: Cl0 +1е→ Cl-1. Это простые анионы неметаллов: Н- (гидрид-анион), S2- (сульфид-анион) и др.; сложные по составу анионы: ОН(гидроксид-анион), SO42- (сульфат-анион), SO32- (сульфит-анион) и др.
Химические формулы – это способ отражения химического состава
вещества. Химическая формула обозначает одну молекулу вещества или
один моль этого вещества. По химической формуле можно также определить качественный состав вещества, число атомов каждого элемента в 1
моль вещества и рассчитать его относительную молекулярную и молярную
массы. Виды химических формул:
1) Простейшая (эмпирическая) формула показывает качественный состав и соотношения, в которых находятся частицы, образующие данное вещество: атомы, ионы или группы атомов (например, простейшие формулы этена и пропена совпадают – СН2).
2) Молекулярная (истинная) формула отражает качественный состав и
число составляющих вещество частиц (например, С2Н4– этен и С3Н6
6
– пропен), но не показывает порядок связи частиц в веществе, т. е.
его структуру.
3) Структурная (графическая) формула отражает порядок соединения частиц, т. е. связи между ними (например, структурная формула молекулы оксида серы (VI): О – S=О).
Число природных и синтезированных веществ составляет более 20
млн. Каждое из них при данных условиях обладает определенной совокупностью физических и химических свойств.
Физические свойства веществ: агрегатное состояние (твердое (тв.),
жидкое (ж.), газообразное (г.)), температура кипения, температура плавления, плотность (ρ), выраженная в г/см3, кг/дм3 и др., растворимость
(например, выраженная в г/100г Н2О), цвет, запах, вкус и др.
Способность данного вещества при определенных условиях или при
взаимодействии с другими веществами образовывать новые вещества
называется его химическими свойствами.
Химические реакции – превращения одних веществ в другие.
Химическое уравнение – это условная запись химической реакции с
помощью химических знаков и формул в стехиометрических соотношениях. Коэффициенты в уравнении реакции – числа, которые показывают
мольные соотношения участвующих в реакции веществ. Если в уравнении
указывают тепловой эффект в расчете на 1 моль реагента или продукта, то
такое уравнение реакции называют термохимическим.
Важнейшие классы неорганических соединений
и их номенклатура
Оксиды – это соединения двух элементов, один из которых кислород в степени окисления -2. По химическим свойствам оксиды подразделяют на несолеобразующие (СО, SiО, N2О, NО) и солеобразующие.
Несолеобразующие (безразличные, индифферентные) оксиды не образуют ни гидратов, ни солей. Солеобразующие оксиды подразделяются
на основные, кислотные и амфотерные.
Основным оксидам (Na2O, СаО и др.) отвечают основания (NaOН,
Са(ОН)2), кислотным (СО2, SO3 и др.) – кислоты (H2СO3, H2SO4).
Кислотные оксиды можно получить из кислот, отнимая от них воду,
поэтому их называют также ангидридами кислот.
Амфотерным оксидам (BeO, ZnO, Al2O3, Fe2O3, Cr2O3, МnO2 и др.)
отвечают гидраты, проявляющие кислотные и основные свойства.
Выделяют пероксиды (перекиси) металлов (Na2O2, ВаО2 и др.).
Степень окисления кислорода в них -1, по своей природе это соли очень
слабой кислоты – пероксида (перекиси) водорода Н2О2.
Основания – сложные вещества, состоящие из металла и одновалентных гидроксогрупп ОН, число которых равно валентности металла
7
(гидроксид натрия NaOH, гидроксид меди (II) Сu(ОН)2 и др.). Основания
классифицируют по их растворимости в воде, по кислотности и по их силе.
По растворимости основания делятся на растворимые (щелочи) и на
нерастворимые.
По кислотности основания делятся на однокислотные (NaOН,
NН4OН), двухкислотные (Сu(ОН)2 Fe(OH)2), трехкислотные (Al(OH)3,
Fe(OH)3).
По силе основания делятся на сильные и слабые. К сильным относятся все щелочи, кроме гидроксида аммония.
Кислоты – сложные вещества, состоящие из водорода, способного
замещаться металлом, и кислотного остатка, причем число атомов водорода равно валентности кислотного остатка (табл.1). Кислоты классифицируются по основности, по наличию кислорода в составе кислоты и по их
силе.
Основностью кислоты называется число атомов водорода в молекуле кислоты, способных замещаться на металл с образованием соли. Соляная НСl и азотная HNO3 кислоты – примеры одноосновных кислот, серная
кислота H2SO4 – двухосновной, ортофосфорная кислота Н3РО4 – трехосновной.
По наличию кислорода в своем составе кислоты делятся на кислородсодержащие (HNO3 , H2SO4 и др.) и бескислородные ( НСl, H2S и др.).
Названия бескислородных кислот имеют окончание водородная: НСl
– хлороводородная (соляная кислота), H2S – сероводородная, HCN – циановодородная (синильная кислота).
Названия кислородсодержащих кислот также образуются от названия соответствующего элемента с добавлением слова кислота: HNO3 –
азотная, Н2CrO4 – хромовая кислота. Если элемент образует несколько
кислот, то название кислоты, в которой он проявляет высшую валентность,
оканчивается на -ная или -овая; если же валентность элемента ниже максимальной, то название кислоты оканчивается на -истая или -овистая
(например, НNO3 – азотная кислота, HNO2 – азотистая, Н3AsO4 – мышьяковая, H3AsO3 – мышьяковистая). Кроме того, одному и тому же оксиду
могут отвечать несколько кислот, различающихся между собой числом
молекул воды: наиболее богатая водой форма имеет приставку орто-, а
наименее богатая водой – приставку мета-. Так, кислота Н3РО4, в которой
на одну молекулу фосфорного ангидрида Р2О5 приходится три молекулы
воды, называется ортофосфорная Н3РО4, а кислота НРО3 – метафосфорная
(в ней на одну молекулу Р2О5 приходится одна молекула воды).
По силе кислоты делятся на сильные и слабые. Сильные кислоты –
азотная, серная и соляная, бромоводородная, иодоводородная, марганцовая, хлорная, хлорноватая и др. Слабые кислоты – сероводородная, циановодородная, фтороводородная, борная, угольная, фосфорная, азотистая,
фосфористая, сернистая, хлористая, хлорноватистая и др.
8
Соли – продукты замещения водорода в кислоте на металл или гидроксогрупп в основании на кислотный остаток (табл.1). При полном замещении получаются средние (нормальные) соли (например, сульфат калия
K2SO4). Кислая соль получается при неполном замещении водорода кислоты на металл (например, гидросульфат калия KНSO4). Основная соль получается при неполном замещении гидроксогрупп основания на кислотный
остаток (например, гидроксохлорид кальция Ca(ОН)С1). Соли, образованные двумя металлами и одной кислотой, называются двойными солями
(например, алюмокалиевые квасцы или сульфат калия-алюминия,
KAl(SO4)2). Соли, образованные одним металлом и двумя кислотами –
смешанные соли (например, фторид-хлорид свинца (II) PbFCl, хлоридгипохлорит кальция Ca(ClO)Cl).
Одна и та же соль может называться по-разному, например, KNO3
называют калиевой селитрой, азотнокалиевой солью, азотнокислым калием, нитратом калия. Наиболее широко применяется международная номенклатура, в которой название соли отражает название металла и латинское название кислотного остатка, которое происходит от латинского
названия элемента, образующего кислоту. Название соли бескислородной
кислоты имеет окончание -ид. Соль кислородсодержащей кислоты в случае максимальной валентности кислотообразующего элемента имеет окончание -aт, а в случае более низкой его валентности окончание -ит. Для
солей, образованных металлами с переменной валентностью, валентность
металла указывают в скобках (FeSO4 – сульфат железа (II), Fe2(SO4) –
сульфат железа (III)).
Название кислой соли имеет приставку гидро-, указывающую на
наличие незамещенных атомов водорода; если таких незамещенных атомов два или больше, то их число обозначается греческими числительными
ди-, три- и т.д. (Na2HPO4 – гидрофосфат натрия, NaH2PO4 – дигидрофосфатм натрия).
Основная соль имеет приставку гидроксо-, указывающую на наличие
незамещенных гидроксильных групп (AlOHCl2 – хлорид гидроксоалюминия, Аl(ОН)2С1 – хлорид дигидроксоалюминия или основной хлорид алюминия).
Кислая соль может быть образована только кислотой, основность которой равна двум или больше, а основная соль – металлом, валентность
которого равна двум или больше:
Са(ОН)2 + H2SO4 = 2Н2О + СаSO4 (средняя соль);
КОН + H2SO4 = Н2О + KHSO4 (кислая соль);
Mg(OH)2 + HC1 = Н2О + MgOHCl (основная соль).
9
Таблица 1
Формулы и названия кислот и солей
Название кислоты
Формула
Названия солей
Азотная
HNO3
Нитраты
Азотистая
HNO2
Нитриты
Кремниевая
H2SiО3
Силикаты
Марганцовая
HMnO4
Перманганаты
Метафосфорная
НРО3
Метафосфаты
Ортофосфорная
Н3РО4
Ортофосфаты (фосфаты)
Фосфористая
H3PO3
Фосфиты
Мышьяковая
H3AsO4
Арсенаты
Мышьяковистая
H3AsO3
Арсениты
Серная
H2SO4
Сульфаты
Сернистая
H2SO3
Сульфиты
Угольная
Н2СО3
Карбонаты
Хромовая
H2CrO4
Хроматы
Дихромовая
H2Cr2O7
Дихроматы
Бромоводородная
НВr
Бромиды
Иодоводородная
HI
Иодиды
Фтороводородная (плавиковая)
HF
Фториды
Циановодородная (синильная)
HCN
Цианиды
Хлороводородная (соляная)
HCl
Хлориды
Хлорная
HСlO4
Перхлораты
Хлорноватая
HСlO3
Хлораты
Хлористая
НClO2
Хлориты
Хлорноватистая
HClO
Гипохлориты
10
Важнейшие химические свойства и способы получения оксидов,
оснований, кислот, амфотерных гидроксидов и солей
Химические свойства основных оксидов:
основный оксид + кислота → соль + Н2О;
основный оксид + кислотный оксид → соль;
оксид щелочного или щелочноземельного металла + Н2О → щелочь.
Способы получения основных оксидов:
Металл + О2 → основный оксид;
нерастворимое основание → основный оксид + Н2О (при t).
Химические свойства кислотных оксидов:
кислотный оксид + основание → соль + Н2О;
кислотный оксид + основный оксид → соль;
кислотный оксид + Н2О → кислота (кроме SiO2).
Способы получения кислотных оксидов:
неметалл + О2 → кислотный оксид;
нерастворимая кислота → кислотный оксид + Н2О (при t).
Химические свойства амфотерных оксидов:
амфотерный оксид + кислота → соль + Н2О:
ZnO+ 2HCl → ZnCl2+ 2Н2О;
амфотерный оксид + тв. щелочь (при t ) → нормальная соль и Н2О:
ZnO + 2KOH → K2ZnO2+ Н2О (при t);
Al2O3+ 2NaOH→ 2NaAlO2+ Н2О (при t);
амфотерный оксид + щелочь (р-р) + Н2О → комплексная соль (гидроксокомплекс):
ZnO + 2NaOH + Н2О → Na2[Zn(OH)4];
Al2O3+ 2NaOH + 3Н2О → 2Na[Al(OH)4].
Способы получения амфотерных оксидов:
амфотерный металл (Zn, A1 и др.) + О2 → амфотерный оксид;
амфотерный гидроксид → амфотерный оксид + Н2О (при t).
Химические свойства растворимых оснований (щелочей):
щелочь + кислота → соль + Н2О;
щелочь + кислотный оксид → соль + Н2О;
щелочь + амфотерный оксид → соль + Н2О;
щелочь + амфотерный гидроксид → соль + Н2О;
щелочь (р-р) + нормальная соль → новое основание + новая соль;
щелочь (р-р) + кислая соль (р-р) → нормальная соль + Н2О.
Способы получения щелочей:
щелочной или щелочноземельный металл + Н2О → щелочь + Н2 ↑;
оксид щелочного или щелочноземельного металла + Н2О → щелочь;
щелочь (р-р) + соль (р-р) → новая соль↓ + щелочь.
11
Химические свойства нерастворимых оснований:
нерастворимое основание + кислота (р-р) → соль + Н2О;
нерастворимое основание → основный оксид + Н2О (при t).
Способ получения нерастворимых оснований:
нормальная соль + щелочь (р-р)→ основание↓+ новая соль.
Химические свойства амфотерных гидроксидов:
амфотерный гидроксид + кислота → соль + Н2О:
Zn(OH)2+ 2HCl→ ZnCl2+ 2Н2О;
амфотерный гидроксид + тв. щелочь (при t) → нормальная соль и Н2О:
Zn(OH)2+ 2KOH → K2ZnO2+ 2Н2О (сплавление);
цинкат калия
Al(OH)3+ NaOH→ NaAlO2+ Н2О (при t);
(недостаток) метаалюминат натрия
Al(OH)3+ 3NaOH→ Na3AlO3+ 3Н2О (при t);
(избыток)
ортоалюминат натрия
амфотерный гидроксид + щелочь (р-р) → комплексная соль (гидроксокомплекс):
Zn OH)2+ 2KOH → K2[Zn(OH)4];
тетрагидроксоцинкат калия
Al(OH)3+ NaOH → Na[Al(OH)4];
(недостаток) тетрагидроксоалюминат натрия
Al(OH)3+ 3NaOH → Na3 [Al(OH)6].
(избыток)
гексагидроксоалюминат натрия
Способы получения амфотерных гидроксидов:
нормальная соль(р-р) + щелочь(р-р) → амфотерный гидроксид+новая соль.
Химические свойства кислот:
кислота + основание → соль + Н2О;
кислота + основный оксид → соль + Н2О;
кислота + амфотерный гидроксид → соль + Н2О;
кислота + амфотерный оксид → соль + Н2О;
кислота + основная соль → нормальная соль + Н2О;
кислота + нормальная соль → новая кислота + новая соль;
кислота + металл (активный) → соль + Н2↑.
Способы получения кислот:
неметалл (Cl2, S и др.) + Н2 → водородное соединение неметалла (при растворении в Н2О→ бескислородная кислота);
кислотный оксид (кроме SiO2) + Н2О → кислородсодержащая кислота;
Химические свойства солей:
нормальная соль (р-р) + щелочь (р-р) → новая соль+ новое основание;
нормальная соль + кислота→ новая соль + новая кислота;
соль(1) р-р + соль(2) р-р → соль (3) + соль (4);
12
соль(1) р-р+металл(1)→ соль(2) + металл(2) , (металл(1) активнее металла(2)).
Способы получения солей:
кислота + основание → соль + Н2О;
кислота + основный оксид → соль + Н2О;
основание + кислотный оксид → соль + Н2О;
основный оксид + кислотный оксид → соль;
соль(р-р) + щелочь(р-р) → соль + основание;
соль(1) + кислота(1) →соль(2) + кислота(2);
соль(1) + соль(2) → соль(3) + соль(4);
соль(1)р-р + металл(1)→ соль(2) + металл(2), (металл(1) активнее металла(2));
кислота (р-р) + металл (активный) → соль + Н2↑;
металл + неметалл → соль.
(При условии протекания реакций в растворах до конца (необратимо), т.е. при
образовании осадка, выделении газа или Н2О).
Химические реакции. Классификация химических реакций
Химические реакции (химические явления) – это процессы, в результате которых из одних веществ образуются другие, отличающиеся от
исходных по составу или строению. При протекании химических реакций
не происходит изменения числа атомов того или иного элемента, взаимопревращения изотопов.
Классификация химических реакций многопланова, в ее основу могут быть положены различные признаки: число и состав реагентов и продуктов реакции, тепловой эффект, обратимость и др.
I. Классификация реакций по числу и составу
реагирующих веществ
А. Реакций, протекающие без изменения качественного состава
вещества. Это многочисленные аллотропные превращения простых веществ (например, кислород ↔ озон (3О2↔2О3), белое олово ↔ серое олово); переход при изменении температуры некоторых твердых веществ из
одного кристаллического состояния в другое – полиморфные превращения
(например, красные кристаллы иодида ртути (II) при нагревании превращаются в вещество желтого цвета того же состава, при охлаждении протекает обратный процесс); реакции изомеризации (например, NH4OCN↔
(NH2)2CO) и др.
Б. Реакции, протекающие с изменением состава реагирующих
веществ.
Реакции соединения – это реакции, при которых из двух или более
исходных веществ образуется одно новое сложное вещество. Исходные
вещества могут быть как простыми, так и сложными, например:
4Р + 5О2 = 2Р2О5 ; 4NO2 + О2 + 2Н2О = 4HNO3 ; СаО+ Н2О =Са(ОН)2.
13
Реакции разложения – это реакции, при которых из одного исходного сложного вещества образуется два или более новых вещества. Вещества, образующиеся в реакциях такого типа могут быть как простыми, так
и сложными, например:
2HI = Н2 + I2; СаCO3=СаО+ CO2; (CuOH)2CO3 = CuO + H2O + CO2 .
Реакции замещения – это процессы, в которых атомы простого вещества замещают атомы какого-нибудь элемента в сложном веществе. Поскольку в реакциях замещения в качестве одного из реагентов обязательно
участвует простое вещество, практически все превращения такого типа являются окислительно-восстановительными, например:
Zn + H2SO4= H2 + ZnSO4 ; 2Al + Fe2O3 = 2Fe + Al2O3; H2S + Br2 = 2HBr + S.
Реакции обмена – это реакции, при которых два сложных вещества
обмениваются своими составными частями. Реакции обмена могут протекать непосредственно между двумя реагентами без участия растворителя,
например: H2SO4 + 2КОН = K2SO4 + 2Н2О ; SiО2(тв) + 4HF(г)=SiF4+ 2Н2О.
Реакции обмена, протекающие в растворах электролитов, называют
реакциями ионного обмена. Такие реакции возможны лишь в том случае,
если одно из образующихся веществ является слабым электролитом, выделяется из сферы реакции в виде газа или труднорастворимого вещества
(правило Бертолле):
AgNO3 + HCl = AgCl↓ + HNO3, или Ag+ + Cl- = AgCl↓;
NH4Cl + КОН = KCl + NH3↑ + H2O, или NH4+ + OH- = H2O + NH3↑;
NaOH + HCl = NaCl + H2O, или Н+ + OH- = H2O.
II. Классификация реакций по тепловому эффекту
А. Реакции, протекающие с выделением тепловой энергии – экзотермические реакции (+ Q).
Б. Реакции, протекающие с поглощением теплоты – эндотермические реакции (– Q).
Тепловым эффектом реакции называют количество теплоты, которое выделяется или поглощается в результате химической реакции. Уравнение реакции, в котором указан ее тепловой эффект, называют термохимическим. Значение теплового эффекта реакции удобно приводить в расчете на 1 моль одного из участников реакции, поэтому в термохимических
уравнениях часто можно встретить дробные коэффициенты:
1/2N2(г) + 3/2Н2(г) = NH3(г) + 46,2 кДж /моль.
Экзотермическими являются все реакции горения, подавляющее
большинство реакций окисления и соединения. Реакции разложения, как
правило, требуют затрат энергии.
14
III. Классификация реакций по фазовому составу веществ
Фазой называют однородную по составу и свойствам часть системы, которая отделена от других фаз поверхностью (границей) раздела. По
количеству фаз, которые образуют реагенты и продукты, все химические
реакции подразделяют на гомогенные и гетерогенные.
В гомогенных (однофазных) реакциях исходные вещества и продукты реакции находятся в одной фазе:
2СО(г) + О2(г) = 2СО2(г);
СН4(г) + С12(г) = CH3Cl(г) + HCl (г);
NaOH(p-p) + HCl (p-p) = NaCl (p-p) + Н2О (ж);
Если хотя бы один из участников реакции (включая катализатор) находится в иной фазе по сравнению со всеми остальными, реакцию называют гетерогенной (многофазной):
Zn(к)+ HCl (p-p) = ZnCl2 (p-p) + Н2(г);
2SО2(г) + О2(г) ↔ 2SО3(г), катализатор V2O5 (тв);
Реакции в гетерогенных системах протекают на границе раздела фаз,
поэтому на их скорость очень существенное влияние оказывает степень
измельчения твердых веществ.
IV. Классификация химический реакций
по участию катализатора
А. Каталитические реакции – это реакции, которые протекают с
участием катализаторов:
2SО2 + О2 ↔ 2SО3 , (катализатор V2O5 );
N2 + 3Н2 ↔ 2NH3 , (катализатор восстановленное Fe).
Б. Некаталитические реакции – это реакции, которые протекают
без участия катализаторов:
ВaO+ СО2 = ВaСО3; 2NO + О2 = 2NO2;
Na2SО4 + CaCl2= CaSО4↓ + 2NaCl.
V. Классификация реакций по обратимости
А. Необратимые химические реакции – реакции, продукты которых
не могут взаимодействовать с образованием исходных веществ, т.е. реакции, которые в данных условиях могут протекать только в одном направлении:
CaO+ СО2 = CaСО3 ; С + О2 = СО2;
Вa(OH)2 + H2SО4 = ВaSО4 ↓ + 2Н2О;
Na2СО3 + 2HCl = 2NaCl + СО2↑+ Н2О.
Б. Обратимые химические реакции – реакции, которые протекают
в данных условиях как в прямом, так обратном направлениях.
N2 + 3Н2 ↔ 2NH3.
15
VI. Классификация реакций по изменению
степеней окисления атомов
А. Реакции, протекающие без изменения степеней окисления химических элементов. К данному типу относят реакции ионного обмена,
многие реакции разложения и соединения (если среди исходных веществ и
продуктов реакции нет простых веществ):
СаСО3 = СаО + СО2 ; ВаО + Н2О = Ва(ОН)2;
2NaOH + H2SO4 = Na2SO4 + 2Н2О.
Б. Реакции, протекающие с изменением степеней окисления химических элементов (окислительно-восстановительные реакции).
Протекание химических реакций в целом обусловлено обменом частицами между реагирующими веществами. Часто обмен сопровождается
переходом электронов от одной частицы к другой. Так, при вытеснении
цинком меди в растворе сульфата меди (II):
Zn(т) +CuSO4(р)=ZnSO4(p)+Cu(т)
электроны от атомов цинка переходят к ионам меди:
Zn0= Zn2++ 2e,
Cu2+ + 2e = Cu0 ,
или суммарно: Zn0 + Cu2+= Zn2+ + Cu0.
Процесс потери электронов частицей называют окислением, а процесс приобретения электронов – восстановлением. Окисление и восстановление протекают одновременно, поэтому взаимодействия, сопровождающиеся переходом электронов от одних частиц к другим, называют
окислительно-восстановительными реакциями (ОВР).
Для удобства описания ОВР используют понятие степени окисления – величины, численно равной формальному заряду, который приобретает элемент, исходя из предположения, что все электроны каждой из его
связи перешли к более электроотрицательному атому данного соединения.
Протекание ОВР сопровождается изменением степеней окисления элементов участвующих в реакции веществ. При восстановлении степень
окисления элемента уменьшается, при окислении – увеличивается. Вещество, в состав которого входит элемент, понижающий степень окисления,
называют окислителем; вещество, в состав которого входит элемент, повышающий степень окисления, называют восстановителем.
Степень окисления элемента в соединении определяют в соответствии со следующими правилами:
1) степень окисления элемента в простом веществе равна нулю;
2) алгебраическая сумма всех степеней окисления атомов в молекуле
равна нулю;
3) алгебраическая сумма всех степеней окисления атомов в сложном
ионе, а также степень окисления элемента в простом одноатомном
ионе равна заряду иона;
16
4) отрицательную степень окисления проявляют в соединении атомы
элемента, имеющего наибольшую электроотрицательность;
5) максимально возможная (положительная) степень окисления элемента соответствует номеру группы, в которой расположен элемент в
Периодической таблице Д.И. Менделеева.
Ряд элементов в соединениях проявляют постоянную степень окисления:
1) фтор, имеющий наивысшую среди элементов электроотрицательность, во всех соединениях имеет степень окисления –1;
2) водород в соединениях проявляет степень окисления +1, кроме гидридов металлов (–1);
3) металлы IA подгруппы во всех соединениях имеют степень окисления +1;
4) металлы IIA подгруппы, а также цинк и кадмий во всех соединениях
имеют степень окисления +2;
5) степень окисления алюминия в соединениях +3;
6) степень окисления кислорода в соединениях равна –2, за исключением соединений, в которых кислород присутствует в виде молекулярных ионов: О2+, О2, О22, О3, а также фторидов OxF2.
Степени окисления атомов элементов в соединении записывают над
символом данного элемента, указывая вначале знак степени окисления, а
затем ее численное значение, например, K+1Mn+7O4-2, в отличие от заряда
иона, который записывают справа, указывая вначале зарядовое число, а затем знак: Fe2+, SO42–.
Окислительно-восстановительные свойства атомов различных элементов проявляются в зависимости от многих факторов, важнейшие из которых – электронное строение элемента, его степень окисления в веществе,
характер свойств других участников реакции.
Соединения, в состав которых входят атомы элементов в своей максимальной (положительной) степени окисления, например, K+1Mn+7O4-2,
K2+1Cr+62O7-2, H+N+5O3-2, Pb+4O2-2, могут только восстанавливаться, выступая в качестве окислителей.
Соединения, содержащие элементы в их минимальной степени окисления, например, N-3H3, H2S-2, HI-1, могут только окисляться и выступать в
качестве восстановителей.
Вещества, содержащие элементы в промежуточных степенях окисления, например H+N+3O2, H2O2-1, S0, I20, Cr+3Cl3, Mn+4O2-2, обладают окислительно-восстановительной двойственностью. В зависимости от партнера по реакции, такие вещества способны и принимать, и отдавать электроны. Состав продуктов восстановления и окисления также зависит от
многих факторов, в том числе среды, в которой протекает химическая реакция, концентрации реагентов, активности партнера по окислительновосстановительному процессу. Чтобы составить уравнение окислительно17
восстановительной реакции, необходимо знать, как изменяются степени
окисления элементов, в какие другие соединения переходят окислитель и
восстановитель.
Классификация окислительно-восстановительных реакций. Различают четыре типа окислительно-восстановительных реакций.
1. Межмолекулярные – реакции, в которых окислитель и восстановитель – разные вещества: Zn0 +Cu+2SO4 =Zn+2SO4 +Cu0.
2. При термическом разложении сложных соединений, в состав которых входят окислитель и восстановитель в виде атомов разных элементов, происходят окислительно-восстановительные реакции, называемые
внутримолекулярными: (N-3H4)2Cr+62O7= N20↑ + Cr+32O3 + 4H2O.
3. Реакции диспропорционирования могут происходить, если соединения, содержащие элементы в промежуточных степенях окисления, попадают в условия, где они оказываются неустойчивыми (например, при повышенной температуре). Степень окисления этого элемента и повышается
и понижается: 2H2O2-1= O02↑ + 2 H2O-2.
4. Реакции контрпропорционирования – это процессы взаимодействия окислителя и восстановителя, в состав которых входит один и тот же
элемент в разных степенях окисления. В результате продуктом окисления
и продуктом восстановления является вещество с промежуточной степенью окисления атомов данного элемента:
Na2S+4O3 + 2Na2S-2 + 6HCl = 3S0+ 6NaCl + 3H2O.
Существуют также реакции смешанного типа. Например, к внутримолекулярной реакции контрпропорционирования относится реакция разложения нитрата аммония: N-3H4 N+5O3 = N+12O + 2H2O.
Составление уравнений окислительно-восстановительных реакций. Для составления уравнений окислительно-восстановительных реакций наиболее часто используют метод электронного баланса и метод электронно-ионных полуреакций.
Метод электронного баланса обычно используют для составления
уравнений окислительно-восстановительных реакций, протекающих между газами, твердыми веществами и в расплавах. Последовательность операций следующая:
1. Записывают формулы реагентов и продуктов реакции в молекулярном
виде:
FeCl3 + H2S → FeCl2 + S + HCl;
2. Определяют степени окисления атомов, меняющих ее в процессе реакции:
Fe3+Cl3 + H2S-2 → Fe2+Cl2 + S0 + HCl;
3. По изменению степеней окисления устанавливают число электронов,
отдаваемых восстановителем, и число электронов, принимаемых окислителем; составляют электронный баланс с учетом принципа равенства
числа отдаваемых и принимаемых электронов:
Fe+3 +1e = Fe+2∙2
18
S-2 – 2e = S0 ∙1
4. Множители электронного баланса записывают в уравнение окислительно-восстановительной реакции как основные стехиометрические коэффициенты: 2FeCl3 + H2S → 2FeCl2 + S + HCl.
5. Подбирают стехиометрические коэффициенты остальных участников
реакции: 2FeCl3 + H2S = 2FeCl2 + S + 2HCl.
Метод электронно-ионных полуреакций применяют при составлении уравнений реакций, протекающих в водном растворе, а также реакций
с участием веществ, в которых трудно определить степени окисления элементов. Согласно этому методу выделяют следующие главные этапы составления уравнения реакций:
1. Записывают общую молекулярную схему процесса с указанием восстановителя, окислителя и среды, в которой протекает реакция (кислотная,
нейтральная или щелочная). Например:
SO2 + K2Cr2O7 + H2SO4(разб.) → ...
2. Учитывая диссоциацию электролитов в водном растворе, данную схему
представляют в виде молекулярно-ионного взаимодействия. Ионы, степени окисления атомов которых не изменяются, в схеме не указывают,
за исключением ионов Н+ и ОН:
SO2 + Cr2O72– + H+ → ...
3. Определяют степени окисления восстановителя и окислителя, а также
продуктов их взаимодействия:
окисление восстановителя:
восстановление окислителя:
+4
+6
2–
S O2 → (S O4)
(Cr+62O7)2–→ 2Cr3+
4. Записывают материальный баланс полуреакции окисления и восстановления:
окисление восстановителя:
восстановление окислителя:
2–
+
SO2+ 2H2O – 2e → SO4 + 4H
Cr2O72– + 14H+ + 6e → 2Cr3+ + 7H2O
5. Суммируют полуреакции, учитывая принцип равенства отданных и
принятых электронов:
SO2 + 2H2O – 2e = SO42–+ 4H+
∙3
2–
+
3+
Cr2O7 + 14H + 6e = 2Cr + 7H2О ∙1
3SO2+ 6H2O + Cr2O72– + 14H+ = 3SO42– + 12H+ + 2Cr3+ + 7H2О
сокращая одноименные частицы, получают общее ионно-молекулярное
уравнение:
3SO2+ Cr2O72–+ 2H+ = 3SO42–+ 2Cr3+ + H2О.
6. Добавляют ионы, не участвовавшие в процессе окислениявосстановления, уравнивают их количества слева и справа, записывают
молекулярное уравнение реакции:
3SO2 + K2Cr2O7 + H2SO4 (разб) = Cr2(SO4)3 + K2SO4 + H2O.
При составлении материального баланса полуреакций окисления и
восстановления, когда изменяется число атомов кислорода, входящих в состав частиц окислителя и восстановителя, следует учитывать, что в водных
19
растворах связывание или присоединение кислорода происходит с участием молекул воды и ионов среды.
В процессе окисления на один атом кислорода, присоединяющийся к
частице восстановителя, в кислотной и нейтральной средах расходуется
одна молекула воды и образуются два иона Н+; в щелочной среде расходуются два гидроксид-иона ОН и образуется одна молекула воды.
В процессе восстановления для связывания одного атома кислорода
частицы окислителя в кислотной среде расходуются два иона Н+ и образуется одна молекула воды; в нейтральной и щелочной средах расходуется
одна молекула Н2О и образуются два иона ОН(табл.2).
Таблица 2
Баланс атомов кислорода
в окислительно-восстановительных реакциях
число атомов киссреда
лорода в исходкислая
нейтральная
щелочная
ных веществах
избыток
O2– + 2Н+→ Н2О O2–+H2О→2OH O2–+H2О→2OH
недостаток
Н2О→ O2– + 2Н+ Н2О→ O2– + 2Н+ 2OH→O2–+Н2О
При составлении уравнений следует учитывать, что окислитель (или
восстановитель) могут расходоваться не только в основной окислительновосстановительной реакции, но и при связывании образующихся продуктов реакции, т.е. выступать в роли среды и солеобразователя. Примером,
когда роль среды играет окислитель, служит реакция окисления металла в
азотной кислоте:
3Cu + 2HNO3(окислитель) + 6HNO3(среда) = 3Cu(NO3)2 + 2NO + 4H2O
или 3Cu + 8HNO3(разб) = 3Cu(NO3)2 + 2NO + 4H2O.
Примером, когда восстановитель является средой, в которой протекает реакция, служит реакция окисления соляной кислоты дихроматом калия: 6HCl(вос-тель) + K2Cr2O7 + 8HCl(среда) = 2CrCl3 + 3Cl2 +2KCl + 7H2O
или 14HCl + K2Cr2O7 = 2CrCl3 + 3Cl2 +2KCl + 7H2O.
При расчете количественных, массовых и объемных соотношений
участников окислительно-восстановительных реакций, используют основные стехиометрические законы химии, и, в частности, закон эквивалентов,
учитывая, что число эквивалентности окислителя равно числу электронов,
которые принимает одна формульная единица окислителя, а число эквивалентности восстановителя равно числу электронов, которые отдает одна
формульная единица восстановителя.
Примеры решения типовых задач
Задача 1. Запишите ионно-молекулярные уравнения реакций взаимодействия водных растворов следующих соединений: а) NaOH и H2SO4;
20
б) FeS и HCl; в) CuSO4 и Na2S; г) CH3COOH и KOH; д) CuSO4 и NaOH.
Решение:
1. Записываем уравнения соответствующих реакций (а, б, в, г, д) в
виде молекул и расставляем коэффициенты.
2. Записываем эти же уравнения, но сильные электролиты пишем в
форме ионов, а слабые электролиты, труднорастворимые соединения, газообразные вещества – в форме молекул.
3. Исключаем из левой и правой частей уравнений одинаковые ионы
и получаем краткое ионное уравнение, выражающее сущность данной реакции.
4. Проверяем запись уравнения по равенству сумм электрических зарядов в левой и правой частях уравнения.
Ионообменные реакции могут протекать обратимо и необратимо.
Равновесие, которое устанавливается при взаимодействии растворов электролитов, смещается в направлении образования труднорастворимого вещества, газа, слабого электролита, комплексного иона.
а) 2NaOH + H2SO4 = Na2SO4 + H2O;
2Na+ + 2OH+ 2H++ SO42- = 2Na+ + SO42-+ H2O;
H+ + OH- = H2O.
б) FeS + 2HCl = FeCl2 + H2S↑;
FeS+2H+ + 2Cl- = Fe2+ + 2Cl- + H2S↑;
FeS + 2H+ = Fe2++ H2S↑.
в) CuSO4 + Na2S = CuS↓+ Na2SO4;
Cu2+ + SO42- + 2Na+ + S2-= CuS↓+ 2Na+ + SO42-;
Cu2-+S2-=CuS↓.
г) CH3COOH + KOH = CH3COOK+H2O;
CH3COOH + K++OH- = CH3COO-+K++H2O;
CH3COOH + OH- = CH3COO- +H2O.
д) CuSO4 + 4NaOH = Na2[Cu(OH)4]+Na2SO4;
Cu2+ + SO42- + 4Na+ + 4OH- = 2Na+ + [Cu(OH)4]2-+ 2Na+ + SO42-;
Cu2+ + 4OH- = [Cu(OH)4]2-.
Задача 2. Запишите следующие ионно-молекулярные уравнения реакций в молекулярной форме:
а) 3Ca2++2PO43- =Ca3(PO4)2;
б) Ba2++ SO42- = BaSO4;
в) CaCO3+2H+=Ca2++H2O+CO2;
г) H++OH- = H2O;
д) Cu2++4NH4OH=[Cu(NH3)4]2++4H2O.
Решение: используя данные таблицы растворимости солей и оснований в воде, записываем уравнения соответствующих реакций в молекулярной форме:
а) 3Ca(NO3)2+2Na3PO4= Ca3(PO4)2↓+6NaNO3;
б) BaCl2 + Na2SO4 = BaSO4↓ + 2NaCl;
в) CaCO3 + 2HCl = CaCl2 + H2O + CO2;
г) HCl + NaOH = NaCl + H2O;
21
д) CuSO4 + 4NH4OH = [Cu(NH3)4]SO4+4H2O.
Если при смешении растворов электролитов не образуются осадки,
газообразные вещества, слабые электролиты, то химическое взаимодействие не происходит, а в растворе находится лишь смесь ионов.
Задача 3. Определите степень окисления хлора в соединениях NaCl,
NaClO, NaClO3, NaClO4 и объясните, какое из них является только окислителем, только восстановителем, а какие могут проявлять как окислительные, так и восстановительные свойства.
Решение: вычислим степень окисления хлора в этих соединениях,
исходя из электронейтральности молекулы и зная, что степень окисления
кислорода равна –2, а натрия +1:
+1
-1 +1 +1 -2
+1 +5 -2
+1 +7 -2
NaCl, NaClO, NaClO3, NaClO4.
Вещество выполняет только восстановительную функцию, то есть
является безусловным восстановителем в том случае, когда его молекула
содержит атом, находящийся в низшей степени окисления, и возможен
только процесс отдачи электронов. Для неметаллов минимальное значение
степени окисления соответствует числу электронов, недостающих до завершения внешнего энергетического уровня, со знаком «минус»: 8 – N, где
N – номер группы периодической системы, в которой находится этот элемент. Для хлора минимальная степень окисления равна –1, поэтому это
вещество может проявлять только восстановительные свойства за счет
атома хлора.
Атом элемента в высшей степени окисления способен только присоединять электроны и является только окислителем. Максимальная степень
окисления равна общему числу валентных электронов со знаком «+» или, в
общем виде, «+N». Для хлора значение максимальной степени окисления
соответствует +7. Поэтому NaClO4 может проявлять только окислительные
свойства. Соединения NaClO3 и NaClO содержат атомы хлора в промежуточных степенях окисления (+5 и +1 соответственно), поэтому в зависимости от условий они могут проявлять как восстановительные, так и окислительные свойства. Например:
NaCl+5O3+3H2SO3=NaCl-1+3H2SO4,
(NaClO3 – окислитель);
+5
+7
NaCl O3+I2+H2O=NaCl O4+2HI,
(NaClO3 – восстановитель).
Задача 4. С помощью метода электронного баланса расставьте коэффициенты в уравнении окислительно-восстановительной реакции:
Cr2(SO4)3+Cl2+KOH  K2CrO4+KCl+K2SO4+H2O.
Определите окислитель и восстановитель, запишите процессы окисления и восстановления.
Решение: определим степени окисления атомов всех элементов,
входящих в состав молекул реагирующих веществ; обратите внимание, что
реакция протекает в щелочной среде (KOH).
Cr+32(S+6O-24)3+Cl02+K+1O-2H+1→ K+2Cr+6O-24+K+1Cl-1+K+12S+6O-24+H+12O-2.
22
Выпишем атомы элементов, изменивших свои степени окисления, и определим число отданных и присоединённых электронов:
2Сr+3 – 6 e → 2Cr+6 23 – процесс окисления, Сr+3 - восстановитель;
Cl02+ 2 e → 2Cl-1 61 – процесс восстановления, Cl02 – окислитель.
Полученные коэффициенты расставляем в уравнении перед соответствующими молекулами, а остальные коэффициенты подбираем обычным способом, исходя из равенства количества атомов в левой и правой частях. В
последнюю очередь проверяем число атомов кислорода в левой и правой
частях уравнения:
Сr2(SO4)3+3Cl2+16KOH=2K2CrO4+6KCl+3K2SO4+8H2O.
1.
2.
3.
4.
5.
6.
Литература
Глинка Н. Л. Общая химия. – М.: Химия, 2006. – 720 с.
Глинка Н.Л. Задачи и упражнения по общей химии. – Л. Химия, 1985. –
264 с.
Карапетьянц М.Х., Дракин С.И. Общая и неорганическая химия. – М.:
Химия, 1993. – 558 с.
Коровин В.Н. Общая химия. – М.: Высшая школа, 2000. – 557 с.
Коровин Н.В., Масленникова Г. Н. и др. Курс общей химии. – М.:
Высшая школа, 1990. – 445 с.
Суворов А.В., Никольский А.Б. Общая химия. – СПб: Химиздат, 2001.
– 512 с.
Задачи и упражнения для самостоятельной работы
1.
2.
3.
4.
5.
6.
7.
Тема: Основные понятия химии.
Современная номенклатура неорганических веществ
Какие вещества называются оксидами? Приведите примеры основных,
кислотных и амфотерных оксидов.
Какие вещества называются основаниями, какие – кислотами? Как образуются нерастворимые в воде основания? Составьте соответствующие уравнения реакций.
Какие кислоты могут быть получены непосредственным взаимодействием с водой оксидов: Р2О5, СО2, N2O5, NO2, SO2?
Как определяют основность кислоты? Как определяют кислотность основания?
Какие вещества называются солями? Приведите примеры солей, относящихся к различным классам, дайте им названия.
Какие из перечисленных кислот образуют кислые соли: HI, H2Se,
H2SeО 3, Н2С2О4, H3РО4, НNO3?
Какие гидроксиды называются амфотерными? Как доказать амфотерный характер ZnO, A12O3, Sn(OH)2, Сr(ОН)3? Составьте соответствующие уравнения химических реакций.
23
8. Напишите формулы высших оксидов и гидроксидов следующих элементов: Cs, Ba, La, Ga, Ta, In, J, Pt, Co, Fe. Укажите их характер.
9. Напишите формулы ангидридов указанных кислот: H2SO3, H3BO3,
Н4Р2О7, НОС1, НМnО4.
10. Ангидридом какой кислоты можно считать Сl2О7: а) хлорной; б)
хлорноватой; в) хлорноватистой?
11. Напишите формулы оксидов, соответствующих указанным гидроксидам: Cu(OH)2, H2SiО3, H3AsО4, H2WO4, Fe(OH)3.Укажите характер оксидов.
12. Какие из указанных гидроксидов могут образовать основные соли: а) Сu(ОН)2; б) Са(ОН)2; в) LiOH; г) А1(ОН)3; д) КОН?
13. Составьте формулы нормальных и кислых солей калия и кальция, образованных: а) угольной кислотой; б) мышьяковистой кислотой.
14. Назвите соли: a) Zn(NO3)2; б) NaH2SbО4; в) К2Н2Р2О7; г) A1(OH)2NO3;
д) СаСrО4; е) NaHS ж) Na2Cr2О7; з) Ba(HSO3)2; и) CrOHSO4; к)
(СuОН)2СО3.
15. Какие из приведенных соединений относятся к пероксидам:
a) NO2; б) К2О2; в) ВаО2; г) МnО2?
1.
2.
3.
4.
5.
6.
7.
8.
Тема: Важнейшие химические свойства и способы получения
оксидов, оснований, кислот, амфотерных гидроксидов и солей
Напишите уравнения реакций, свидетельствующих об основных
свойствах оксидов: FeO, Cs2O, HgO, Bi2O3.
Напишите уравнения реакций, доказывающих кислотный характер
SeO2, SO3, Mn2O7, P2O5, СrО3.
Какие вещества могут быть получены при взаимодействии кислоты с
солью, кислоты с основанием, соли с солью? Приведите примеры химических реакций.
Назовите различные способы получения солей и составьте соответствующие уравнения химических реакций.
Составьте уравнения химических между кислотами и основаниями,
приводящие к образованию солей: NaNO3, NaHSO4, Na2HPO4, K2S,
Fe2(SO4)3.
С какими из перечисленных ниже веществ будет реагировать соляная
кислота: N2O5, Zn(OH)2, CaO, AgNO3, H3PO4, H2SO4? Составьте уравнения химических реакций.
Какие из указанных веществ реагируют с гидроксидом калия: HNO3,
CaO, CO2, CuSO4, Cd(OH)2, Р2О5? Составьте уравнения химических
реакций.
Напишите уравнения реакций образования Mg2P2O7, Ca3(PO4)2,
Mg(C1О4)2, Ba(NO3)2 в результате взаимодействия: а) основного и кислотного оксидов; б) основания и кислотного оксида; в) основного оксида и кислоты; г) основания и кислоты.
24
9. Какие соли можно получить, имея в распоряжении CuSO4, AgNO3,
К3РО4, ВаС12? Напишите уравнения реакций и назовите полученные
соли.
10. Составьте уравнения химических реакций получения хлорида магния:
а) действием кислоты на металл; б) действием кислоты на основание; в) действием соли на соль.
11. Напишите уравнения реакций, с помощью которых можно получить в
лаборатории следующие вещества: а) хлороводород; б) сульфид свинца; в) сульфат бария; г) ортофосфат серебра; д) гидроксид железа (III);
е) нитрат меди (II).
12. Можно ли получить раствор, содержащий одновременно: а) Ва(ОН)2 и
НС1; б) СаС12 и Na2CO3; в) NaCl и AgNO3; г) КС1 и NaNO3. Укажите,
какие комбинации невозможны и почему? Напишите уравнения протекающих реакций.
13. При помощи каких реакций можно осуществить следующие переходы
от одного вещества к другому:
а) С → CО2 → H2CO3 → BaCO3 → BaCl2;
б) MgO → MgCl2 → Mg(OH)2 → MgOHNО3 → Mg(NO3)2;
в) Zn → ZnS → ZnO → (ZnOH)2SO4 → ZnO;
г) Cu(OH)2 → CuCl2 → ZnCl2 → ZnOHCl → ZnSO4;
д) CO2 → MgCO3 → Mg(HCO3)2 → MgCO3 → MgO.
14. Осуществите превращения согласно схемам:
а) K2S→ FeS → H2S→ S→ SO2→ K2SO3 → SO2;
б) Сu(NO3)2 → NO2→ HNO3 → Fe(NO3)2 → Fe(OH)2→ FeCl2;
в) NaCl → Na → NaOH → Na2CO3 → NaCl → AgCl;
г) K2CO3 → CO2 → СаСО3 → СаО → Са(ОН)2 → СаС12;
д) Al→ Al2O3 → AlCl3 → Al(OH)3 → NaАl(OH)4 → Al(NO3)3;
ж) Mn(OH)2 → MnCl2 → Mn → MnSO4 → Mn(OH)2→ MnO;
з) Zn→ ZnSO4 → Zn(OH)2 → ZnCl2 → ZnS → H2S;
и) NiCl2 → Ni → NiSO4 → Ni(OH)2 → Ni(NO3)2 → NiO.
15. Составьте уравнения возможных реакций:
1) Са(NO3)2 + K2CO3 →
7) Na2SiO3 +Ba(OH)2→
2) Ba(NO2)2 + K2SO4→
8) FeCl3 +CsOH→
3) K2CO3 +HCl→
9) FeSO4 +Na3PO4→
4) Na2CO3 + H2SO4→
10) Pb(NO3)2 + K2SO4→
5) NaOH +Fe(NO3)2→
11) Na2SiO3 +Ba(OH)2→
6) Pb(NO3)2 + K2SO4→
12) FeCl2 +LiOH→
Тема: Классификация химических реакций
1. Напишите уравнения реакций в водных растворах электролитов в молекулярном и ионном виде:
a) Pb(NO3)2+KI →
ж) AlBr3+AgNO3 →
б) NiCl2+H2S →
з) Na2S+H2SO4 →
25
2.
3.
4.
5.
6.
7.
8.
9.
в) K2CO3+HCl →
и) NH4Cl+Ca(OH)2 →
г) Na2SO3+H2SO4 →
к) NaClO+HNO3 →
д) CrSO4+NaOH →
л) FeS+HCl →
Составьте ионно-молекулярные уравнения следующих реакций:
a) Al2(SO4)3 + Pb(NO3)2 →
г) HCN + NaOH →
6) ZnS + HCl →
д) Сu(ОН)2 +Na2S→
в) Fe(OH)3+ HNO3 →
e) Cu(OH)2 + H2S →
Составьте молекулярные уравнения к следующим ионномолекулярным уравнениям:
а) Рb2+ + 2I-→
г) HCN + ОН-→
б) Са2+ + СО3 2-→
д) Сu2+ + 2ОН-→
в) NH4OH + Н+ →
е) CN- + Н+ →
Определите степень окисления элементов в следующих соединениях:
CrO3, Са(НСО3)2, CaCr2O7, Ca(MnO4)2, CaMnO4, K3[Fe(CN)6].
Определите степень окисления серы в следующих соединениях: SF6,
SO2Cl2, K2S2O7, KHSO4, H2S, KHS, K2S2O3, SOCl2 .
Определите степень окисления мышьяка в следующих соединениях:
Ca3As2, KAs(OH)6, KH2AsO3, KAsO2, Mg3(AsO4)2, AsCl3, AsH3, AsCl4,
AsOCl .
В предложенных соединениях определите степень окисления указанного элемента и укажите его роль в окислительно- восстановительных реакциях (окислитель, восстановитель; окислитель или восстановитель в
зависимости от условий):
а) Re: ReO2, Na2ReO3, Re, HReO4, ReF4, ReF7;
б) Si: SiCl2, SiO2, Na2SiO3, H2SiO3, SiH4;
в) U: UCl3, U(OH)2Cl2, UO2, UO3, U;
г) Pa: Pa2O5, (PaO2)2SO4, Pa, PaCl5, HPaO3.
Подберите коэффициенты в уравнениях химических реакций методами
электронного и электронно-ионного баланса:
а) Аg + HNO3 (конц.) →
б) Аg + HNO3 (разб.) →
в) Mg + HNO3 (конц.) →
г) Mg + HNO3 (разб.) →
д) Mg + HNO3 (очень разб.) →
В приведенных уравнениях окислительно-восстановительных реакций
определите окислитель и восстановитель, составьте уравнения электронного баланса, расставьте коэффициенты
а) NaOH +S = Na2S+Na2SO3;
б) BaCrO4 =BaO +Cr2O3 +O2;
в) Na2SO3 + KMnO4 +KOH = Na2SO4 + K2MnO4 +H2O;
г) FeSO4 + KMnO4+H2SO4 = Fe2(SO4)3 +MnSO4 + K2SO4 + H2O;
д) Na2SO3 + KMnO4 +H2O = Na2SO4 + MnO2 + KOH;
ж) H2S + K2Cr2O7 + H2SO4 = K2SO4 +Cr2(SO4)3 + H2O.
26
10. Напишите уравнения окислительно-восстановительных реакций, протекающих в растворах, подберите необходимые коэффициенты методом ионно-электронного баланса:
1) MnO4- +H+ +NaCl → Cl2 + Mn2+ +H2O +…
2) Cr(OH)4)- + Br2 + OH- → CrO42- +Br- +H2O+ …
3) Cr3+ + S2O82- + H2O → Cr2O72- + SO42- +H+ + …
4) NaOH +S → Na2S +Na2SO3 + ....
5) AgCl + Mn2+ +OH- → Ag+ MnO(OH)2 + Cl- + H2O
6) SnCl2 +H2O2 +H+ +Cl- → SnCl4 + H2O+....
Контрольная работа №1
Задание 1.
Осуществите превращения согласно схемам (табл.3) .
Таблица 3
№
варианта
1
2
3
4
5
6
7
8
Схемы химических процессов
а) Na→NaOH→NaHCO3→Na2CO3→Na2SO4→NaCl→Na
б) NH4NO2→N2→NO→NO2→HNO3→AgNO3→NO2→NO
а) Са→Са(ОН)2→СаС12→СаСО3→Са(НСО3)2→СаСО3→СаО→Са
б) FeS→Fe2O3→FeCl2→Fe(OH)3→Fe2O3→FeO→Fe→Fe(NO3)2→
→FeCl2
а) Na2О→NaBr→NaCl→Cl2→KC1О3→KCl→KОH→H2
б) Al→AlCl3→Al(OH)3→Al2O3→Al→Na[Al(OH)4] →A12(SO4)3→
→A1(NO3)3→A12O3.
а) С12→НС1→С12→КС1→С12→Са(ОС1)2→НОС1→НС1→СuС12
б) Al→Al(OH)3→Al2(SO4)3→AlCl3→Na[Al(OH)4] →NaA1O2→
→Al(OH)3→Al
а) Zn→ZnS→H2S→S→SO2→H2SO4→SO2→KHSO3→SO3→S.
б) MnO2→Cl2→KClO3→Cl2→HCl→MnCl2→Mn(OH)2→Mn.
а) HNO3→AgNO3→O2→NO2→HNO2→KNO2→KNO3→CO2
б) FeS2→SO2→S→Na2S→NaHS→H2S→ CuS→ CuO→ Cu(OH)2→
→CuSO4
а) N2→NH3→NO→HNO3→NH4NO3→NH3→NH4NO2→N2→Ca3N2
б) Cu→ CuCl2→ZnCl2→K2[Zn(OH)4] →ZnSO4→Zn(NO3)2→ZnO →
→Na2[Zn(OH)4]
а) СО→С→СО→СО2→СаСО3→Са(НСО3)2→СаО→CaC2→CO2
б) Cu→ Cu(NO3)2→ CuO→CuCl2→Cu(OH)2→ CuO→ Cu→
→CuSO4→H2SO4
27
9
а) Na→NaOH→Na2CO3→Na2SO4→NaCl→NaNO3→NaNO3 →
→NH4NO2→N2
б) N2→NH3→NO→HNO3→O2→P2O5→Ba(H2PO4)2→BaHPO4 →
→Ba3(PO4)2
10
а) Si→Mg2Si→SiH4→SiO2→Na2SiO3→K2SiO3→SiO2→SiF4
б) Zn→Na2[Zn(OH)4] →ZnSO4→ZnCl2→Zn(OH)2→ZnCl2→Zn →
→K2ZnO2
11
а) Са→Са(ОН)2→СаСО3→Са(НСОз)2→СаСО3→CaO→Ca(OH)2→
→CaCl2→Ca
б) H2S→S→H2SO4→NaHSO4→Na2SO4→NaNO3→HNO3→
→NH4NO3→N2O
12
а) NaH→Na→H2→CaH2→NH3→NH4Cl→N2→Ca3N2→NH3
б) Fe→FeCl3→Fe(NO3)3→Fe2O3→FeO→FeCl2→FeCl3→
→Fe(NO 3)3→ Fe2O3
13
а) Cl2→KC1O3→KCl→KOH→K2[Zn(ОH)4] →Zn(OH)2→ZnSO4→
→Zn→ZnCl2
б) K→KOH→KC1O3→O2→H2O→H2→H2S→SO2→NaHSO3→
→Na2SO4
14
а) Fe→Fe2(SO4)3→Fe2O3→Fe→Fe(OH)2→Fe→Fe(NO3)3→NO2→
→HNO3→Fe(NO3)3
б) KMnO4→O2→SiO2→Si→SiH4→Na2SiO3→CaSiO3→CaCO3
15
а) MgO→MgCl2→Mg(OH)2→ (MgOH)Cl→MgCl2→Mg→MgSO4
б) KNO3→KNO→KNO3→P2O5→CaHPO4→Ca3(PO4)2→H3PO4→P
16
17
а) Fe→Fe(NO3)3→Fe2O3→FeO→Fe→FeSO4→Fe(OH)2→
→Fe(OH)3→ KFeO2
б) CuCl2→ Cu(OH)2→ (CuOH)Cl→ Cu→ Cu(NO3)2→CuO→CuSO4
а) CaCl2→Ca→CaO→Ca(OH)2→CaCO3→Ca(HCO3)2→СаС12
→Са(ОН)2→Са(ОС1)2
б) Si→SiO2→Na2SiO3→Na2SO4→NaCl→NaOH→Fe(OH)2→
→Fe(OH)3→Fe
18
а) Fe→FeSO4→Fe2(SO4)3→Fe(NO3)3→Fe(NO3)2→Fe(OH)3→
→KFeO2→Fe(OH)3
б) Ca3(PO4)2→P→Ca3P2→PH3→P2O5→NaH2PO4→Na3PO4→
→Ca3(PO4)2
19
а) NH3→NH4HCO3→NaHCO3→CO→K2CO3→CO2→C→SiC
б) Na2SO4→NaCl→NaNO3→O2→ CuO→Cu(OH)2→Cu(NO3)2→Cu
28
20
а) FeS→SO2→SO3→H2SO4→CuSO4→Cu→ Cu(NO3)2→NO2→NO
б) KCl→ С12→FeС12→Fe(OH)2→ FeS→ SO2 →FeS →Fe2O3 →
→Fe(NO3)3
Задание 2.
Таблица 4
№
вари
Ионно-молекулярные реакции обмена
анта
1 Составьте молекулярные и ионно-молекулярные уравнения
реакций взаимодействия в растворах между:
а) NaHCO3 и NaOH; б) К2SiO3 и НСl; в) ВаСl2 и Na2SО4.
2
Составьте молекулярные и ионно-молекулярные уравнения
реакций взаимодействия в растворах между:
а) К2S и НСl; б) FeSO4 и (NH4)2S; в) Сr(ОН)3 и КОН.
3 Составьте по три молекулярных уравнения реакций, которые
выражаются ионно-молекулярными уравнениями:
а) Мg2+ + СО32- = МgCO3; б) Н+ + ОН- = Н2О.
4 Какое
из
веществ:
Аl(ОН)3,
Н2SO4,
Ва(ОН)2
будет
взаимодействовать с гидроксидом калия? Запишите молекулярные
и ионно-молекулярные уравнения этих реакций.
5 Составьте молекулярные и ионно-молекулярные уравнения
реакции взаимодействия в растворах между: а) КНСО3 и Н2SO4;
б) Zn(OH)2 и NaOH; в) СаСl2 и AgNO3.
6 Составьте молекулярные уравнения реакций взаимодействия в
растворах между: а) СuSO4 и Н2S; б) ВаСО3 и НNO3; в) FeCl3 и
КОН.
7 Составьте по три молекулярных уравнения реакций, которые
выражаются ионно-молекулярными уравнениями:
а) Сu2+ + S2- → СuS;
б) SiO32- + 2Н+ → Н2SiO3.
8 Составьте молекулярные и ионно-молекулярные уравнения
реакций взаимодействия в растворах между: а) Sn(OH)2 и НСl;
б) ВеSO4 и КОН; в) NH4Сl и Ва(ОН)2.
9 Какое из веществ: КНСО3, СН3СООН, NiSO4, Na2S
взаимодействует с раствором серной кислоты? Запишите
молекулярные и ионно-молекулярные уравнения этих реакций.
10 Составьте молекулярные и ионно-молекулярные уравнения
реакций взаимодействия в растворах между: а) АgNO3 и К2СrО4;
б) Рb(NO3)2 и КI; в) СdSO4 и Na2S.
11 Составьте молекулярные уравнения реакций, которые выражаются
ионно-молекулярными уравнениями:
а) СаСО3 + 2Н+ = Са2+ + Н2О + СО2;
29
12
13
14
15
16
17
18
19
20
б) Аl(ОН)3 + ОН- = АlО2- + 2Н2О;
в) Рb2+ + 2I- = РbI2.
Составьте молекулярные и ионно-молекулярные уравнения
реакций взаимодействия в растворах между: а) Ве(ОН)2 и NaOH;
б) Сu(ОН)2 и НNO3; в) ZnOHNO3 и НNO3.
Составьте молекулярные и ионно-молекулярные уравнения
реакций взаимодействия в растворах между: а) Na3РО4 и СаСl2;
б) К2СО3 и ВаСl2; в) Zn(OH)2 и КОН.
Составьте молекулярные уравнения реакций, которые выражаются
ионно-молекулярными уравнениями:
а) Fe(ОН)3 + 3Н+ = Fe3+ + 3Н2О;
б) Сd2+ + 2ОН- = Сd(ОН)2;
в) Н+ + NO 2 = НNO2.
Составьте молекулярные и ионно-молекулярные уравнения
реакций взаимодействия в растворах между: а) СdS и НСl;
б) Сr(ОН)3 и NaOH; в) Ва(ОН)2 и СоСl2.
Составьте
молекулярные
уравнения
реакций,
которые
выражаются ионно-молекулярными уравнениями:
а) Zn2+ + H2S = ZnS + 2H+;
б) НСО 3 + Н+ = H2O + CO2;
в) Аg+ + Сl- = AgCl.
Составьте молекулярные и ионно-молекулярные уравнения
реакций взаимодействия в растворах между: а) Н2SO4 и Ва(ОН)2;
б) FeCl3 и NH4OH; в) СН3СООNa и НСl.
Составьте молекулярные и ионно-молекулярные уравнения
реакций взаимодействия в растворах между: а) FeCl3 и КОН;
б) NiSO4 и (NH4)2S; в) МgСО3 и НNO3.
Составьте молекулярные уравнения реакций, которые выражаются
ионно-молекулярными уравнениями:
а) Ве(ОН)2 + 2ОН- = ВеО22- + 2Н2О;
б) СН3СОО- + Н+ = СН3СООН;
в) Ва2+ + SO42- = ВаSO4.
Какое из веществ: NaCl, NiSO4, Be(OH)2, КHCO3 взаимодействует
с раствором гидроксида натрия. Запишите молекулярные и
ионно-молекулярные уравнения этих реакций.
Задание 3.
В предложенных соединениях определите степень окисления указанного
элемента и укажите его роль в окислительно-восстановительных реакциях
(окислитель, восстановитель; окислитель или восстановитель в зависимости от условий) (табл. 5).
30
Таблица 5
№
варианта
1
2
3
4
5
6
7
8
9
10
11
12
13
14
15
16
17
18
19
Элемент
Соединения
S
Fe
N
Cl
P
Mn
I
Cr
N
Te
Mo
As
Sb
S
Au
C
Br
V
Pb
S, H2SO3, H2S, KHSO3, Na2S2O3
FeCl3, FeCl2, NaFeO2, Na2FeO4, K3[Fe(CN)6]
N2O, NH4NO3, HNO2, NO2, NH2OH
HCl, HClO, HClO2, Cl2O7, Ca(ClO4)2
H3PO3, PH3, Na2HPO4, Ca3P2
Mn, HMnO4, MnO2, K2MnO4, MnCl2
KI, HIO3, HOI, KIO4, I2
CrCl2, CrI2, KCrO2, K2Cr2O7, Cr2O3
N2O5, Na3N, NH4F, NaNO2, N2O4
TeO3, H2Te, TeO2, H6TeO6, TeBr2
MoO3, K2MoO4, H2[MoF8], MoO2, Mo
AsH3, H3AsO4, As, As2S5, KAsO2
HSbO3, SbCl3, Sb, Sb2O5, Sb(OH)2Cl
SO3, H2S2O2, SO2, H2SO3, SOCl2
Au, AuCl3, AuCN, HAuCl3
CO, CO2, CH4, H2CO3, Ca(HCO3)2
Br2, KBrO3, KBrO4, HBr, Na2[CdBr4]
VO2, V, VOCl2, V2O5, K3[VF6]
Pb, PbCl2, PbO2, PbCl4, PbOHNO3
20
Se
SeO2, H2SeO3, H2SeO4, Au2(SeO4)3, H2Se
Задание 4.
Таблица 6
№
вариОкислительно - восстановительные реакции
анта
1
Реакции выражаются схемами:
а) P + HNO3 + H2O→ H3PO4 + NO;
б) KMnO4 + Na2SO3 + KOH → K2MnO4 + Na2SO4 + H2O.
Составьте электронные уравнения. Расставьте коэффициенты в
уравнениях реакций. Для каждой реакции укажите, какое вещество
является окислителем, какое – восстановителем; какое вещество
окисляется, какое – восстанавливается.
31
2
3
4.
5
6
7
8
Исходя из степени окисления фосфора в соединениях РН3, Н3РО4,
Н3РО3, определите, какое из них является только окислителем,
только восстановителем и какое может проявлять как
окислительные, так и восстановительные свойства? Почему?
На основании электронных уравнений расставьте коэффициенты в
уравнении реакции, идущей по схеме:
КBr + KBrO3 + H2SO4 → Br2 + К2SO4 + Н2О.
Составьте электронные уравнения и укажите, какой процесс –
окисление или восстановление, происходит при следующих
превращениях: As3- → As5+ ; N3+ → N3- ; S4+ → S0.
На основании электронных уравнений расставьте коэффициенты в
уравнении реакции, идущей по схеме:
Na2SO3 + KMnO4 + H2O → Na2SO4 + MnO2 + KOH.
Исходя из степени окисления хлора в соединениях НСl, НСlO,
НСlO4, определите какое из них является только окислителем,
только восстановителем и какое может проявлять как
окислительные, так и восстановительные свойства. Почему?
На основании электронных уравнений расставьте коэффициенты в
уравнении реакции, идущей по схеме:
РbS + HNO3 → S + Pb(NO3)2 + NO + H2O.
Составьте электронные уравнения и укажите, какой процесс окисление или восстановление – происходит при следующих превращениях: Mn6+ → Mn2+; Cl3+ → Cl-; N3- → N5+.
На основании электронных уравнений расставьте коэффициенты в
уравнении реакции, идущей по схеме:
NaCrO2 + PbO2 + NaOH → Na2CrO4 + Na2PbO2 + H2O.
Реакции выражаются схемами:
а) Н2S + Сl2 + Н2О → Н2SO4 + НСl;
б) Р + НIO3 + Н2O → Н3РО4 + НI.
Составьте электронные уравнения. Расставьте коэффициенты в
уравнениях реакций. Для каждой реакции укажите, какое вещество
является окислителем, какое – восстановителем; какое вещество
окисляется, какое – восстанавливается.
Реакции выражаются схемами:
а) HNO3 + Ca → NH4NO3 + Ca(NO3)2 + H2O;
б) K2S + KMnO4 + H2SO4 → S + K2SO4 + MnSO4 + H2O.
Составьте электронные уравнения. Расставьте коэффициенты в
уравнениях реакций. Для каждой реакции укажите, какое вещество
является окислителем, какое – восстановителем; какое вещество
окисляется, какое – восстанавливается.
Исходя из степени окисления хрома, иода и серы в соединениях
K2Cr2O7, KI и H2SO3, определите, какое из них является только
окислителем, только восстановителем и какое может проявлять как
32
9
10
11
12
13
14
окислительные, так и восстановительные свойства. Почему?
На основании электронных уравнений расставьте коэффициенты в
уравнении реакции, идущей по схеме:
Сu2О + НNO3 → Сu(NO3)2 + NO + H2O.
Реакции выражаются схемами:
а) HNO3 + Zn → N2O + Zn(NO3)2 + H2O;
б) FeSO4 + KClO3 + H2SO4 → Fe2(SO4)3 + KCl + H2O.
Составьте электронные уравнения. Расставьте коэффициенты в
уравнениях реакций. Для каждой реакции укажите, какое вещество
является окислителем, какое – восстановителем; какое вещество
окисляется, какое – восстанавливается.
Реакции выражаются схемами:
а) K2Cr2O7 + HCl → Cl2 + CrCl3 + KCl + H2O;
Б)Au + HNO3 + HCl→ AuCl3 + NO + H2O.
Составьте электронные уравнения. Расставьте коэффициенты в
уравнениях реакций. Для каждой реакции укажите, какое вещество
является окислителем, какое – восстановителем; какое вещество
окисляется, какое – восстанавливается.
Реакции выражаются схемами:
а) H2S + Cl2 + H2O → H2SO4 + HCl;
б) K2Cr2O7 + H2S + H2SO4 → S + Cr2(SO4)3 + K2SO4 + H2O.
Составьте электронные уравнения. Расставьте коэффициенты в
уравнениях реакций. Для каждой реакции укажите, какое вещество
является окислителем, какое – восстановителем; какое вещество
окисляется, какое – восстанавливается.
Реакции выражаются схемами:
а) KClO3 + Na2SO3 → KCl + Na2SO4;
б) KMnO4 + HBr → Br2 + KBr + MnBr2 + Н2О.
Составьте электронные уравнения. Расставьте коэффициенты в
уравнениях реакций. Для каждой реакции укажите, какое вещество
является окислителем, какое – восстановителем; какое вещество
окисляется, какое – восстанавливается.
Реакции выражаются схемами:
а) P + HClO3 + H2O → H3PO4 + HCl;
б) H3AsO3 + KMnO4 + H2SO4 → H3AsO4 + MnSO4 + K2SO4 + H2O.
Составьте электронные уравнения. Расставьте коэффициенты в
уравнениях реакций. Для каждой реакции укажите, какое вещество
является окислителем, какое – восстановителем; какое вещество
окисляется, какое – восстанавливается.
Реакции выражаются схемами:
а) NaCrO2 + Br2 + NaOH → Na2CrO4 + NaBr + H2O;
б) Cd + KMnO4 + H2SO4 → CdSO4 + MnSO4 + K2SO4 + H2O.
Составьте электронные уравнения. Расставьте коэффициенты в
33
15
16
17
18
19
20
уравнениях реакций. Для каждой реакции укажите, какое вещество
является окислителем, какое – восстановителем; какое вещество
окисляется, какое – восстанавливается.
Могут ли происходить окислительно-восстановительные реакции
между веществами: а) NH3 и KMnO4; б) HNO2 и HI; в) HCl и H2Se?
Почему?
На основании электронных уравнений расставьте коэффициенты в
уравнении реакций, идущей по схеме:
KMnO4 + KNO2 + Н2SO4 → MnSO4 + KNO3 + K2SO4 + H2O
Реакции выражаются схемами:
а) НСl + СrO3 → Сl2 + CrCl3 + H2O;
б) FeS + HNO3 → Fe(NO3)2 + S + NO + H2O.
Cоставьте электронные уравнения. Расставьте коэффициенты в
уравнениях реакций. Для каждой реакции укажите, какое вещество
является окислителем, какое – восстановителем; какое вещество
окисляется, какое – восстанавливается.
Реакции выражаются схемами:
а) I2 + Cl2 + Н2О → HIO3 + HCl
б) K2Cr2O7 + H3PO3 + H2SO4→ Cr2(SO4)3 + H3PO4 + K2SO4 + H2O.
Cоставьте электронные уравнения. Расставьте коэффициенты в
уравнениях реакций. Для каждой реакции укажите, какое вещество
является окислителем, какое – восстановителем; какое вещество
окисляется, какое – восстанавливается.
Могут ли происходить окислительно-восстановительные реакции
между веществами: а) РН3 и НBr; б) К2Сr2O7 и H3PO3; в) HNO3 и
H2S? Почему?
На основании электронных уравнений расставьте коэффициенты в
уравнении реакции, идущей по схеме:
AsH3 + HNO3 → H3AsO4 + NO2 + H2O.
Реакции выражаются схемами:
а) H2SO3 + HClO3 → H2S4 + HCl;
б) FeSO4 + K2Cr2O7 + H2SO4 → Fe2(SO4)3 + Cr2(SO4)3+ K2SO4+ H2O.
Составьте электронные уравнения. Расставьте коэффициенты в
уравнениях реакций. Для каждой реакции укажите, какое вещество
является окислителем, какое – восстановителем; какое вещество
окисляется, какое – восстанавливается.
Реакции выражаются схемами:
а) Cr2O3+KClO3 +KOH→ K2CrO4 + KCl + H2O;
б) MnSO4+ PbO2 +HNO3→ HMnO4 + Pb(NO3)2 + PbSO4 + H2O.
Составьте электронные уравнения. Расставьте коэффициенты в
уравнениях реакций. Для каждой реакции укажите, какое вещество
является окислителем, какое – восстановителем; какое вещество
окисляется, какое – восстанавливается.
34
Глава 2. Строение атома. Периодическая система и Периодический
закон Д.И. Менделеева в свете современных представлений о строении
атома. Химическая связь. Комплексные соединения
Строение атома
Атом имеет сложное строение, он состоит из ядра и движущихся вокруг ядра электронов (масса ~ 0, заряд -1); атом электронейтрален, вся
масса атома сосредоточена в ядре.
Ядро состоит из нуклонов: протонов – р+ (масса 1, заряд +1, их
число (Nр+) равно номеру элемента в периодической системе Д.И. Менделеева) и нейтронов – n0 (масса 1, заряд 0, их число равно Nn0 = Ar – Nр+,
где Ar относительная атомная масса элемента). Например, у атома меди
29
+
64 Сu число протонов Nр = 29, число электронов Nē = 29, число нейтронов Nn0 = Ar(Сu) – Nр+ = 64 – 29 = 35.
Изотопы – разновидности атомов одного и того же химического
элемента, которые имеют разные массовые числа. Свойства атомов химического элемента зависят не от их масс, а от зарядов их атомных ядер
(числа р+). Каждый химический элемент встречается в природе в виде смеси изотопов. Например, Ar(К) = 39,102, т.к. в природе встречаются два вида
изотопов калия: 39К с массовым числом 39 и 40К с массовым числом 40.
Изотопы водорода 11Н – протий, 21Н – дейтерий, 31Н – тритий.
Изобары – атомы различных элементов, имеющие одинаковые массовые числа 4018Ar, 4019K, 4020Ca.
Совокупность электронов, окружающих атомное ядро, называется
электронной оболочкой. В электронной оболочке различают слои, на которых расположены электроны с различным запасом энергии, поэтому их
называют энергетическими уровнями. Число уровней в атоме химического
элемента равно соответствующему ему номеру периода в таблице Менделеева (у атома Р, элемента III периода, три уровня, а у атома Ва, элемента
VI периода, шесть уровней). Энергетические уровни, содержащие максимальное число электронов – завершенные.
Часть атомного пространства, в которой вероятность нахождения
данного электрона наибольшая (≈ 90 %), называется атомной орбиталью.
Виды атомных орбиталей: s-орбиталь (имеет форму шара, может разместить не более 2 ē), р-орбиталь (имеет форму объемной восьмерки, на рподуровне может находиться не более 6 ē), d-орбиталь (имеет форму объемной восьмерки, на d-подуровне может находиться не более 10 ē), fорбиталь (на f-подуровне может находиться не более 14 ē). Современная
теория строения электронных оболочек атома базируется на следующих
экспериментальных и теоретических данных:
1. Гипотеза Планка о дискретности, квантовании энергии. Согласно
Планку, энергия излучается и поглощается порциями – квантами : ∆Е=hν,
35
где ∆Е – поглощенная или излученная энергия, h – постоянная Планка,
равная 6,63·10-34 Дж·сек, ν – частота излучения.
2. Опыты Резерфорда по облучению тонкой золотой фольги αчастицами, на основании которых Резерфорд впервые смог оценить размер
атома. Эта величина была равной ~10-8см, при этом оказалось, что практически вся масса атома сосредоточена в области ~10-13см (ядро).
3. Гипотеза де Бройля о волновых свойствах любой микрочастицы:
любой вид движения носит одновременно и волновой и корпускулярный
характер. Математическое выражение этой гипотезы имеет вид:

h
,
mv
где λ – длина волны собственных волновых колебаний частицы, h –
постоянная Планка, v – скорость и m –масса частицы.
4. Волновое уравнение Шредингера. В нем Шредингер исходил из
предположения, что устойчивому состоянию электрона в атоме соответствует стоячая волна. Состояние электрона в атоме описывает волновая
функция (ψ-функция), являющаяся решением уравнения Шредингера. Эта
функция (точнее квадрат ее модуля |ψ|2) дает вероятность нахождения
электрона на данном расстоянии от центра атома. Поскольку ψ-функция
связана с вероятностью, то вводят понятие орбиталь. Под орбиталью понимают часть атомного пространства, в которой вероятность нахождения
данного электрона наибольшая (равна ≈ 90 %), т.е.  dV  0,9 . Орбиталь
имеет какую-то ограниченную поверхность, поэтому говорят о типе (виде)
орбитали, ее форме и т.п.
Непосредственно из уравнения Шредингера вытекают три числа, получившие названия квантовых чисел, описывающих состояние электрона в
атоме (n, l, m). Однако скоро выяснилось, что их недостаточно для полного
описания состояния электрона в атоме. Это связано с тем, что электрон
имеет собственный момент количества движения, так называемый спин.
Для учета этого факта в волновую функцию было введено четвертое, спиновое квантовое число (ms), модифицировав волновую функцию, которая в
настоящее время описывается четырьмя квантовыми числами, т.е. ψ=f (n, l,
m, ms).
Квантовые числа. Набор квантовых чисел определяет энергетическое состояние электрона в атоме. Когда говорят о смысле квантовых чисел, выделяют физический (энергетический) смысл и геометрический
(форма и взаимное расположение орбиталей в пространстве атома).
1. Первое квантовое число называется главное квантовое число и
обозначается n. Принимает целочисленные значения от l до ∞, т.е.[l…∞].
С главным квантовым числом связан основной запас энергии электрона в
атоме. С увеличением значения n энергия электрона в атоме увеличивается. Главное квантовое число определяет энергетический уровень, на кото36
ром находится электрон. В этом случае уровни принято обозначать заглавными латинскими буквами, начиная с К, т.е.:
Значения n: 1 2 3 4 5 6 7
Уровень : K L M N O P Q.
Количество электронов на данном уровне (емкость уровня) можно определить по формуле N = 2n2, где n – значение главного квантового числа.
2. Второе квантовое число называется орбитальное квантовое число и обозначается l. Оно зависит от главного квантового числа n и при
данном значении его принимает целочисленные значения от 0 до (n- 1),
т.е. [0, n- 1].Орбитальное квантовое число связано с моментом количества
движения электрона в атоме. С ростом l энергия электрона в атоме растет,
n и l определяют весь запас энергии электрона в атоме. Значения орбитального квантового числа определяют подуровень данного уровня, тип и
форму орбитали. При этом для орбиталей используются условные обозначения:
Значение l : 0 1 2 3 4 5
Тип орбитали : s p d f g h
3. Третье квантовое число называется магнитное квантовое число и
обозначается m или ml. Оно зависит от орбитального и при данном значении его принимает целочисленные значения от –l до +l, через единицу,
включая 0, всего 2l+1 значений. Магнитное квантовое число связано с
энергией магнитного взаимодействия электрона с внешним магнитным полем. Число значений магнитного квантового числа (2l +1) дает число орбиталей данного типа. Сами значения m определяют пространственное
расположение орбиталей данного типа относительно друг друга. В отсутствии внешнего магнитного поля все орбитали данного типа имеют одинаковую энергию.
Подуровень: s
p
d
f
Возможные
значения m:
0 -1; 0; +1 -2; -1; 0; +1; +2 -3;-2;-1, 0; +1; +2; +3
4. Четвертое квантовое число называется спиновое квантовое число
и обозначается ms или s. Оно определяется внутренними свойствами электрона и может принимать два условных значения +1/2 и – 1/2. Спиновое
квантовое число определяет собственный момент количества движения
электрона. Считается, что когда ms= +1/2 (графическое изображение – ↑),
то электрон вращается относительно оси, совпадающей с направлением
движения по часовой стрелке, когда – 1/2 (графическое изображение – ↓),
то – против часовой стрелки.
В атоме все электроны физически неразличимы, поэтому мы можем
определить только набор квантовых чисел последнего по энергии состояния, которое занимает какой-то электрон.
37
Заполнение электронных состояний в атоме. Набор квантовых чисел определяет энергетическое состояние электрона в атоме. В атоме число
электронов равно порядковому номеру элемента или, точнее, заряду ядра
атома. При заполнении электронных состояний атома в первую очередь
заполняется то, которое имеет меньшую энергию.
При заполнении электронных состояний атома последовательно реализуются следующие принципы:
1. Минимум энергии: электроны занимают наиболее энергетически
выгодную орбиталь, на которой имеется хотя бы одно свободное место.
2. Принцип запрета Паули: в атоме не может быть двух и более
электронов, обладающих одинаковым набором всех четырех квантовых
чисел (следовательно, на одной орбитали не может располагаться более
двух электронов, они должны различаться спиновым квантовым числом).
3. Правило Гунда (Хунда): электроны располагаются по орбиталям
таким образом, чтобы их суммарный спин был максимальным (ms), т. е. на
подуровне должно быть максимальное число неспаренных электронов.
4. Правила Клечковского:
1-ое правило: при увеличении заряда ядра атома последовательное
заполнение электронных орбиталей происходит от орбиталей с меньшим
значением суммы главного и орбитального квантовых чисел (n+l) к орбиталям с большим значением этой суммы.
2-ое правило: при одинаковых значениях суммы n+l заполнение орбиталей происходит в порядке возрастания значений главного квантового
числа n.
Подуровень:
Сумма n+l :
4s
4+0=4
3d
3+2=5
5p
5+1=6
4d
4+2=6
6p
6+1=7
4f
4+3=7
Последовательность заполнения электронных состояний в атоме:
1s 2s22p6 3s23p6 4s23d104p6 5s24d105p6 6s25d14f145d96p6 7s26d15f146d97p6
Отметим, что в 6-ом и 7-ом периодах сначала один электрон занимает d-состояние, затем происходит полное заполнение f-подуровня, и
лишь после этого протекает заполнение d-подуровня до 10 электронов.
Следует отметить, что после полного заполнения d- и f-подуровней они
«проваливаются» по энергии, и соотношение по энергии имеет вид:
Ens>E(n-1)d>E(n-2)f.
2
Периодический закон и периодическая система
Д. И. Менделеева в свете современных представлений о
строении атома
Периодический закон – величайшее достижение химической науки,
основа современной химии: свойства простых тел, а также формы и
свойства простых и сложных соединений элементов находятся в перио38
дической зависимости от относительных атомных масс элементов. Периодический закон открыт Д.И. Менделеевым в 1869 году. Графическим
изображением периодического закона (ПЗ) является периодическая система химических элементов (ПСХЭ).
Состав и структура ПСХЭ. Каждый элемент, изображенный соответствующим символом, имеет порядковый номер и занимает определенное
место в ПСХЭ. Порядковый номер равен заряду ядра атома и числу электронов в оболочке атома.
Горизонтальный ряд элементов, расположенных по возрастанию порядковых номеров, в котором закономерно изменяются свойства при переходе от металлов к неметаллам, называется периодом. В ПСХЭ семь периодов, из них 1-й, 2-й, 3-й – малые, 4-й, 5-й, 6-й, 7-й – большие. Номер периода указывает на число уровней в атоме данного элемента.
Вертикальный ряд, объединяющий элементы с одинаковой валентностью в высших оксидах, называется группой. Всего восемь групп. В
пределах одной группы не все элементы явно сходны по своим свойствам.
Поэтому каждая группа делится на две подгруппы – главную и побочную.
Главные подгруппы включают в себя элементы малых периодов и сходные
с ними по свойствам элементы больших периодов. Побочные подгруппы
включают в себя только элементы больших периодов.
Многие свойства элементов зависят от электронной конфигурации, и
в их изменении по мере увеличения порядкового номера элемента наблюдается периодичность. Свойства химических элементов периодически (т. е.
через определенные промежутки – периоды) повторяются потому, что периодически повторяется одинаковое строение внешних энергетических
уровней их атомов.
К таким свойствам атомов элементов относятся: атомный радиус,
энергия ионизации, сродство к электрону, электроотрицательность, степень окисления (валентность), оптические и магнитные свойства.
Все элементы делятся на металлы и неметаллы. Металлы – это элементы, атомы которых отдают свои внешние электроны. Неметаллы –
это элементы, атомы которых стремятся принять на внешний энергетический уровень электроны, недостающие до восьми (в случае водорода – до
двух). Чем легче атом металла отдает свои внешние электроны, тем более
сильно будут выражены его металлические свойства. И наоборот, чем легче атом неметалла принимает недостающие электроны на внешний слой,
тем более сильно будут выражены его неметаллические свойства.
Изменение металлических и неметаллических свойств атомов
элементов в периодах и группах: в пределах одного периода с ростом порядкового номера металлические свойства элементов ослабевают, а неметаллические усиливаются, т.к. растет число электронов на внешнем уровне
атомов (оно равно номеру группы); число энергетических уровней в пределах периода не изменяется (оно равно номеру периода); радиус атомов
39
уменьшается, «сжимается». В пределах одной группы (главной подгруппы)
с ростом порядкового номера металлические свойства элементов усиливаются, а неметаллические ослабевают, т.к. число электронов на внешнем
уровне атомов одинаково; число энергетических уровней в атомах растет,
радиус атомов увеличивается.
Энергия ионизации атомов (I) равна энергии, которую необходимо
затратить, чтобы перенести один электрон из атома в бесконечность (при
этом образуется положительный ион): А – е → А+ . Измеряется энергия
ионизации в кДж/моль, эВ/моль. Она служит количественной характеристикой восстановительной активности элементов. В периодах слева направо энергия ионизации возрастает, восстановительная активность уменьшается. В группах s-, p- и в третьей группе d-элементов сверху вниз величины
I и восстановительная активность элементов увеличиваются. В группах dэлементов, кроме третьей, энергия ионизации уменьшается, как правило, в
обратном направлении, т.е. снизу вверх, и нарастают восстановительные
свойства элементов.
Сродство к электрону (Е) – энергия, которая выделяется при присоединении к атому одного электрона с образованием отрицательного иона:
А + е → А- (Cl + e → Cl-). Измеряется энергия сродства к электрону в
кДж/моль, эВ/моль. Она служит количественной характеристикой окислительной активности элементов. Сродство к электрону возрастает с уменьшением радиуса, т.е. в периодах – слева направо и в группах неметаллов –
снизу вверх. В этих же направлениях увеличивается окислительная активность элементов.
Электроотрицательность (ЭО) определяется как способность атомов
притягивать (принимать) электроны. Общее стремление атома к присоединению электрона определяется арифметической полусуммой значений
энергии ионизации и сродства к электрону. Оценивать электроотрицательность имеет смысл только для элементов, имеющих окислительную
активность. В периодах ЭО возрастает слева направо; в группе ЭО больше
у элемента, расположенного выше. Наиболее электроотрицательным из
всех элементов является фтор.
Химическая связь
Под химической связью понимают такое взаимодействие атомов, которое связывает их в молекулы, ионы, радикалы, кристаллы. Химическая
связь осуществляется s- и p-электронами внешнего и d-электронами предпоследнего слоя. Она характеризуется следующими параметрами:
1) длиной связи – межъядерным расстоянием между двумя химически
связанными атомами;
2) валентным углом – углом между воображаемыми линиями, проходящими через центры химически связанных атомов;
40
3) энергией связи – количеством энергии, затрачиваемой на ее разрыв в
газообразном состоянии;
4) кратностью связи – числом электронных пар, посредством которых
осуществляется химическая связь между атомами.
Для возникновения химической связи необходимо перекрывание частей электронных оболочек, в противном случае, химическая связь не образуется. В зависимости от симметрии электронных облаков, в результате
перекрывания которых образуется химическая связь, суммарное электронное облако будет иметь различную симметрию. Различают три вида связи:
σ – связь, которая образуется при перекрывании облаков вдоль линии, соединяющей центры атомов, при этом максимальная электронная
плотность достигается в межъядерном пространстве и имеет цилиндрическую симметрию относительно линии, соединяющей центры атомов;
π – связь, возникающую при перекрывании электронных облаков над
и под линией, соединяющей центры атомов;
δ – связь, которую образуют только d-электроны за счет перекрывания всех четырех лепестков своих электронных облаков, расположенных в параллельных плоскостях.
Существует и другой подход к классификации химической связи,
основанный на характере распределения электронной плотности между
атомами в молекуле, т. е. химическая связь рассматривается с точки зрения
принадлежности электронной пары тому или иному атому. Возможны три
случая:
1) электронная пара связывает в молекуле два одинаковых атома (в
этом случае она в равной мере принадлежит им обоим), такая связь называется ковалентной неполярной;
2) электронная пара связывает два различных атома, смещаясь в сторону более электроотрицательного атома, такая связь носит название ковалентной полярной связи;
3) полная передача электронной пары во владение одного из атомов;
это происходит при взаимодействии двух атомов, резко отличающихся по
электроотрицательности – способности удерживать электронную пару в
своем электрическом поле (ΔЭО > 1,7); атом, отдавший электроны, становится положительно заряженным ионом, а атом, принявший их, – отрицательным ионом; такая связь называется ионной.
Характер связи во многом определяют физико-химические свойства
веществ. Вещества, молекулы которых характеризуются ковалентной неполярной связью, могут в твердом состоянии образовывать молекулярные
и атомные кристаллические решетки.
В молекулярных решетках наблюдается очень слабое межмолекулярное взаимодействие. В узлах кристаллической решетки молекулы
удерживаются за счет образования в них мгновенных и наведенных диполей. Силы, связывающие молекулы за счет образования мгновенных и
41
наведенных диполей, часто называют ван-дер-ваальсовыми. Ван-дерваальсовы силы являются очень слабыми, вследствие чего кристаллическую решетку легко разрушить незначительным нагреванием. Все вещества, имеющие молекулярные кристаллические решетки, обладают низкими температурами плавления и кипения. Ван-дер-ваальсова сила растет с
увеличением количества электронов в молекуле.
Простые вещества, обладающие атомной кристаллической решеткой, наоборот, характеризуются очень высокими температурами плавления
и кипения. В этом случае бесконечное число атомов соединено неполярными ковалентными связями в гигантскую молекулу. Энергия разрыва ковалентной неполярной связи велика. Поэтому, чтобы разрушить такую
кристаллическую решетку, требуются большие энергетические затраты. В
то же время, если вещество характеризуется одним межатомным расстоянием в кристаллической решетке, оно будет обладать и очень высокой
твердостью. Примером может служить алмаз.
Температуры плавления и кипения веществ, в решетках которых расположены молекулы с ковалентной полярной связью, также низки, но
выше, чем у веществ с неполярными молекулами. В большинстве своем
это газы при комнатной температуре (например, хлористый водород, сероводород). Прямой зависимости между величиной дипольного момента и
температурой кипения не наблюдается. Она определяется молекулярной
массой соединения, за исключением аммиака, воды и фтористого водорода. Эти соединения в ряду им подобных обладают наивысшими температурами плавления и кипения, резкое их увеличение объясняется образованием между молекулами водородных связей. Атомы азота, кислорода и
фтора являются самыми электроотрицательными атомами. Они сильно
смещают электронную плотность в молекуле на себя, в результате этого у
водорода образуется практически свободная орбиталь, а, например, у атома фтора имеется свободная электронная пара. Появляется возможность
образовывать между молекулами дополнительные связи по донорноакцепторному механизму, что приводит к существованию не только в
жидкой, но и газовой фазе молекул состава (HF)n и (H2O)n. Наличие таких
молекул приводит к повышению температур кипения и плавления по сравнению с теми же величинами у аналогов.
Наибольшими температурами плавления и кипения обладают вещества с ионными кристаллическими решетками. Это объясняется сильным электростатическим взаимодействием положительных и отрицательных ионов. Силы же отталкивания одноименных ионов значительно меньше, т. к. они расположены на больших расстояниях друг от друга. Вследствие этого все вещества, имеющие ионные кристаллические решетки, обладают высокими значениями ее энергий образования. Ионная связь осуществляется в галогенидах, оксидах и в солях типа нитратов, сульфатов и
т. п. Для разрушения ионных кристаллов требуется значительная тепловая
42
энергия, что и определяет высокие температуры плавления и кипения, которые, в свою очередь, будут зависеть от заряда ионов, их радиусов и электронной подкладки: чем выше заряд и меньше радиус иона, тем выше температура плавления. Другим свойством ионных веществ является способность образовывать расплавы, хорошо проводящие электрический ток.
Механизмы образования химической связи
(метод валентных связей)
1. Обменный механизм заключается в спаривании двух электронов,
принадлежащих разным атомам и имеющим противоположные спины. Его
можно выразить следующей схемой:
2. Донорно-акцепторный механизм – образование химической связи
за счет предоставления одним атомом в общее пользование пары электронов, а другим – свободной орбитали. Атом, предоставляющий электронную пару, называется донором, а атом, имеющий свободную орбиталь, –
акцептором. Схема образования связи:
Для объяснения пространственного расположения атомов в молекуле
используют теорию гибридизации Л. Полинга.
Гибридизацией называют процесс взаимодействия электронных орбиталей, их слияния, приводящий к выравниванию по форме и энергии. В
зависимости от того, какие орбитали участвуют в образовании гибридных
орбиталей, рассматривают различные типы гибридизации и пространственные конфигурации образовавшихся гибридных орбиталей (рис. 2).
Рис. 2. Пространственное расположение электронных
облаков для различных типов гибридизации.
sp-гибридизация: взаимодействуют одна s- и одна p-орбитали, в результате образуются две орбитали нового типа, которые называются spгибридными; эти орбитали имеют форму неправильной восмерки, оси их
лежат на одной прямой, образуя угол 1800, молекула имеет линейное строение;
sp2-гибридизация: взаимодействуют одна s- и две p-орбитали, в результате чего образуются три орбитали нового типа, которые называются
43
sp2-гибридными; оси этих орбиталей лежат в одной плоскости, углы между
ними равны 1200, молекула имеет форму правильного треугольника;
sp3-гибридизация: взаимодействуют одна s- и три p- орбитали, в результате гибридизации образуются четыре новые sр3-гибридные орбитали,
оси которых направлены от центра к вершинам тетраэдра, углы между
осями равны 109028′, молекула имеет тетраэдрическое строение.
В молекуле аммиака (NH3) атом азота тоже находится в sp3-гибридном
состоянии, три пары электронов являются общими с атомами водорода и
участвуют в образовании связей, а одна электронная пара (неподеленная
электронная пара) принадлежит только атому азота. Для предсказания
геометрии молекулы аммиака атом азота следует поместить в центр тетраэдра, атомы водорода – в три вершины, а неподеленную электронную пару
– в четвертую вершину, таким образом, молекула аммиака имеет пирамидальную форму.
Аналогично можно рассмотреть строение молекулы воды. В ней две
неподеленные электронные пары кислорода занимают две вершины тетраэдра, а две другие заняты атомами водорода, что приводит к угловой форме молекулы воды.
Неподеленная электронная пара образует более размытое «электронное облако», чем связывающая (обобществленная, общая), поэтому она занимает больший объем, приводя к уменьшению валентных углов по сравнению с тетраэдрическими.
Валентные электроны располагаются не только на s- и p-орбиталях, но
и на d, последние также принимают участие в образовании гибридных орбиталей. Осуществляются два случая гибридизации с участием dорбиталей: sp3d2 и sp3d. В первом случае молекула реализует октаэдрическое строение, а во втором образуется тригональная бипирамида (табл.7).
Таблица 7
Геометрические конфигурации молекул
Число
Коордиn m Тип молеку- Форма моле- Примеры
электрон- нация
лы
кулы
ных пар
2
Линейная 2 0 АХ2
Линейная
BeH2,
BeCl2
4
Тетраэдр 4 0 АХ4
Тетраэдр
СН4, SiCl4
3 1 АХ3Е
Тригональная NH3, PH3
бипирамида
2 2 АХ2Е2
Угловая
Н2О, SCl2
5
Триго5 0 АХ5
Тригональная PCl5,AsF5
нальная
бипирамида
бипира4 1 АХ4Е
Дисфеноид
SF4
мида
3 2 АХ3Е2
Т-образная
ClF3
2 3 АХ2Е2
Линейная
XeF2
44
6
Октаэдр
6
5
0
1
АХ6
АХ5Е
4
2
АХ4Е2
Октаэдр
Квадратная
бипирамида
Плоский
квадрат
SF6
BrF5
XrF4
А – многовалентный атом; X – атомы, связанные с атомом А; n – число атомов X;
E – неподеленная пара электронов; m – количество неподеленных электронных пар.
Металлическая связь. В отличие от ионных и ковалентных соединений металлы обладают высокой электропроводностью и теплопроводностью. Высокая электропроводность металлов указывает на то, что электроны свободно могут передвигаться во всем его объеме, т.е. металл можно
рассматривать как кристалл, в узлах решетки которого расположены ионы,
связанные электронами, находящимися в общем пользовании, т. е. в металлах имеет место сильно нелокализованная химическая связь. Совокупность электронов, обеспечивающих эту связь, называют электронным газом.
Более общий подход к представлению об ионных, ковалентных и металлических кристаллах можно получить, применяя представления метода
молекулярных орбиталей.
Комплексные соединения
При рассмотрении видов химической связи отмечалось, что силы
притяжения возникают не только между атомами, но и между молекулами
и ионами. Такое взаимодействие может приводить к образованию новых
более сложных комплексных (или координационных) соединений.
Комплексными называют соединения, имеющие в узлах кристаллической решетки агрегаты атомов (комплексы), способные к самостоятельному существованию в растворе и обладающие свойствами, отличными от свойств составляющих их частиц (атомов, ионов или молекул).
В молекуле комплексного соединения (например, K4[Fe(CN)6]) различают следующие структурные элементы: ион-комплексообразователь
(для данного комплекса Fe), координированные вокруг него присоединенные частицы – лиганды или адденды (CN-), составляющие вместе с комплексообразователем внутреннюю координационную сферу ([Fe(CN)6]4-),
и остальные частицы, входящие во внешнюю координационную сферу
(K+). При растворении комплексных соединений лиганды остаются в
прочной связи с ионом-комплексообразователем, образуя почти не диссоциирующий комплексный ион. Число лигандов называется координационным числом (в случае K4[Fe(CN)6] координационное число равно 6).
Координационное число определяется природой центрального атома и лигандов, а также соответствует наиболее симметричной геометрической
45
конфигурации: 2 (линейная), 4 (тетраэдрическая или квадратная) и 6 (октаэдрическая конфигурация).
Характерными комплексообразователями являются катионы:
2+
Fe , Fe3+, Co3+, Co2+, Cu2+, Ag+, Cr3+, Ni2+. Способность к образованию комплексных соединений связана с электронным строением атомов. Особенно
легко образуют комплексные ионы элементы d-семейства, например: Ag+,
Au+, Cu2+, Hg2+, Zn2+, Fe2+, Cd2+, Fe3+, Co3+, Ni2+, Pt2+, Pt4+и др. Комплексообразователями могут быть А13+ и некоторые неметаллы, например, Si и В.
Лигандами могут служить как заряженные ионы: F-, ОН-, NO3-, NO2-,
Cl-, Вг-, I-, CO32-, CrO42-, S2O32-, CN-, PO43- и др., так и электронейтральные
полярные молекулы: NH3, Н2О, РН3, СО и др. Если все лиганды у комплексообразователя одинаковы, то комплексное соединение однородное,
например [Pt(NH3)4]Cl2; если лиганды разные, то соединение неоднородное, например [Pt(NH3)3Cl]Cl. Между комплексообразователем и лигандами обычно устанавливаются координационные (донорно-акцепторные)
связи. Они образуются в результате перекрывания заполненных электронами орбиталей лигандов вакантными орбиталями центрального атома. В
комплексных соединениях донором является комплексообразователь, акцептором – лиганд.
Количество химических связей между комплексообразователем и
лигандами определяет координационное число комплексообразователя.
Характерные координационные числа: Cu+, Ag+,Au+= 2; Cu2+,Hg2+,Pb2+,Pt2+,
Pd2+ =4; Ni2+,Ni3+,Co3+,А13+ = 4 или 6; Fe2+, Fe3+, Pt4+, Pd4+, Ti4+, Pb4+, Si4+ =6.
Заряд комплексообразователя равен алгебраической сумме зарядов
составляющих его ионов, например: [Fex(CN)6]4-, x + 6(-1) = 4-; x = 2.
Входящие в состав комплексного иона нейтральные молекулы не
оказывают влияния на заряд. Если вся внутренняя сфера заполнена только
нейтральными молекулами, то заряд иона равен заряду комплексообразователя. Так, у иона [Cux(NH3)4]2+ заряд меди х = 2+. Заряд комплексного
иона равен зарядам ионов, находящихся во внешней сфере. В K4[Fe(CN)6]
заряд [Fe(CN)6] равен -4, так как во внешней сфере находятся 4 катиона К+,
а молекула в целом электронейтральна.
Лиганды во внутренней сфере могут замещать друг друга при сохранении одного и того же координационного числа.
Классификация и номенклатура комплексных соединений. С
точки зрения заряда комплексной частицы все комплексные соединения
можно разделить на катионные, анионные и нейтральные.
Катионные комплексы образуют катионы металлов, координирующие вокруг себя нейтральные или анионные лиганды, причем суммарный
заряд лигандов меньше по абсолютной величине, чем степень окисления
комплексообразователя, например [Co(NH3)6]Cl3. Катионные комплексные
46
соединения помимо гидроксокомплексов и солей, могут быть кислотами,
например H[SbF6] – гексафторсурьмяная кислота.
В анионных комплексах, напротив, лигандов-анионов такое число,
что суммарный заряд комплексного аниона отрицателен, например
[Ni(CO)4]. В анионных комплексах в качестве лигандов выступают гидроксид-анионы – это гидроксокомплексы (например Na2[Zn(OH)4] – тетрагидроксоцинкат калия), или анионы кислотных остатков – это ацидокомплексы(например K3[Fe(CN)6] – гексацианоферрат (III) калия).
Нейтральные комплексы могут быть нескольких видов: комплекс
нейтрального атома металла с нейтральными лигандами (например
Ni(CO)4 – тетракарбонил никеля, [Сr(С6Н6)2] – дибензолхром).
В
нейтральных комплексах другого вида заряды комплексообразователя и
лигандов уравновешивают друг друга (например, [Pt(NH3)2Cl2] – хлорид
гексаамминплатины (IV), [Co(NH3)3(NO2)3] – тринитротриамминкобальт).
Классифицировать комплексные соединения можно по природе лиганда. Среди соединений с нейтральными лигандами различают аквакомплексы, аммиакаты, карбонилы металлов. Комплексные соединения, содержащие в качестве лигандов молекулы воды, называют аквакомплексами. При кристаллизации вещества из раствора катион захватывает часть
молекул воды, которые попадают в кристаллическую решетку соли. Такие
вещества называются кристаллогидратами,
например А1С13·6Н2О.
Большинство кристаллогидратов представляет собой аквакомплексы, поэтому их точнее изображать в виде комплексной соли ([А1(Н2О)6]С13 –
хлорид гексаакваалюминия). Комплексные соединения с молекулами аммиака в качестве лиганда называют аммиакатами, например
[Pt(NH3)6]C14 – хлорид гексаамминплатины (IV). Карбонилами металлов
называют комплексные соединения, в которых лигандами служат молекулы оксида углерода (II), например, [Fe(CO)5] – пентакарбонил железа,
[Ni(CO)4] – тетракарбонил никеля.
Известны комплексные соединения с двумя комплексными ионами в
молекуле, для которых существует явление координационной изомерии,
которая связана с разным распределением лигандов между комплексообразователями, например: [Ni(NH3)6][Co(NO2)6] – гексанитрокобальтат (III)
гексаамминникеля (III).
При составлении названия комплексного соединения применяются
следующие правила:
1) если соединение является комплексной солью, то первым называется
анион в именительном падеже, а затем катион в родительном падеже;
2) при названии комплексного иона сначала указываются лиганды, затем
комплексообразователь;
3) молекулярные лиганды соответствуют названиям молекул (кроме воды
и аммиака, для их обозначения применяются термины «аква» и «амин»);
47
4) к анионным лигандам добавляют окончание – о, например: F- – фторо,
С1- – хлоро, О2- – оксо, CNS- – родано, NO3- – нитрато, CN- – циано, SO42- –
сульфато, S2O32- – тиосульфато, СО32- – карбонато, РО43- – фосфато, ОН- –
гидроксо;
5) для обозначения количества лигандов используются греческие числительные: 2 – ди-, 3 – три-, 4 – тетра-, 5 – пента-, 6 – гекса-;
6) если комплексный ион – катион, то для названия комплексообразователя используют русское наименование элемента, если анион – латинское;
7) после названия комплексообразователя римской цифрой в круглых
скобках указывают его степень окисления;
8) у нейтральных комплексов название центрального атома дается в именительном падеже, а его степень окисления не указывается.
Свойства комплексных соединений. Химические реакции с участием комплексных соединений разделяют на два типа:
1) внешнесферные – при их протекании комплексная частица остается неизменной (реакции обмена);
2) внутрисферные – при их протекании происходят изменения в степени окисления центрального атома, в строении лигандов или изменения в
координационной сфере (уменьшение или увеличение координационного
числа).
Одним из важнейших свойств комплексных соединений является их
диссоциация в водных растворах. Большинство растворимых в воде ионных комплексов – сильные электролиты, они диссоциируют на внешнюю
и внутреннюю сферы: K4[Fe(CN)6] ↔ 4K+ + [Fe(CN)6]4 -.
Комплексные ионы достаточно устойчивы, они являются слабыми
электролитами, ступенчато отщепляя в водный раствор лиганды:
[Fe(CN)6]4 - ↔ [Fe(CN)5]3- + CN- (число ступеней равно числу лигандов).
Если суммарный заряд частицы комплексного соединения равен нулю, то имеем молекулу неэлектролита, например [Pt(NH3)2C14].
При обменных реакциях комплексные ионы переходят из одних соединений в другие, не изменяя своего состава. Электролитическая диссоциация комплексных ионов подчиняется закону действующих масс и количественно характеризуется константой диссоциации, которая носит
название константы нестойкости Кн . Чем меньше константа нестойкости комплекса, тем в меньшей степени он распадается на ионы, тем устойчивее это соединение. У соединений, характеризуемых высокой Кн, комплексные ионы неустойчивы, т. е. их практически нет в растворе, такие соединения являются двойными солями. Отличие между типичными представителями комплексных и двойных солей заключается в том, что последние диссоци-ируют с образованием всех ионов, которые входят в состав этой соли, например: KA1(SO4)2 ↔ К+ + А13+ + 2SO42- (двойная
соль);
48
К [Fe(CN)6] ↔ 4К+ + [Fe(CN)6]4- (комплексная соль).
Литература
1. Глинка Н. Л. Общая химия. – М.: Химия, 2006. – 720 с.
2. Глинка Н.Л. Задачи и упражнения по общей химии. – Л. Химия,
1985. – 264 с.
3. Карапетьянц М.Х., Дракин С.И. Общая и неорганическая химия. –
М.: Химия, 1993. – 558 с.
4. Коровин В.Н. Общая химия. – М.: Высшая школа, 2000. – 557 с.
5. Коровин Н.В., Масленникова Г.Н. и др. Курс общей химии. – М.:
Высшая школа, 1990. – 445 с.
6. Суворов А.В., Никольский А.Б. Общая химия. – СПб: Химиздат,
2001. – 512 с.
Примеры решения типовых задач
Задача 1. Распределите электроны по уровням, напишите электронную формулу атома Ca.
Решение: заряд ядра атома определяем по порядковому номеру элемента; число уровней равно номеру периода, в котором находится элемент;
максимальное число валентных электронов определяется номером группы,
т.к. элемент расположен в главной подгруппе. На первых двух уровнях
максимальное число электронов – 2 и 8 соответственно (определяем по
формуле 2n2, где n – номер периода). Общее число электронов предпоследнего энергетического уровня просчитываем.
Итак, электроны в атоме кальция распределяются по уровням следующим образом: +20 )2 )8 )8 )2.
Электронная формула: 1s22s22p63s23p64s2 (валентные электроны).
Задача 2. Напишите электронную формулу элемента № 82.
Решение: № элемента 82, следовательно, у него 82 электрона. Воспользуемся принципами и схемой заполнения электронных состояний в
атоме. Электронная формула атома Pb имеет вид:
1s22s22p63s23p64s23d104p65s24d105p66s24f145d106p2.
Задача 3. Напишите электронную структуру элемента с номером 24,
укажите его семейство и приведите аналоги.
Решение: воспользуемся принципами и схемой заполнения электронных состояний в атоме. Электронная формула атома Cr имеет вид:
1s22s22p63s23p64s13d5. Поскольку последний электрон при заполнении перешел в d-состояние, то это d-элемент. Электронными аналогами его будут
все элементы, имеющими конфигурацию ns1(n-1)d5. В случае хрома его
электронным аналогом является молибден.
Задача 3. Распределите электроны по уровням, напишите электронную формулу атома Fe в основном и возбужденном состоянии, укажите
49
квантовые состояния его валентных электронов и распределите их по
квантовым ячейкам.
Решение: заряд ядра атома определяем по порядковому номеру элемента (+ 26); число уровней – по номеру периода (4). Так как железо расположено в побочной подгруппе, то валентные электроны его распределяются между s – подуровнем внешнего энергетического уровня и d – подуровнем предвнешнего уровня, общее число электронов равно 8 (номер
группы). Так, если в 4s состоянии находятся 2 электрона, то в 3dсостоянии – 6 (номер группы минус 2). На первых двух уровнях максимальное число электронов – 2 и 8 соответственно (определяется удвоенным квадратом главного квантового числа – 2n2). Общее число электронов
предпоследнего энергетического уровня атома железа просчитываем, оно
равно 14.
Итак, в невозбужденном атоме железа электроны распределяются
следующим образом: +26 )2 )8 )14 )2.
Электронная формула атома железа в невозбужденном состоянии
имеет вид: 1s22s22p63s23p63d64s2 ( валентные электроны).
Порядок заполнения квантовых ячеек подуровня электронами определяется правилом Гунда по принципу наибольшего суммарного спина:
3d
4s
В возбужденном атоме железа один s – электрон переходит на 4р –
подуровень. В возбужденном состоянии электроны распределяются по
квантовым ячейкам следующим образом:
Fе *: 1s22s22p63s23p63d64s14p1 или ... 3d64s14p1
3d
4s
4p1
Задача 4. Определите квантовые числа последнего состояния (электрона) в атоме железа.
Решение. Электронная конфигурация железа: 1s22s22p63s23p64s23d6.
3d
4s
ml= -2; -1; 0; +1; +2
Следовательно, последний электрон приходит на 3d-орбиталь, ему соответствует следующий набор квантовых чисел: n=3, l=2, ml= -2, ms = -1/2.
Задача 2. Напишите электронные формулы ионов Mn2+, S2-, укажите число неспаренных электронов.
Решение:
50
1. Распределение электронов по энергетическим уровням у атома Mn
(d-элемент): 1s22s22p63s23p63d54s2 (валентные электроны).
3d
4s
При образовании положительных ионов у d- элементов первыми всегда
удаляются s-электроны внешнего слоя, т.е. для иона Mn+2 справедлива
электронная формула: 1s22s22p63s23p63d54s0 (валентные электроны).
3d
4s
Число неспаренных электронов равно 5.
2. При образовании отрицательных ионов происходит дальнейшее
заполнение энергетических подуровней согласно правилу Клечковского,
то есть для атома серы S: 1s22s22p63s23p4 (валентные электроны).
3s
3p
Для иона S2– произойдет «достройка» 3р-подуровня: 1s22s22p63s23p6.
3s
3p
Число неспаренных электронов равно 0.
Задача 4. Укажите положение элемента в периодической системе
(период, группу, подгруппу), если последний электрон в его атоме находится в состоянии: а) 3d6; б) 4p2. Ответ обоснуйте.
Решение:
а) последний электрон атома в d – состоянии, следовательно, это элемент побочной подгруппы. В атомах элементов побочных подгрупп последний валентный электрон попадает на d – подуровень предпоследнего
энергетического уровня, из этого следует, что n=4. Величину главного
квантового числа (4) определяет номер периода. Так как 4s-состояние заполняется электронами раньше, чем 3d, то общее число электронов, которые могут быть валентными, равно 4: 3d24s2. Значит, это элемент 4-го периода IV группы побочной подгруппы – титан (Ti).
б) электрон в атоме в p – состоянии, следовательно, это элемент
главной подгруппы. В атомах элементов главных подгрупп валентные
электроны расположены на внешнем энергетическом уровне, отсюда величина главного квантового числа 4 (номер периода). Так как p -состояние
внешнего уровня заполняется электронами после s – состояния, то общее
число валентных электронов 4, что определяет номер группы. Следова51
тельно, это элемент 4-го периода IV группы главной подгруппы – германий.
Задача 5. Определите вид химической связи в веществах: CH4, Ar,
Cа, RbF, Si, ZnSe. Какой тип кристаллической решетки они имеют в конденсированном состоянии? Укажите, какие частицы находятся в узлах их
кристаллической решетки.
Решение: для кристаллов метана и аргона характерна молекулярная
решетка. В узлах первой кристаллической решетки находятся неполярные
молекулы CH4, между которыми действуют дисперсионная составляющая
ван-дер-ваальсовых сил. Дисперсионное взаимодействие проявляется и в
кристаллах между одноатомными молекулами аргона.
Кальций – металл, для него характерна металлическая решетка.
Связь между катионами кальция, находящимися в узлах решетки, и обобществленными электронами – металлическая.
В соединении RbF типичная ионная связь, так как ΔЭОRbF = 4,1 – 0,86
= 3,24. Для кристаллов RbF характерна ионная решетка, в узлах которой
находятся положительные ионы Rb+ и отрицательные ионы F-.
Кристаллический кремний (неметалл) имеет ковалентную кристаллическую решетку типа алмаза, в которой каждый атом кремния связан с
четырьмя соседними атомами неполярной ковалентной связью.
Между атомами цинка и селена, находящимися в узлах кристаллической решетки, действует полярная ковалентная связь, так как ΔЭОZnSe = 2,4
– 1,7= 0,7.
Задача 6. Рассмотрите строение молекулы BCl3 с позиции метода
валентных связей: укажите квантовые состояния валентных электронов в
атомах, образующих связи в молекуле, форму и валентные углы между
связями; нарисуйте модель молекулы BCl3 и определите полярна ли она.
Решение: определяем валентные электроны в атомах, образующих
связи, и распределяем их по квантовым ячейкам: В…2s22p1. Атом бора в
молекуле BCl3 образует три химические связи, следовательно, он находится в возбуждённом состоянии: B*….2s12p2. Происходит гибридизация sp2
электронных орбиталей с образованием трех гибридных электронных облаков. В образовании связи у хлора участвует p-электрон: Cl…3s23p5. Связи B–Cl образованы перекрытием гибридного электронного облака атома В
и p-электронного облака атома Cl. Направленность гибридных электронных облаков трехвалентного атома В определяет направленность связей и
форму молекулы BCl3. Связи B–Cl направлены под углом 1200, молекула
BCl3 имеет форму плоского треугольника. Схема образования связей в молекуле BCl3:
52
Cl
Cl
B
B
Cl
Cl
Cl
Cl
Векторы дипольных моментов связей μсв>0 направлены в сторону
атома с большей электроотрицательностью. Суммарный дипольный момент молекулы равен 0, т.е. молекула неполярна, хотя все связи B-Cl полярны.
Задача 7. Почему комплексный ион [Cu(NH3)2]+ имеет линейное
строение?
Решение: Линейное строение иона является следствием образования
двух гибридных 4sp-орбиталей иона Cu+, которые акцептируют электронные пары молекул аммиака.
Задача 8. Комплексный ион [Zn(NH3)4]2+ имеет тетраэдрическое
строение. Какие орбитали комплексообразователя используются для образования связей с молекулами аммиака?
Решение: Тетраэдрическое строение характерно для sp3-гибридных
орбиталей. Ион Zn2+ имеет свободные 4s и 4p-орбитали, гибридизация которых приводит к образованию 4 sp3-орбиталей.
Задача 9. Почему ион [NiCl4]2- парамагнитен (тетраэдр), а ион
[Ni(CN)4]4- диамагнитен (плоский квадрат)?
Решение: Ионы Cl- слабо взаимодействуют с ионами Ni2+. Электронные пары от Cl- поступают на орбитали с главным квантовым числом
4. При этом оставшиеся у иона никеля 3d-электроны остаются неспаренными и ион [NiCl4]2- – парамагнитен.
В [Ni(CN)4]4 - вследствие sp2d-гибридизации происходит спаривание
электронов никеля и комплексный ион становится диамагнитен. Ион
[NiCl4]2- – внешнеорбитальный и высокоспиновый, так как электронные
пары от хлора поступают на свободные орбитали с более высоким квантовым числом 4.
Ион [Ni(CN)4]4- – внутриорбитальный и низкоспиновый, так как
электронные пары акцептируются гибридной орбиталью, включающей
«внутренние» 3d – подуровни атома никеля; все электроны спарены, при
гибридизации образуется комплексный ион плоского строения.
Задача 10. Константа нестойкости иона [Ag(CN)2]- составляет 1·10-21.
Вычислите концентрацию ионов серебра в 0,05 М растворе К[Ag(CN)2],
содержащем, кроме того, 0,01 моль/л KCN.
Решение: Вторичная диссоциация комплексного иона протекает по
уравнению: [Ag(CN)2]- ↔ Ag+ + 2CN- .
В присутствии избытка ионов CN-, создаваемого в результате диссоциации KCN (которую можно считать полной), это равновесие смещено
влево настолько, что количеством ионов CN- , образующимся при вторич53
ной диссоциации, можно пренебречь. Тогда [CN-] = C KCN = 0,01моль/л. По
той же причине равновесная концентрация ионов [Ag(CN)2]- может быть
приравнена общей концентрации комплексной соли (0,05 моль/л). По
условию задачи: Kн = [Ag+] · [CN-]2 / [Ag(CN)2]- = 1·10-21
Отсюда выражаем концентрацию ионов Ag+:
[Ag+] = 1·10-21 · [Ag(CN)2]- / [CN-]2 .
Подставив значения концентраций ионов CN- и [Ag(CN)2]-, получим:
[Ag+] = 1·10-21 · 0,05/(0,01)2= 5·10-19 моль/л .
Задачи и упражнения для самостоятельной работы
1.
2.
3.
4.
5.
6.
7.
8.
9.
Тема: Строение атома
Напишите электронные формулы атомов элементов с порядковыми
номерами 7 и 26. Распределите электроны этих атомов по квантовым
ячейкам. К какому электронному семейству относится каждый из этих
элементов?
Какое максимальное число электронов может быть на s-, p-, d-, fорбиталях данного энергетического уровня? Почему? Напишите
электронную формулу атома элемента с порядковым номером 32.
Какие орбитали атома заполняются электронами раньше: 4s или 3d; 5s
или 4р? Почему? Напишите электронную формулу атома элемента с
порядковым номером 21.
Какие орбитали атома заполняются электронами раньше: 4d или 5s; 6s
или 5р? Почему? Напишите электронную формулу атома элемента с
порядковым номером 43.
Напишите электронные формулы атомов элементов с порядковыми
номерами 15 и 41. Сколько свободных d-орбиталей у атомов
последнего элемента?
Что такое изотопы? Чем можно объяснить, что у большинства
элементов периодической системы атомные массы выражаются
дробным числом? Могут ли атомы разных элементов иметь
одинаковую массу? Как называются подобные атомы?
Напишите электронные формулы атомов элементов с порядковыми
номерами 21 и 23. Сколько свободных d-орбиталей в атомах этих
элементов?
Напишите электронные формулы атомов элементов с порядковыми
номерами 16 и 27. Чему равен максимальный спин р-электронов у
атомов первого и d-электронов у атомов второго элемента?
Сколько и какие значения может принимать магнитное квантовое
число ml при орбитальном числе l= 0,1,2 и 3? Какие элементы в
периодической системе называют s - р-, d- и f-элементами? Приведите
примеры.
54
10. Какие значения могут принимать квантовые числа n, l, ml и mS,
характеризующие состояние электронов в атоме? Какие значения они
принимают для внешних электронов атома магния?
11. Какие
из
электронных
формул,
отражающих
строение
невозбужденного атома некоторого элемента неверны:
а) 1s 22s 22р53s 1;
б) 1s 22s 22р63s 23р63d4;
в 1s 22s 22р63s 23р64s 2;
г) 1s 22s 22p63s 23d2 ?
Атомам каких элементов отвечают правильно составленные
электронные формулы?
12. Напишите электронные формулы атомов элементов с порядковыми
номерами 24 и 33, учитывая, что у первого происходит «провал»
одного 4s -электрона на 3d-подуровень. Чему равен максимальный
спин d-электронов у атома № 24 и р-электронов у атома № 33?
13. Составьте электронные формулы атомов элементов с порядковыми
номерами 32 и 42, учитывая, что у последнего происходит «провал»
одного 5s -электрона на 4d-подуровень. К какому электронному
семейству относится каждый из этих элементов?
14. Квантовые числа для электронов внешнего энергетического уровня
атомов некоторого элемента имеют следующие значения: n=4; l=0;
ml=0; ms=+1/2. Напишите электронную формулу атома этого элемента
и определите, сколько свободных 3d-орбиталей он содержит.
15. В чем заключается принцип Паули? Может ли быть на каком-нибудь
подуровне атома р7- или d12- электронов? Почему? Составьте
электронную формулу атома элемента с порядковым номером 22 и
укажите его валентные электроны.
1.
2.
3.
4.
5.
Тема: Периодическая система элементов
Что такое энергия ионизации? В каких единицах она выражается? Как
изменяется восстановительная активность s- и р- элементов в группах
периодической системы с увеличением порядкового номера? Почему?
Что такое электроотрицательность? Как изменяется электроотрицательность p-элементов в периоде, в группе периодической системы с
увеличением порядкового номера? Почему?
Что такое сродство к электрону? В каких единицах оно выражается?
Как изменяется окислительная активность неметаллов в периоде и в
группе периодической системы с увеличением порядкового номера?
Исходя из положения германия и технеция в периодической системе,
составьте формулы мета-, ортогерманиевой кислот и оксида технеция,
отвечающие их высшей степени окисления.
Исходя из положения германия, молибдена и рения в периодической
системе, составьте формулы водородного соединения германия, оксида
молибдена и рениевой кислоты, отвечающие их высшей степени
окисления.
55
6. Составьте формулы оксидов и гидроксидов элементов третьего
периода периодической системы, отвечающих их высшей степени
окисления. Как изменяется кислотно-основной характер этих
соединений при переходе от натрия к хлору? Напишите уравнения
реакций, доказывающих амфотерность гидроксида алюминия.
7. Какой из элементов четвертого периода – хром или селен обладает
более выраженными металлическими свойствами? Какой из этих
элементов образует газообразное соединение с водородом? Ответ
мотивируйте.
8. Марганец образует соединения, в которых он проявляет степень
окисления +2, +3, +4, +6, +7. Составьте формулы его оксидов и
гидроксидов, отвечающих этим степеням окисления. Напишите
уравнения реакций, доказывающих амфотерность гидроксида марганца
(IV).
9. У какого элемента четвертого периода – хрома или селена сильнее
выражены металлические свойства? Какой из этих элементов образует
газообразное соединение с водородом? Ответ мотивируйте строением
атомов хрома и селена.
10. Какую низшую степень окисления проявляют хлор, сера, азот и
углерод? Почему? Составьте формулы соединений алюминия с
данными элементами в этой степени окисления. Как называются
соответствующие соединения?
11. У какого из р-элементов пятой группы периодической системы –
фосфора или сурьмы сильнее выражены неметаллические свойства?
Какое из водородных соединений данных элементов более сильный
восстановитель? Ответ мотивируйте строением атома этих элементов.
12. Какую низшую и высшую степени окисления проявляют кремний,
мышьяк, селен и хлор? Почему? Составьте формулы соединений
данных элементов, отвечающих этим степеням окисления.
13. Хром образует соединения, в которых он проявляет степени окисления
+2, +3, +6. Составьте формулы его оксидов и гидроксидов,
отвечающих этим степеням окисления. Напишите уравнения реакций,
доказывающих амфотерность гидроксида хрома (III).
14. Какую низшую и высшую степени окисления проявляют углерод,
фосфор, сера и йод? Почему? Составьте формулы соединений данных
элементов, отвечающих этим степеням окисления.
15. Атомы каких элементов четвертого периода периодической системы
образуют оксид, отвечающий их высшей степени окисления Э2О5?
Какой из них дает газообразное соединение с водородом? Составьте
формулы кислот,
отвечающих этим оксидам, и изобразите их
графически?
Тема: Химическая связь и строение молекул
56
1. Какую химическую связь называют ионной? Каков механизм ее
образования? Какие свойства ионной связи отличают ее от
ковалентной? Приведите два примера типичных ионных соединений.
Напишите уравнения превращения соответствующих ионов в
нейтральные атомы.
2. Какую химическую связь называют ковалентной? Чем можно
объяснить направленность ковалентной связи?
3. Какую ковалентную связь называют полярной? Что служит
количественной мерой полярности ковалентной связи? Исходя из
значений электроотрицательности атомов, определите, какая из связей
H–I, Br–F наиболее полярна.
4. Какой способ образования ковалентной связи называют донорноакцепторным? Какие химические связи имеются в ионах NH4+ и Н3О+ ?
Укажите донор и акцептор.
5. Какую ковалентную связь называют  -связью и какую  -связью?
Разберите на примере строения молекулы азота.
6. Сколько неспаренных электронов имеет атом хлора в нормальном и
возбужденном состояниях? Распределите эти электроны по квантовым
ячейкам. Чему равна валентность хлора, обусловленная неспаренными
электронами?
7. Распределите электроны атома серы по квантовым ячейкам. Сколько
неспаренных электронов имеют ее атомы в нормальном и
возбужденном состояниях? Чему равна валентность серы,
обусловленная неспаренными электронами?
8. Что называют электрическим моментом диполя? Какая из молекул НСl,
НВr, НI имеет наибольший момент диполя? Почему?
9. Какую химическую связь называют металлической? Каков механизм ее
образования, ответ поясните на конкретном примере.
10.Какую химическую связь называют водородной? Между молекулами
каких веществ она образуется? Почему Н2О и НF, имея меньшую
молекулярную массу, плавятся и кипят при более высоких
температурах, чем их аналоги?
11.Какие электроны атома бора участвуют в образовании ковалентных
связей? Как метод валентных связей (ВС) объясняет симметричную
треугольную форму молекулы ВF3?
12.Как метод валентных связей (ВС) объясняет линейное строение
молекулы ВеCl2 и тетраэдрическое СН4?
13.Как метод валентных связей (ВС) объясняет угловое строение молекул
Н2S и линейное молекулы СО2?
14.Что следует понимать под степенью окисления атома? Определите
степень окисления атома углерода и его валентность, обусловленную
числом неспаренных электронов, в соединениях СН4, СН3ОН, НСООН,
СО2.
57
15.Какие кристаллические структуры называют ионными, атомными,
молекулярными и металлическими? Кристаллы каких веществ: алмаз,
хлорид натрия, диоксид углерода, цинк – имеют указанные структуры?
Тема: Комплексные соединения
1. Гидроксид алюминия растворяется
в NaOH с образованием
комплексного иона [Al(OH)4] . Напишите реакцию растворения и
определить заряд иона-комплексообразователя.
2. Почему гидроксид двухвалентной меди растворяется в аммиаке?
3. Приведите названия солей: K4[Fe(CN)6],
[Pt(NH3)2Cl2], Na2[PdI4],
K2[Co(NH3)2 (NO2)4].
4. Приведите примеры аквакомплексов, аммиакатов, ацидокомплексов.
5. Определите валентность комплексообразователей, координационные
числа и заряды комплексных ионов в соединениях: K3[Co(NO2)6];
Cu2[Fe(CN)6], K[Au(CN)2], [Cd(NH3)4](OH)2.
6. Напишите формулу комплексного соединения, состоящего из ионов:
К+, Cd2+, CN-.
Координационное число трехвалентного кобальта равно 6. Напишите
7.
координационные формулы следующих комплексных соединений:
Co(NO2)3·3NаNO2; Co(NO2)3·6NH3 ; Co(NO2)3·KNO2·2NH3 .
8. Координационное число двухвалентной меди равно 4. Составьте формулы аммиачного и цианистого комплексов двухвалентной меди, укажите их валентности и приведите примеры солей, в состав которых
входили бы эти комплексные ионы.
9. Укажите, в каком из указанных одномолярных растворов комплексных
солей концентрация ионов CN- будет наибольшей, а в каком –
наименьшей: K[Au(CN)2], K2[Cu(CN)4], K[Ag(CN)2], K2[Hg(CN)4]? Константы нестойкости комплексных ионов равны:
Kн([Ag(CN)2]-)= 10-21; Kн([Au(CN)2]-) = 5· 10-39;
Kн([Cu(CN)4]2-)= 5· 10-28; Kн([Нg (CN)4] 2-) = 4· 10-41.
10. Константа нестойкости комплексного иона [Fe(CN)6]4- равна 10-37, а
иона [Fe(CN)6]3- – 10-44. Какой из этих ионов наиболее прочен?
11. Изобразите распределение электронов в ионе [Ni(NH3)4]2+ парамагнетизм которого определяется двумя неспаренными электронами.
12. Комплексный ион [Cr(H2O)6]3+ является парамагнитным, изобразите его
электронную структуру.
13. Ион [Ag(CN)2]- имеет линейную строение, изобразите его электронную
структуру.
14. Изобразите распределение электронов в ионе [FeF6]4-, учитывая, что его
парамагнетизм отвечает 4 неспаренным электронам.
15. Какие комплексные соединения называются двойными солями? Напишите уравнения диссоциации солей К4[Fe(CN)6] и (NH4)2Fe(SO4)2 в
водном растворе. В каком случае выпадает осадок гидроксида железа
58
(II), если к каждой из них прилить раствор щелочи? Напишите молекулярное и ионно-молекулярное уравнение реакции.
Контрольная работа №2
1. Распределите электроны по энергетическим уровням, напишите
электронную атома, предложенного в задании. Укажите квантовые состояния валентных электронов в невозбужденном и возбужденном состояниях и распределите их по квантовым ячейкам. Определите квантовые числа
последнего (невозбужденного) состояния электрона в атоме.
2. Напишите электронные формулы предложенных ионов, укажите
квантовые состояния валентных электронов и распределите их по квантовым ячейкам.
3. В какой последовательности заполнятся электронами предложенные в задании подуровни, дайте объяснение.
4. По квантовому состоянию последнего электрона укажите положение элемента в периодической таблице (период, группу, подгруппу). Укажите, к какому электронному семейству относится заданный элемент, приведите его электронные аналоги.
5. Определите вид химической связи и тип кристаллической решетки
в предложенных веществах, укажите, какими свойствами они обладают?
6. Рассмотрите строение предложенных в задании молекул с позиции
метода валентных связей, укажите квантовые состояния валентных электронов в атомах, образующих связи в молекулах, форму и валентные углы
между связями. Нарисуйте модели молекул и определите, являются ли они
полярными.
7. Свойства комплексных соединений.
Таблица 8
№
Задание
Вари1
2
3
4
5
6
анта
ЭлеменЭнергеКвантоты
Ионы
тические вые соВеще- Строение
молекуподуров- стояния
ства
лы
ни
1
Pb
Sn4+, H 1+
3p, 3s,
4p6, 5d5
BBr3, Si, OF2, Br 2,
4p, 4s,
Zn
CH2Cl2
2+
45
2
2
Te
Cr ,Si
5s, 4p,
3p , 5d
Ва,
РH3,
4d, 4s
CoCl2, Ne
C6H6,
H2Se
2+
26
1
3
Сu
Co , As
6p, 7s,
4p , 6d
Na, NaH,
SnI4 ,
6d, 5f
NaCl
SeS2,
C2Br 4
59
4
Ti
Sn2+, N3-
4s, 3s,
3p, 3d
3d1, 5p4
Сu, О2 ,
СuCl2
5
Sn
Pt2+, H1-
4d, 4p,
3d, 4f
5d2, 6p5
Co,
CoCl2, Xe
6
Se
Mg2+,
5d3, 5p6
7
Mo
Fe3+, Te2-
4p, 5s,
3d, 3p
7s, 6d,
6p, 5f
CH4,
Cалм, SiC
Zn, ZnS,
Sе
8
Ag
Ga3+, O2-
6p, 5f,
7s, 4d
4p6, 5d5
W, CsCl,
CS2
9
Au
Fe2+,Ti4+
5s, 4s,
4d, 5p
4d1, 4p2
CaF2, F2,
Ca
10
Sc
As3+,S-2
3d3, 4p3
11
Ta
Mn2+,Ni3+
5d, 6s,
4d, 4p
3s, 4p,
4s, 3p
Au, NaH,
H2
CO2, Pd,
GaSb
12
As
Sb3+, V2+
4s, 3p,
3s, 3d
4p1, 5d7
GaCl3,
GaAs, Ga
13
Nb
Hg2+, Br-
4f, 3p,
4s, 5p
3p5, 5d2
Ca, CaH2,
CaCl2
14
Gа
Au3+,Au1+
3d1, 5p4
15
Fe
Al3+, P3-
6s, 5p,
5d, 4f
4d, 3s,
3d, 5d
GeCl4,
CdSe, Ge
BCl3, Si,
Al
16
Cd
Bi3+, Se2-
3d5, 6s1
17
Hf
Cl- , Cl7+
5s, 4p,
4d, 4s
4s, 5p,
4f, 3p,
18
At
As5+,Те-2
19
As
Ga3+, N3-
4f , 5p,
6s, 5d,
6s, 4d,
5d, 4p
60
5d4, 4p4
4p6, 6d1
3d2, 5p5
4p5, 5d6
4d2, 4p3
3d1, 4p2
SnBr2,
SbBr3,
CHI3
CH2O,
CCl4,
SiCl2
PBr3,
CS2, H2Se
H2Te,
CH3Br,
SF2
AlBr3,
C2F4,
CCl4
AsCl3,
SiCl4,
CO2
BBr3, F2,
C2Cl4
NCl3,
C6H6,
H2Te
GeBr2,
GeBr4,
CHCl3
CоS, Br3,
AsF3
OF2, Cl2,
C2Cl4
GaCl3,
GeCl4,
H2O
PCl3, Ge, NH3, H2S,
Na2SO4
C2H2
Li, LiCl,
H2Se,
LiH
CH3I,
SeF2
Ве, ВеS,
AlBr3,
Р
C2F4,CCl4
К, P2O5,
SbCl3,
В
GeCl4,
SO3
20
W
N5+, Cl3+
3s, 3p ,
4p, 4s,
4p6, 5d2
NaOH,
NH4Cl,
Cl2
BCl3, F2,
C2H 4
№ вариЗадание №7.
анта
1
Определите, чему равны заряд комплексного иона, степень
окисления и координационное число комплексообразователя в
соединениях [Сu(NH3)4]SO4, K2[PtCl6], К[Ag(CN)2]. Напишите
уравнения диссоциации этих соединений в водных растворах.
2
Составьте координационные формулы следующих комплексных
соединений серебра: AgCl·2NH3, AgCN·KCN, AgNO2·NaNO2.
Координационное число серебра равно двум. Напишите
уравнения диссоциации этих соединений в водных растворах.
3
Составьте координационные формулы следующих комплексных
соединений платины: PtCl4·6NH3, PtCl4·4NH3, PtCl4·2NH3.
Координационное число платины (IV) равно шести. Напишите
уравнение диссоциации этих соединений в водных растворах.
Какое из соединений является комплексным неэлектролитом?
4
Составьте координационные формулы следующих комплексных
соединений кобальта: СоСl3·6NH3, СоCl3·5NH3, СoCl3·4NH3.
Координационное число кобальта (IV) равно шести. Напишите
уравнения диссоциации этих соединений в водных растворах.
5
Определите, чему равны заряд комплексного иона, степень
окисления и координационное число сурьмы в соединениях
Rb[SbBr6], K[SbCl6], Na[Sb(SO4)2]. Как диссоциируют эти
соединения в водных растворах?
6
Из сочетания частиц Сr3+, Н2О, Сl- и К+ можно составить семь
координационных формул комплексных соединений хрома,
одна из которых [Cr(H2O)6]Cl3. Составьте формулы других
шести соединений и напишите уравнения их диссоциации в
водных растворах.
7
Определите, чему равны заряд комплексного иона, степень
окисления и координационное число комплексообразователя в
соединениях К4[Fe(CN)6], K4[TiCl8], K2[HgI4]. Как диссоциируют
эти соединения в водных растворах?
8
Из сочетания частиц Со3+, NH3, NO2- и K+ можно составить семь
координационных формул комплексных соединений кобальта,
одна из которых [Co(NH3)6(NO2)3. Составьте формулы других
шести соединений и напишите уравнения их диссоциации в
водных растворах.
9
Определите, чему равен заряд следующих комплексных ионов:
[Cr(H2O)4Cl2],
[HgBr4],
[Fe(CN)6],
если
3+
2+
3+
комплексообразователями являются Сr , Hg , Fe . Напишите
61
10
11
12
13
14
15
16
17
18
формулы соединений, содержащих эти комплексные ионы.
Определите, чему равен заряд комплексных ионов
[Cr(NH3)5NO3],
[Pd(NH3)Cl3],
[Ni(CN)4],
если
3+
2+
2+
комплексообразователями являются Сr , Pd , Ni . Напишите
формулы комплексных соединений, содержащих эти ионы.
Составьте координационные формулы следующих комплексных
соединений кобальта: 3NaNO2·Co(NO2)3, СoCl3·3NH3·2H2O,
2KNO2·NH3·Co(NO2)3. Координационное число кобальта (III)
равно шести. Напишите уравнения диссоциации этих
соединений в водных растворах.
Напишите выражения для констант нестойкости комплексных
ионов [Ag(NH3)2]+, [Fe(CN)6]4-, [PtCl6]2-. Чему равны степень
окисления
и
координационное
число
их
комплексообразователей?
Константы нестойкости комплексных ионов [Со(СN)4]2-,
[Hg(CN)4]2-, [Cd(CN)4]2- соответственно равны 8·10-20, 4·10-41,
1,4·10-17. В каком растворе ионов СN- больше (при равной
молярной концентрации растворов)?
Напишите выражения для констант нестойкости следующих
комплексных ионов: [Ag(CN)2]-, [Ag(NH3)2]+, [Ag(SCN)2]-, зная,
что они соответственно равны 1,0·10-21, 6,8·10-8, 2,0·10-11.
Укажите, в каком растворе больше ионов Ag (при равной
молярной концентрации растворов).
При прибавлении раствора КСN к раствору [Zn(NH)3]4SO4
образуется растворимое комплексное соединение K2[Zn(CN)4].
Напишите молекулярное и ионно-молекулярное уравнение
реакции. Константа нестойкости какого иона больше:
[Zn(NH3)4]2+ или [Zn(CN)4]2-? Почему?
Напишите уравнения диссоциации солей К3[Fe(CN)6] и
NH4Fe(SO4)2 в водном растворе. К каждой из них прилили
раствор щелочи. В каком случае выпадает осадок гидроксида
железа (III)? Напишите молекулярное и ионно-молекулярное
уравнения реакции. Какие комплексные соединения называются
двойными солями?
Составьте координационные формулы следующих комплексных
соединений платины(II), координационное число которой равно
4: PtCl2·3NH3,
PtCl2·NH3·KCl,
PtCl2·2NH3.
Напишите
уравнения диссоциации этих соединений в водных растворах.
Какое из соединений является комплексным неэлектролитом?
Какие комплексные соединения называются двойными солями?
Напишите уравнения диссоциации солей К3[Fe(CN)6] и
KA1(SO4)2 в водном растворе. К каждой из них прилили раствор
щелочи. Напишите молекулярное и ионно-молекулярное
62
19
20
уравнение реакции.
Хлорид серебра растворяется в растворах аммиака и
тиосульфата натрия. Дайте этому объяснение и напишите
молекулярные
и
ионно-молекулярные
уравнения
соответствующих реакции.
Константы нестойкости комплексных ионов [Co(NH3)6]3+,
[Fe(CN)6]4-, [Fe(CN)6]3- соответственно равны 6,2·10-36; 1,0·10-37;
1,0·10-44. Какой из этих ионов является более прочным?
Напишите выражения для констант нестойкости указанных
комплексных ионов и формулы соединений, содержащих эти
ионы.
Глава 3. Термодинамика химических процессов
Основные понятия термодинамики
Термодинамика – это наука, изучающая законы обмена энергией
между системой и окружающей средой.
Термодинамическая система – тело или группа тел, находящихся
во взаимодействии, мысленно или реально обособленные от окружающей
среды (под взаимодействием подразумевается обмен энергией и веществом).
Различают три типа термодинамических систем в зависимости от их
возможности обмениваться с окружающей средой веществом и энергией.
Система, которая не обменивается с окружающей средой ни веществом, ни
энергией – изолированная система. Закрытая система – система, которая обменивается с окружающей средой энергией, но не обменивается веществом. Открытая система – система, которая обменивается с окружающей средой и веществом, и энергией.
Гомогенная система – система, внутри которой нет поверхностей,
разделяющих отличающиеся по свойствам части системы (фазы). Гетерогенная система – система, внутри которой присутствуют поверхности,
разделяющие отличающиеся по свойствам части системы. Фаза – совокупность гомогенных частей гетерогенной системы, одинаковых по физическим и химическим свойствам, отделенная от других частей системы видимыми поверхностями раздела.
Параметры состояния – величины, характеризующие какое-либо
макроскопическое свойство рассматриваемой системы, позволяющие
определить состояние системы непосредственным измерением этих величин. Различают экстенсивные параметры состояния, пропорциональные
количеству вещества системы (масса, объем, внутренняя энергия), и интенсивные параметры состояния, не зависящие от количества вещества
системы (давление, температура, вязкость).
Состояние системы определяется различными функциями, называемыми функциями состояния. Функция состояния – функция независи63
мых параметров системы, изменение которой не зависит от пути перехода
системы из начального состояния в конечное, а зависит только от состояния системы в этих точках. Для функции состояния можно измерить или
рассчитать лишь ее изменение (∆). Изменение функции состояния рассматривается как разность ее значений в конечном и исходном состояниях
системы, изменения функций состояния могут принимать как положительные, так и отрицательные значения.
Термодинамический процесс – всякое изменение (хотя бы одного
параметра) термодинамического состояния системы.
Обратимый процесс – процесс, допускающий возможность возвращения системы в исходное состояние без того, чтобы в окружающей
среде остались какие-либо изменения.
Равновесный процесс – процесс, при котором система проходит
через непрерывный ряд состояний, бесконечно близких к состоянию равновесия. Характерные особенности равновесного процесса:
1) бесконечно малая разность действующих и противодействующих
сил;
2) совершение системой в прямом процессе максимальной работы;
3) бесконечно медленное течение процесса, связанное с бесконечно малой разностью действующих сил и бесконечно большим числом
промежуточных состояний.
Самопроизвольный процесс – процесс, который может протекать
без затраты работы извне, причем в результате может быть получена работа в количестве, пропорциональном произошедшему изменению состояния
системы. Самопроизвольный процесс может протекать обратимо или необратимо.
Несамопроизвольный процесс – процесс, для протекания которого
требуется затрата работы извне в количестве, пропорциональном производимому изменению состояния системы.
Энергия – мера способности системы совершать работу, общая качественная мера движения и взаимодействия материи. Энергия является
неотъемлемым свойством материи. Различают потенциальную энергию,
обусловленную положением тела в поле некоторых сил, и кинетическую
энергию, обусловленную изменением положения тела в пространстве.
Внутренняя энергия (U) – сумма энергии хаотического (теплового)
движения всех микрочастиц системы (молекул, атомов, ионов и т. д.),
энергии колебательных и вращательных движений атомов в молекулах и
энергии взаимодействия этих частиц, т. е. полный запас энергии в веществе – кинетической и потенциальной. Измерить и рассчитать абсолютное
значение внутренней энергии невозможно, можно лишь с достаточной
точностью измерить ее изменение (∆U). Это характерно для всех функций
состояния системы. Поскольку для нахождения изменения функций состо-
64
яния и сравнения их между собой необходим единый уровень отсчета, то
ввели понятие стандартного состояния вещества.
Стандартные величины – величины, определенные для веществ,
находящихся в стандартном состоянии (обозначаются надстрочным индексом 0). Одно из возможных стандартных состояний характеризуется стандартными условиями: давлением (p0), составом (молярная концентрация
С0), температурой (t0 или Т0):
p0= 101325 Па = 1 атм; С0= 1 моль/л; t0=250C (298,15 К)
Стандартные величины для веществ приводятся в справочниках.
Теплота (Q) – форма передачи энергии путем неупорядоченного движения образующих тело частиц (молекул, атомов и т. д.). Количественной
мерой теплоты служит количество теплоты, т.е. количество энергии, получаемой (+ Q) или отдаваемой системой (– Q) при теплообмене. Теплота
измеряется в единицах энергии: Дж, кал (1 кал = 4,184 Дж).
Работа (A) – форма передачи энергии путем упорядоченного движения частиц (макроскопических масс) под действием каких-либо сил. – A
работа, совершенная системой против внешних сил; + A работа, совершенная внешними силами над системой (Дж). Вся работа делится на механическую работу расширения (или сжатия) и прочие виды работы (полезная работа). Механическая работа представляется произведением силы
на перемещение. Электрическая работа рассматривается как произведение
заряда на разность потенциалов. Работа расширения идеального газа равна
произведению давления на изменение объема. Теплота и работа являются
эквивалентными формами передачи энергии.
Первый закон термодинамики
Первое начало (закон) термодинамики устанавливает соотношение
между теплотой Q, работой A и изменением внутренней энергии системы
∆U: если веществу или совокупности веществ (системе) сообщить извне
энергию Q, то эта энергия будет расходоваться на изменение внутренней
энергии системы ∆U и на совершение работы системой против внешних
сил: Q = ∆U + A.
Этот закон представляет собой одну из форм закона сохранения энергии. Если в результате химической реакции система поглотила количество
теплоты Q и совершила работу А, то изменение внутренней энергии определяется уравнением: ∆U = Q – A.
Если в ходе реакции при постоянном давлении (p=const, изобарный
процесс) совершается только работа расширения, то А = p∆V (∆V=V2-V1).
Тогда ∆U = Qp - p∆V или Qp = ∆U +p∆V = (U2 – U1) + p(V2 – V1). Группируя члены правой части равенства, получаем: Qp = (U2 + pV2) – (U1 +pV1).
Величину U + pV называют энтальпией (теплосодержанием) и обозначают символом Н:
H = U + pV .
65
Тепловой эффект изобарного процесса (реакции, протекающей при
постоянном давлении) равен изменению энтальпии системы:
Qp = H2 – H1 = ∆H .
Если реакция протекает при постоянном объеме (∆V=0, изохорный
процесс), то работа расширения A=p∆V=0, тогда
Qv = ∆U,
т.е. тепловой эффект реакции, протекающей при постоянном объеме, равен изменению внутренней энергии.
Теплоемкость – количество теплоты, поглощаемое телом при нагревании на 10С или 1 К. Теплоемкость единицы массы вещества называют
удельной теплоемкостью [Дж/кг·К]. Теплоемкость 1 моля вещества – молярная (мольная) теплоемкость [Дж/моль· К].
Экзотермические – реакции, протекающие с выделением тепла (+Q),
для них ∆Н < 0.
Эндотермические –реакции, протекающие с поглощением тепла (–Q),
для них ∆Н > 0.
Уравнения реакций, в которых указывается значение теплового эффекта, называются термохимическими. В таких уравнениях обязательно
указываются агрегатные состояния веществ. Например, реакция горения
этана выражается термохимическим уравнением:
С2Н 6(г) + 3,5O2 = 2СО2(г) + 3Н2О(ж), ∆Н = –1559,84 кДж.
Стандартная энтальпия образования химического соединения
0
(∆Н обр или ∆Н0298) – стандартное изменение энтальпии в результате реакции образования 1 моля этого вещества из простых веществ, взятых в
том агрегатном состоянии, в котором они находятся при стандартных
условиях и данной температуре [кДж/моль].
Стандартные энтальпии образования простых веществ принимаются равными нулю (рассматриваются аллотропные формы, устойчивые при
стандартных условиях и T=0 K).
Стандартная энтальпия химической связи – это изменение энтальпии в реакции образования одного моля двухатомных молекул (или других
двухатомных частиц) из атомов веществ, находящихся в газообразном состоянии:
H(г) + Cl(г) = HCl(г)
С(г) + H(г) = CH(г)
Образование химической связи – процесс экзотермический, и поэтому энтальпия его всегда имеет отрицательное значение. Обратная реакция
– диссоциации двухатомной молекулы (частицы) – процесс эндотермический. Энтальпия этого процесса называется энтальпией разрыва связи, она
имеет положительное значение. Для многоатомных молекул эта величина
является условной и отвечает энтальпии такого процесса, при котором
данная химическая связь разрывается, а все остальные связи остаются без
изменения. Реально для многоатомных молекул можно определить лишь
66
среднее значение энтальпии разрыва химической связи, потому что энтальпии последовательных разрывов связей в них не равноценны.
Химические связи условно подразделяют на прочные или сильные –
энергия разрыва связи > 500 кДж/моль; слабые от ~ 100 до ~ 20 кДж/моль,
водородные ~ 15 кДж/моль. Межмолекулярные (ван-дер-ваальсовы) силы
имеют энергию связи ~ 5 кДж/моль и менее.
Закон Гесса: тепловой эффект химической реакции или физикохимического процесса не зависит от пути перехода системы из начального состояния в конечное, а определяется только природой и состоянием
исходных и конечных веществ.
С + О2 = СО2, ∆Н= –393,5 кДж/моль;
С + ½ О2 = СО, ∆Н1= –110,5 кДж/моль;
СО + ½ О2 = СО2, ∆Н2= –283,0 кДж/моль;
∆Н = ∆Н1 + ∆Н2= –110,5+(–283,0) = –393,5 кДж/моль.
Следствия из закона Гесса:
1. Тепловой эффект реакции равен разности между суммой стандартных энтальпий образования продуктов реакции и суммой стандартных энтальпий образования исходных веществ с учетом коэффициентов реакции.
∆Н0р-ии= ∑ i ∆Н0298(конечн.прод.) – ∑ j∆Н 0298(исходн.в-в);
для реакции A + bB = cC + dD:
∆Н0р-ии = (c∆Н0298(C) +d∆Н0298(D)) – (a∆Н0298(A) + b∆Н0298(B)).
2. Тепловой эффект прямой реакции равен тепловому эффекту обратной реакции, взятому с обратным знаком.
3. Если в результате ряда последовательных реакций система переходит в состояние, полностью совпадающее с исходным, то сумма тепловых
эффектов этого ряда реакций равна нулю.
А→В, ∆Н1; В→С, ∆Н2 ; С→А, ∆Н3;
∆Н1 + ∆Н2 + ∆Н3 = 0.
Второй закон термодинамики
Ограниченность первого начала термодинамики заключается в том,
что оно не определяет направления обмена тепловой энергией между телами. Первое начало не запрещает переход тепла от менее нагретого к более нагретому телу. Суть второго начала состоит в том, что оно определяет направление самопроизвольного протекания процессов, в том числе и
химических реакций.
Самопроизвольным называется процесс, который протекает самостоятельно без воздействия внешних факторов. Для объяснения протекания таких процессов применяется еще одна функция состояния, которая с
учетом первого начала термодинамики позволяет предсказать направление
67
протекания процесса в изолированных системах. Такой функцией состояния является энтропия (S).
Интерпретировать энтропию можно как с точки зрения макроскопического состояния системы, так и с точки зрения микроскопического. Энтропию можно рассматривать как меру неупорядоченности частиц, составляющих систему. Размерность энтропии [S] = Дж/моль·К.
Второй закон (начало) термодинамики: в изолированной системе
самопроизвольно протекают только те процессы, которые сопровождаются увеличением энтропии.
В самопроизвольном процессе ∆S>0; в равновесном процессе ∆S=0;
в несамопроизвольном процессе ∆S<0.
Значение энтропии для простых веществ не равно нулю. С повышением температуры энтропия всегда возрастает. Возрастает она в процессах
плавления, испарения, сублимации и уменьшается в процессах конденсации и кристаллизации. Энтропия возрастает при увеличении числа молекул газообразных веществ и понижается при их уменьшении.
Третий закон термодинамики
Третье начало (закон) термодинамики: энтропия любого индивидуального вещества, существующего в виде идеального кристалла при
температуре, равной абсолютному нулю, равна нулю.
Таким образом, энтропия является единственной функцией состояния, для которой можно рассчитать абсолютное значение.
Расчет ∆S реакции: энтропия является функцией состояния, поэтому ее изменение не зависит от пути перехода системы из одного состояния
в другое, а определяется только значениями энтропии системы в исходном
и конечном состояниях:
∆S0р-ии = ∑i∆S0298(конечн.прод.) - ∑ j∆S0298(исходн.в-в).
Таким образом, вероятность протекания процесса в изолированной
системе определяют два фактора: энтальпийный и энтропийный. Самопроизвольному течению реакции способствует понижение энтальпии (∆H<0) и
повышение энтропии (∆S>0).
Энергия Гиббса
Для оценки самопроизвольности процесса, протекающего при постоянной температуре и давлении, используют термодинамическую функцию,
называемую изобарно-изотермическим потенциалом или энергией Гиббса (G):
∆G = ∆H – T ∆S .
Изменение энергии Гиббса учитывает как энтальпийный, так и энтропийный факторы реакции и, кроме того, влияние температуры. Использовать данную функцию можно как для изолированных, так и для закры-
68
тых систем, но если процесс протекает при постоянных температуре и
давлении.
В закрытых системах при постоянных температуре и давлении самопризвольно могут протекать только те процессы, которые сопровождаются уменьшением энергии Гиббса системы (∆G<0).
Если ∆G=0, то система находится в состоянии равновесия. Из нескольких реакций, имеющих отрицательные значения ∆G, наиболее предпочтительна та, для которой значение ∆G минимально.
Расчет ∆G реакции: энергия Гиббса является функцией состояния и
рассчитывается по формуле:
∆G0р-ии = ∑i∆G0298(конечн. прод.) - ∑ j∆G0298(исходн. в-в).
∆G имеет размерность кДж/моль. Для простых веществ свободная
энергия Гиббса равна нулю.
Рассмотрим возможность самопроизвольного протекания химической
реакции в зависимости от знака энтальпийного и энтропийного членов в
выражении изменения свободной энергии. Возможны четыре случая.
а) когда ∆H<0, а ∆S>0, то изменение свободной энергии всегда отрицательно ∆G<0; такие реакции самопроизвольно протекают в прямом
направлении;
б) когда ∆H>0, а ∆S<0; такие реакции в прямом направлении самопроизвольно не идут, самопроизвольно протекает обратная реакция;
в) когда как ∆H<0, так и ∆S<0; в этом случае направление самопроизвольного протекания процесса определяется температурой: при низкой
температуре самопроизвольно идет прямая реакция, так как по абсолютной
величине изменение энтальпии больше произведения температуры на изменение энтропии; при равновесии изменение свободной энергии равно
нулю (∆G = 0), а температура, при которой система находится в равновесии, равна: T = ∆H/∆S;
г) когда изменение ∆H>0 и ∆S>0 направление реакции также зависит
от температуры: при низкой температуре самопроизвольно будет протекать обратная реакция, а при высокой – прямая.
Это характерно для реакций диссоциации, например: N2O4=2NO2.
Для данной реакции ∆H0298= 57,3 кДж; ∆S0298=176,6 Дж/К; ∆G0298=4,7 кДж.
Температура, при которой устанавливается состояние равновесия 324,5 К,
ниже этой температуры самопроизвольно протекает реакция димеризации,
а выше – реакция диссоциации.
Литература
1. Глинка Н. Л. Общая химия. – М.: Химия, 2006. – 720 с.
2. Глинка Н.Л. Задачи и упражнения по общей химии. – Л. Химия,
1985. – 264 с.
69
3. Карапетьянц М.Х., Дракин С.И. Общая и неорганическая химия. –
М.: Химия, 1993. – 558 с.
4. Коровин В.Н. Общая химия. – М.: Высшая школа, 2000. – 557 с.
5. Коровин Н.В., Масленникова Г.Н. и др. Курс общей химии. – М.:
Высшая школа, 1990. – 445 с.
6. Суворов А.В., Никольский А.Б. Общая химия. – СПб: Химиздат,
2001. – 512 с.
Примеры решения типовых задач
Задача 1. Вычислите теплоту сгорания этилового спирта С2Н5ОН.
Решение: уравнение реакции горения этилового спирта имеет вид:
С2Н5ОН(ж) + 3О2 = 2СО2 + 3Н2О(газ), ∆Н=?
∆Нр-ии = (2 ∆Н0298(СО2) + ∆Н0298(Н2О(газ))) – ∆Н0298(С2Н5ОН(ж));
∆Н р-ии = 2(–393,51) + 3(–241,84) – (–277,63) = –1234,91 кДж/моль.
Задача 2. Рассчитайте тепловой эффект реакции окисления кремния,
протекающией по уравнению: Si(т)+2H2O(ж)=SiO2(k)+2H2(г), по стандартным
теплотам образования веществ.
Решение: находим стандартные теплоты образования веществ (см.
приложение):
∆Н0298(SiO2(k))= – 859,3 кДж/моль; ∆Н0298(H2O(ж))= – 285,8 кДж/моль.
Тепловой эффект реакции ∆Н0298 рассчитываем по уравнению первого
следствия из закона Гесса:
∆Н0р-ии= ∑ i ∆Н0298(конечн.прод.) – ∑ j∆Н 0298(исходн.в-в).
Учитывая, что теплоты образования простых веществ равны нулю, находим тепловой эффект реакции:
∆Н0р-ии=∆Н0298(SiO2(k)) – 2∆Н0298(H2O(ж))= –859,3 –2(– 285,8)= –287,7 кДж.
Задача 3. Исходя из теплоты образования газообразного диоксида
углерода ∆Н0298= –393,5 кДж/моль и термохимического уравнения:
С (графит) + 2N2O (г) = СО2 (г) + 2N2 (г) ; ∆Н = – 557,5 кДж , вычислите теплоту
образования N2O (г).
Решение: тепловой эффект реакции ∆Н0298 реакции равен:
∆Н0р-ии= ∑ i ∆Н0298(конечн.прод.) – ∑ j∆Н 0298(исходн.в-в);
∆Н0р-ии=(∆Н0298(СО2 (г)) + 2∆Н0298(N2 (г)))– (∆Н0298 С (графит))+ 2∆Н0298N2O (г)),
отсюда: ∆Н0р-ии=∆Н0298(СО2 (г)) – 2∆Н0298N2O (г), т.к. ∆Н0298 простых веществ
равны нулю.
Вычисляем теплоту образования N2O (г):
∆Н0298N2O=(∆Н0298(СО2 (г)) – ∆Н0р-ии)/2=((– 393,5– (–557,5))/2=82 кДж/моль.
Задача 4. Вычислите ∆G0298 для реакции протекающей по уравнению: TiO2(k) + 2C(k) = Ti(k)+ 2CO(г), если известно, что ∆Н0298=718 кДж,
∆S=365 Дж/К. Возможно ли протекание данной реакции в стандартных
условиях?
Решение: Изменение изобарно-изотермического потенциала определяется по уравнению: ΔG = ΔН – TΔS.
70
∆G0р-ии=718 – 298·365·10–3 = 609,2 кДж. Знак ∆G0 показывает направление
самопроизвольного протекания реакции; ∆G0р-ии > 0, поэтому в стандартных условиях данная реакция самопроизвольно протекать не будет.
Задача 5. Определите температуру, при которой находится в равновесии система: NH3 (г) + HCl (г) ↔ NH4Cl (к) .
Решение: изменение изобарно-изотермического потенциала определяется по уравнению: ΔG = ΔН – TΔS , т.к. при равновесии изменение свободной энергии равно нулю (∆G = 0), то температура, при которой система
находится в равновесии, равна T = ∆H/∆S.
Определяем тепловой эффект в реакции образования хлорида аммония из газообразных аммиака и хлористого водорода по уравнению первого следствия из закона Гесса:
∆Н0р-ии= ∆Н0298(NH4Cl(k)) – (∆Н0298(NH3(г)) +∆Н0298(HCl(г))) (стандартные
теплоты образования веществ см. приложение);
∆Н0р-ии= –315,4–(–92,3) –(–46,2)= –176,9 кДж.
Определяем энтропию реакции:
0
∆S р-ии= ∆S0298(NH4Cl (k)) – (∆S0298(NH3 (г)) + ∆S0298(HCl (г)));
∆S0р-ии= 94,6–192,5 –186,7= –284,6 Дж или –284,6·10-3 кДж.
Температура, при которой система находится в равновесии:
T = ∆H/∆S= – 176,9 /–284,6·10-3= 621,6 К.
Задача 6. Не производя вычислений, определить знак изменения энтропии в следующих реакциях:
NH4NO3 (к) = N2O (г) + 2Н2О(г)
(1);
2Н2 (г) + О2 (г) = 2Н2О(г)
(2);
2Н2 (г) + О2 (г) = 2Н2О(ж)
(3).
Решение: в реакции (1) 1 моль вещества в кристаллическом состоянии образует 3 моля газов, следовательно, ∆S1> 0. В реакциях (2) и (3)
уменьшается как общее число молей, так и число молей газообразных веществ, так что ∆S2 < 0 и ∆S3 > 0. При этом ∆S3 имеет более отрицательное
значение, чем ∆S2 , так как ∆S2 (Н2О(ж)) < ∆S3 (Н2О(г)).
Задача 7. Определить знаки ∆Н, ∆S, ∆G для реакции:
АВ(к) + В2(г) = АВ3(к), протекающей при температуре 298 К в прямом
направлении. Как будет изменяться значение ∆G с ростом температуры?
Решение: самопроизвольное протекание реакции указывает на то,
что для нее ∆G < 0. В результате реакции общее число частиц в системе
уменьшается, причем расходуется газ В2, а образуется кристаллическое
вещество АВ3; это означает, что система переходит в состояние с более
высокой упорядоченностью, т. е. для рассматриваемой реакции ∆S < 0. Таким образом, в уравнении ΔG = ΔН – TΔS величина ΔG – отрицательна, а
второй член правой части уравнения (– TΔS) положителен. Это возможно
только в том случае, если ΔН < 0. С ростом температуры положительное
значение члена – TΔS в уравнении возрастает, так что величина ΔG будет
становиться менее отрицательной.
71
Задачи и упражнения для самостоятельной работы
1. Укажите гомогенную систему:
а) 3Fe + 4Н2О(пар) = Fe3O4 + 4Н2(г) ; б) С (тв) + О2(г) = СО2(г);
в) СО(г) + 2Н2(г) =СН3ОН(г);
г)2Сr (тв) + ЗС12(г) =2СгС13(тв).
2. Предскажите знак изменения энтропии в реакции:
2Н2(г)+О2(г)=2Н2О(ж):
3. Тепловой эффект реакции SO2(г) + 2H2S(г) = 3S(тв) + 2Н2О(ж) равен
–234,50 кДж. Определите стандартную теплоту (энтальпию) образования H2S(г) , если ∆H°298(SO2)= –296,9 кДж/моль; ∆Н0298(Н2О(ж)) =
–285,8 кДж/моль.
4. Окисление аммиака протекает по уравнению:
4NH3(г) + 3О2(г) = 2N2(г) + 6Н2О(ж); ∆Н° = –1528кДж.
Определите
стандартную теплоту образования NH3(г), если
∆Н°298(Н2О(ж))= –285,8 кДж/моль.
5. Определите тепловой эффект реакции (∆Н°298):
Fe2O3 + 2A1 = 2Fe + А12О3, если ∆Н°298(А12О3)= –1676,0 кДж/моль;
∆H0298(Fe2O3)= – 822,2 кДж/моль.
6. Определите тепловой эффект реакции:
NaH(к)+ Н2О(ж)= NaOH(р)+ Н2(г) по стандартным теплотам образования веществ, участвующих в реакции, если ∆H°298(NaHк) = – 56,94
кДж/моль, ∆Н°298 (NaOHp) = – 469,47 кДж/моль, ∆Н0298(Н2Ож) = –285,8
кДж/моль.
7. Исходя из теплот реакций окисления As2O3 кислородом и озоном
As2O3 + O2 = As2O5,
ΔH° = – 271кДж,
3As2O3 + 2О3 = 3As2O5, ΔH° = – 1096кДж ,
вычислите теплоту образования озона из молекулярного кислорода.
8. Пользуясь значениями ΔG° образования отдельных соединений, вычислите ΔG° реакций:
а) СОСl2(г) = CO(г) + Cl2(г) ; б) SO2(г) + NO2(г) = SO3(г) + NO(г).
Определите возможность протекания реакций в стандартных условиях.
9. При сгорании 1 л ацетилена, взятого при нормальных условиях, выделяется 58,2 кДж. Вычислить теплоту образования С2Н2(г), если теплоты образования СО2 и Н2О(ж) равны –393 и –286 кДж/моль.
10.При восстановлении 12,7 г оксида меди (II) углем (с образованием
СО) поглощается 8,24 кДж. Определить ∆Н0298 образования СuО.
11.Определите знак изменения энтропии для реакции: АВ(к) + В2(г) =
АВ3(г). Возможно ли протекание этой реакции в стандартных условиях?
72
12.Определите знаки ∆Н, ∆S, ∆G для реакции: 2А2(к) + В2(г) = 2А2В(ж),
протекающей при температуре 298 К в прямом направлении. Будет
ли ∆G возрастать или убывать с ростом температуры?
13.Рассчитайте значения ∆ G °298 следующих реакций и установите, в
каком направлении они могут протекать самопроизвольно в
стандартных условиях при 25 °С:
а) NiO(к) + Рb(к) = Ni(к) + РbО(к) ; б) Рb(к) + СuО(к) = РbО(к) + Сu(к);
в) 8Аl(к) + 3Fe3O4(к) = 9Fe(к) + 4А12О3(к) .
14.Какие из перечисленных оксидов могут быть восстановлены алюминием при 298 К: CaO, FeO, CuO, РbО, Fe2O3, Cr2O3?
15.Вычислите значение ∆Н0298 для протекающих в организме реакций
превращения глюкозы: а) С6Н12О6 (к) =2С2Н5ОН (ж)+ 2CO2(г);
б) С6Н12О6 (к) + 6О2 (г) =6Н2О (ж)+ 6CO2(г).
Какая из этих реакций поставляет организму больше энергии?
Глава 4. Химическая кинетика. Обратимость химических реакций.
Химическое равновесие
Основные понятия химической кинетики
Химическая кинетика – раздел химии, который изучает скорость
химических реакций и ее зависимость от различных факторов.
Скорость химических реакций определяется изменением концентрации одного из веществ, участвующих в реакции, в единицу времени:
υ ср= ±∆С/∆t (моль / л · с),
где υ ср – средняя скорость; С – молярная концентрация вещества; ∆ –
знак разности; t – время.
Знак (+) берется в том случае, если скорость определяется по изменению концентрации продуктов реакции; знак (–) – если скорость определяется по изменению концентрации исходных веществ.
Понятие «концентрация» применимо лишь к гомогенной системе,
поэтому приведенное выше определение скорости может быть дано только
для гомогенных реакций: газофазных или протекающих в растворах.
В гетерогенных реакциях взаимодействие протекает на границе раздела двух фаз. Для таких реакций скорость определяется изменением количества вещества в единицу времени на единице поверхности раздела
фаз:
υ = ±∆n/∆t · S (моль / с · м2),
где n – количество вещества; ∆t – промежуток времени от t1 до t2; S – поверхности раздела фаз.
Изменение количества вещества, по которому определяют скорость
химической реакции, – это внешний фактор, наблюдаемый исследователем. По сути, все процессы осуществляются на микроуровне. Для того
чтобы частицы прореагировали, они, как минимум, должны столкнуться,
причем столкнуться эффективно: чтобы в них разрушились или ослабли
73
существующие связи и смогли образоваться новые. Для этого реагирующие частицы должны обладать достаточной энергией.
Минимальный избыток энергии, которым должны обладать частицы
исходных веществ по сравнению со средней энергией молекул, чтобы произошло их результативное столкновение, называют энергией активации.
Таким образом, на пути всех частиц, вступающих в реакцию, имеется
энергетический барьер, равный энергии активации Еа. Если барьер невелик, то большинство частиц может его преодолеть, поэтому скорость реакции высокая; если энергия активации значительна, реакция протекает
медленно.
Факторы, влияющие на скорость химических реакций
Скорость реакции зависит от природы и концентрации реагирующих веществ, температуры, давления, присутствия катализатора и его
свойств, степени измельчения твердой фазы, от облучения квантами света
и других факторов.
1. Природа реагирующих веществ. Под природой реагирующих
веществ понимают природу химической связи в молекулах реагентов и ее
прочность. Разрыв связей и образование новых связей определяют величину константы скорости, и, тем самым, влияют на процесс протекания реакции.
Величина энергии активации является тем фактором, посредством
которого сказывается влияние природы реагирующих веществ на скорость
реакции: если энергия активации мала, то скорость такой реакции большая, например, все реакции ионного обмена протекают практически мгновенно, очень велики скорости реакций с участием радикалов; если энергия
активации велика, то скорость такой реакции мала, например, это многие
реакции между веществами с ковалентными химическими связями, между
газообразными веществами.
2. Концентрация реагирующих веществ. Количественную характеристику зависимости скорости реакции от концентрации устанавливает
закон действующих масс (Гульдберг и Вааге, 1867г.): скорость химической реакции прямо пропорциональна концентрации реагирующих веществ, возведенных в степени, равные стехиометрическим коэффициентам в уравнении реакции.
Для реакции аА + bВ =сС + dD математическое выражение закона
действующих масс имеет вид:
υ =k·[А]а ·[ В]b или υ =k·САа ·СВb ,
где v – скорость химической реакции; [А], [В] или СА, СВ – молярные концентрации реагирующих веществ; а, b – стехиометрические коэффициенты
реагирующих веществ; k – коэффициент пропорциональности.
Подобные выражения называют кинетическими уравнениями реакций. Коэффициент пропорциональности k в кинетическом уравнении
74
называют константой скорости. Константа скорости численно равна
скорости реакции при концентрациях реагирующих веществ 1 моль/л; k
зависит от природы реагирующих веществ, температуры, способа выражения концентрации, но не зависит от величины концентрации реагирующих
веществ.
Для гетерогенных реакций концентрации твердых веществ в уравнение скорости не включаются, так как реакция идет только на поверхности раздела фаз. Например, кинетическое уравнение реакции горения угля
С(тв)+О2(г)=СО2(г) будет иметь вид : υ =k·[О2].
Сумма показателей степеней концентраций реагентов в кинетическом уравнении реакции называется порядком химической реакции. Порядок по данному веществу (частный порядок) определяется как показатель степени при концентрации этого вещества. Например, общий порядок
реакции: H2 + I2 = 2HI равен двум, частные порядки по водороду и по иоду
равны единице, т.к. υ=k · [H2] · [I2].
3. Температура. Зависимость скорости реакции от температуры выражается правилом Вант-Гоффа (1884г.): при повышении температуры
на каждые десять градусов скорость реакции возрастает примерно в 2 4 раза. Математическое выражение правила Вант-Гоффа:
υ 2 = υ 1· γ ∆t/10
где υ 1 и υ 2 – скорость реакции при t1 и t2; ∆t = t2 – t1; γ – температурный
коэффициент, показывающий, во сколько раз увеличивается скорость реакции при повышении температуры на 10 ºС.
Зависимость константы скорости реакции от температуры выражается уравнением Аррениуса (1889г.):
k = A· е – Е/RT
где Е – энергия активации, кал/моль; Дж/моль; е – основание натурального логарифма; А – постоянная, не зависящая от температуры; R – газовая
постоянная.
Влияние температуры на скорость реакции объясняется тем, что при
повышении температуры резко (в геометрической прогрессии) возрастает
число активных молекул.
4. Поверхность реагирующих веществ и давление. В гетерогенных
реакциях взаимодействие веществ происходит на поверхности раздела фаз,
и чем больше площадь этой поверхности, тем выше скорость реакции. В
данном случае увеличение поверхности соприкосновения соответствует
увеличению концентрации реагирующих веществ.
На скорость реакций с участием газообразных веществ, влияет изменение давления. Уменьшение или увеличение давления приводит к соответствующим изменениям объема, а поскольку количества веществ при
этом не изменяются, будут изменяться концентрации реагирующих веществ.
75
5. Катализ. Одним из методов ускорения химической реакции является катализ, который осуществляется при помощи введения катализаторов, увеличивающих скорость реакции, но не расходующихся в результате
ее протекания. Механизм действия катализатора сводится к уменьшению
энергии активации реакции, т.е. к уменьшению разности между средней
энергией активных молекул и средней энергией молекул исходных веществ. Скорость химической реакции при этом увеличивается. Как правило, термин «катализатор» применяют к тем веществам, которые увеличивают скорость химической реакции. Вещества, которые уменьшают скорость реакции, называют ингибиторами.
Катализаторы принимают самое непосредственное участие в процессе, но по окончании его могут быть выделены из реакционной смеси в исходном количестве. Для катализаторов характерна селективность, т.е.
способность влиять на прохождение реакции в определённом направлении,
поэтому из одних и тех же исходных веществ могут быть получены различные продукты в зависимости от используемого катализатора.
Особое место занимают биокатализаторы – ферменты, представляющие собой белки. Ферменты оказывают влияние на скорости строго
определенных реакций, т. е. обладают очень высокой селективностью.
Ферменты ускоряют реакции в миллиарды и триллионы раз при комнатной
температуре. При повышенной температуре они теряют свою активность,
так как происходит денатурация белков.
Различают два типа катализа: гомогенный катализ, когда катализатор и исходные вещества находятся в одной фазе, и гетерогенный, когда
катализатор и исходные вещества находятся в разных фазах, т.е. реакции
протекают на поверхности катализатора. Катализатор не влияет на состояние равновесия в системе, а лишь изменяет скорость, с которой достигается это состояние. Это следует из того, что равновесию отвечает минимум
изобарно-изотермического потенциала (энергии Гиббса), и константа равновесия имеет одинаковое значение, как в присутствии катализатора, так и
без него.
Действие гомогенного катализатора заключается в том, что он реагирует с одним из исходных веществ с образованием промежуточного соединения, которое, в свою очередь, вступает в химические реакции с другим исходным веществом, давая желаемый продукт реакции и «освобождая» катализатор. Таким образом, при гомогенном катализе процесс протекает в несколько стадий, но с меньшими значениями энергии активации
для каждой стадии, чем для прямого некаталитического процесса.
Пусть вещество A реагирует с веществом B, образуя соединение AB:
A + B = AB .
Реакция протекает с незначительной скоростью. При добавлении катализатора K протекают реакции: A + K = AK и AK + B = AB + K .
Сложив эти два уравнения, получим: A + B = AB .
76
Примером реакции, протекающей с участием гомогенного катализатора, может служить реакция окисления оксида серы (IV) до оксида серы
(VI): без катализатора:
SO2 + 0,5O2 = SO3 ;
с катализатором NO2: SO2 + NO2 = SO3 + NO , NO + 0,5O2 = NO2 .
Действие гетерогенного катализатора заключается в том, что молекулы газа (или жидкости) адсорбируются на поверхности кристалла катализатора, что приводит к перераспределению электронной плотности в
адсорбированных молекулах и ослаблению химической связи в них вплоть
до полной диссоциации молекулы на атомы. Это значительно облегчает
взаимодействие адсорбированных молекул (атомов) реагирующих веществ
между собой. Чем больше поверхность, тем эффективнее катализатор. В
качестве гетерогенных катализаторов широко используются металлы (никель, платина, палладий, медь), кристаллические алюмосиликаты цеолиты, Al2O3, Al2(SO4)3 и др.
Обратимость химических реакций.
Химическое равновесие
Все химические реакции с точки зрения обратимости можно разделить на обратимые и необратимые. Обратимыми называются реакции,
протекающие одновременно в двух противоположных направлениях; необратимыми – реакции, протекающие практически до конца в одном
направлении.
Признаками практической необратимости реакций являются:
1) выделение газа: Na2CO3 + 2HCl = 2NaCl + H2O + CO2↑;
2) выпадение осадка: BaCl2 + Na2SO4 = BaSO4
NaCl;
3) образование мало диссоциирующего вещества (слабого электролита):
NaOH + HCl = NaCl + H2O;
4) выделение большого количества энергии:
H2 + Cl2 = 2HCl , + Q (реакция протекает со взрывом).
Большинство химических процессов являются обратимыми, т.е. при
одних и тех же условиях протекают прямая и обратная реакции. Состояние
обратимого процесса, при котором скорости прямой и обратной реакций
равны, называют химическим равновесием. Необходимым признаком
химического (термодинамического) равновесия системы является неизменность её состояния во времени при заданных внешних условиях. Химическое равновесие носит динамический характер: какое количество исходных веществ вступает в реакцию, такое же образуется в результате обратной реакции. Система стремится к минимуму свободной энергии, который наблюдается в состоянии равновесия. Химические равновесия чаще
всего изучаются при постоянном давлении и температуре.
Каждое подвижное химическое равновесие характеризуется своей
константой – константой равновесия Кр. Для системы аА+bВ ↔ сС+dD
скорость прямой реакции: υпр =k1·[А]а ·[ В]b ;
скорость обратной реакции: υобр =k2·[С]с ·[ D]d .
77
Применяя закон действующих масс к прямой и обратной реакциям
обратимого процесса получают выражение для расчета константы химического равновесия:
КР = k1/k2 = [С]с ·[ D]d / [А]а ·[ В]b ,
где Кр – константа химического равновесия (зависит от природы веществ,
температуры и давления); [A],[B],[M],[D] – равновесные молярные концентрации реагирующих веществ, моль/л; a, b, с, d — стехиометрические
коэффициенты реагирующих веществ.
В состоянии равновесия отношение произведения концентраций
продуктов реакции к произведению концентраций исходных веществ постоянно, причем концентрация каждого вещества взята в степени, равной числу молей вещества, участвующих в реакции.
Величина константы химического равновесия характеризует полноту
протекания обратимой реакции. Чем больше величина КР, тем глубже протекает реакция, т. е. тем больше образуется продуктов реакции.
Например, для системы 2NO + O2 ↔ 2NO2 константы химического
равновесия равна: КР = [NO2]2/ [NO]2[O2].
В случае гетерогенных реакций в выражение для константы равновесия входят только концентрации тех веществ, которые находятся в газовой
фазе. Например, для реакции СО2 + С ↔ 2CO константа равновесия имеет
вид: КР = [CO]2/[CO2].
Для реакций, протекающих между газообразными веществами, константа равновесия может быть выражена и через парциальные давления
реагирующих веществ. В общем случае константа равновесия КР должна
быть выражена через активности а реагирующих веществ. Для идеальных
растворов коэффициенты активности равны единице, активности будут
равны молярным концентрациям. Константа равновесия КР химической
реакции связана со стандартным изменением энергии Гиббса уравнением:
∆G0T = – RTlnKp= – 2,3RTlgKp.
Факторы, влияющие на константу равновесия: основным фактором, влияющим на константу равновесия, является природа реагирующих
веществ, прочность химических связей в соединениях. Другим фактором,
определяющим значение константы равновесия, служит температура. Зависимость константы равновесия от температуры выражается уравнением:
lnKP = –∆H/RT + ∆S/R
Константа равновесия не зависит от парциальных давлений и концентраций реагирующих веществ. Их изменение влияет только на смещение положения равновесия и степень превращения веществ. Степень превращения вещества – отношение количества вещества в равновесной смеси
к исходному количеству этого вещества.
Смещение химического равновесия
78
Состояние химического равновесия сохраняется сколь угодно долго
при неизменных условиях — температуре, давлении, концентрации. Любое изменение этих условий нарушает состояние химического равновесия,
так как нарушается равенство скоростей прямой и обратной реакций, но
через некоторый промежуток времени устанавливается новое состояние
равновесия. Переход системы из одного равновесного состояния в другое в
результате изменения условий называется смещением или сдвигом химического равновесия.
Общим принципом смещения положения равновесия в системе является принцип Ле-Шателье: если на систему, находящуюся в состоянии
равновесия, оказать внешнее воздействие, то равновесие сместится в
сторону той реакции, которая ослабляет это воздействие.
На смещение химического равновесия влияют следующие факторы:
а) изменение концентраций реагирующих веществ: при повышении
концентрации одного из веществ химическое равновесие смещается в сторону той реакции, которая уменьшает концентрацию этого вещества, т.е.
при увеличении концентрации исходных веществ химическое равновесие
смещается в сторону образования продуктов реакции и наоборот;
б) изменение давления (для реакций в газовой фазе): при увеличении
давления химическое равновесие в системе сместится в сторону той реакции, в результате которой уменьшается объем системы (для реакций, протекающих с участием газообразных веществ – в сторону той реакции, которая ведет к образованию меньших количеств (объемов) газообразных
веществ);
в) изменение температуры: при повышении температуры химическое равновесие смещается в сторону эндотермической реакции (– Q); при
понижении температуры – в сторону экзотермической реакции (+ Q).
Отметим, что рассмотренные закономерности справедливы для закрытых систем, в которых невозможен обмен веществом с окружающей
средой и реакции протекают при постоянном объеме.
Литература
1. Глинка Н. Л. Общая химия. – М.: Химия, 2006. – 720 с.
2. Глинка Н.Л. Задачи и упражнения по общей химии. – Л. Химия,
1985. – 264 с.
3. Карапетьянц М.Х., Дракин С.И. Общая и неорганическая химия. –
М.: Химия, 1993. – 558 с.
4. Коровин В.Н. Общая химия. – М.: Высшая школа, 2000. – 557 с.
5. Коровин Н.В., Масленникова Г.Н. и др. Курс общей химии. – М.:
Высшая школа, 1990. – 445 с.
79
6. Суворов А.В., Никольский А.Б. Общая химия. – СПб: Химиздат,
2001. – 512 с.
Примеры решения типовых задач
Задача 1. Во сколько раз изменится скорость реакции при понижении температуры от 400 до 20 0С, если температурный коэффициент равен
2,5?
Решение: если некоторая химическая реакция имеет γ = 2,5, а температура меняется от 400 до 20 0С, то изменение скорости реакции можно
рассчитать по формуле: υ2/ υ1 =γ ∆t/10 ; υ2/ υ1 =2,540-20/10 = 2,52,0=6,25. Скорость
реакции уменьшится в 6,25.
Задача 2. Во сколько раз изменится скорость прямой и обратной реакции в системе: 2SO2(г)+O2(г)↔2SO3(г),
а) если объем газовой смеси уменьшить в 4 раза;
б) если давление в системе увеличить в 2 раза?
Решение: запишем выражение закона действующих масс для скорости прямой и скорости обратной реакции:
υпр=kпр[SO2]2[O2];
υобр=kобр[SO3]2.
а) Уменьшение объема газовой смеси в 4 раза равносильно увеличению концентрации реагентов в 4 раза, поэтому после уменьшения объема
скорость прямой (υ'пр) и скорость обратной (υ'обр) реакций будут равны
υ'пр=kпр(4[SO2])2[4O2]=64kпр[SO2]2[O2];
υ'обр =kобр(4[SO3])2=16kобр[SO3]2;
отсюда: υ'пр/ υпр = 64 и υ'обр/ υобр= 16, то есть скорость прямой реакции увеличится в 64 раза, а скорость обратной реакции увеличится только в 16 раз.
б) Увеличение давления в 2 раза равносильно увеличению концентрации веществ в 2 раза:
υ'пр=kпр(2[SO2])2[2O2]=8kпр[SO2]2[O2];
υ'обр=kобр(2[SO3])2=4kобр[SO3]2;
отсюда, при увеличении давления в 2 раза скорость прямой реакции увеличится в 8 раз, а скорость обратной реакции увеличится в 4 раза.
Задача 3. Предскажите условия, при которых равновесие обратимой
реакции: CaCO3(т)↔CaO(т)+CO2(г) ; ∆H = – 177, 99 кДж, сместится в сторону
протекания прямой реакции.
Решение: согласно правилу Ле Шателье, если на систему, находящуюся в равновесии, производится какое-либо внешнее воздействие путем
изменения термодинамических параметров (температуры, давления, концентрации), то оно благоприятствует протеканию той из двух противоположных реакций, которая ослабляет это воздействие.
Рассмотрим влияние температуры на положение равновесия в данной системе: изменение энтальпии данной системы H>0, следовательно,
прямая реакция является эндотермической (– Q), поэтому повышение тем80
пературы будет способствовать ее протеканию, и равновесие реакции сместится в сторону разложения CaCO3.
Давление оказывает влияние на равновесие обратимой реакции в том
случае, когда в результате ее протекания изменяется число молей газообразных веществ. В левой части уравнения реакции газообразные вещества
отсутствуют, в правой части имеется 1 моль CO2. Изменение числа молей
газообразных веществ n в результате протекания прямой реакции равно:
n = n2– n1 = 1– 0 = 1, это означает, что прямая реакция протекает с увеличением объема газообразных веществ, поэтому ее протеканию будет благоприятствовать понижение давления в системе.
Влияние концентрации реагирующих веществ на скорость реакции
описывается законом действующих масс: υ пр = kпр ; υ обр = kобр[CO2].
Концентрации твердых веществ (CaCO3 и CaO) не входят в выражение для
расчета скоростей прямой и обратной реакции, так как они изменяются незначительно, и это изменение включено в соответствующую константу
скорости.
Таким образом, сместить равновесие в сторону протекания прямой
реакции можно только снижением скорости обратной реакции, то есть
уменьшением концентрации CO2 путем вывода его из сферы реакции. Катализатор в равной степени изменяет скорость обеих противоположных
реакций, поэтому не оказывает влияния на смещение равновесия.
Задача 4. В системе А(г)+2В(г) = С(г) равновесные концентрации равны: [А]равн= 0,06 моль/л; [В]равн.= 0,12 моль/л; [С]равн.= 0,216 моль/л. Найдите константу равновесия реакции и исходные концентрации веществ А и В.
Решение: константа равновесия данной реакции выражается уравнением: К= [С] / [А]·[ В]2 . Подставляя в него данные задачи, получаем:
К=0,216/0,06·(0,12)2 = 2,5.
Для нахождения исходных концентраций веществ А и В учтем, что,
согласно уравнению реакции, из 1 моля А и 2 молей В образуется 1 моль
С. Поскольку по условию задачи в каждом литре системы образовалось
0,216 моля вещества С, то при этом было израсходовано 0,216 моля вещества А и 0,216 · 2 = 0,432 моля вещества В.
Таким образом, искомые исходные концентрации равны:
[А0] = 0,06 + 0,216 = 0,276 моль/л; [В0] = 0,12 + 0,432 = 0,552 моль/л.
Задача 5. В закрытом сосуде смешано 8 молей SO2 и 4 моля О2. Реакция протекает при постоянной температуре. К моменту наступления
равновесия в реакцию вступает 80% первоначального количества SO2.
Определите давление газовой смеси при равновесии, если исходное давление составляло 300 кПа.
Решение: уравнение протекающей реакции: 2SO2(г) + О2(г)↔2SO3(г) .
Согласно условию задачи, в реакцию вступило 80%, т. е. из 6,4 моля SO2
осталось неизрасходованным 1,6 моля SO2. По уравнению реакции на каждые 2 моля SO2 расходуется 1 моль О2 , причем образуется 2 моля SO3.
81
Следовательно, на 6,4 моля SO2 в реакцию вступило 3,2 моля О2 и
образовалось 6,4 моля SO3; осталось неизрасходованным 4 – 3,2 = 0,8 моля
О2.
Таким образом, общее число молей газов составляло до протекания
реакции 8 + 4=12 молей, а после достижения равновесия 1,6 + 0,8 + 6,4 =
8,8 моля.
В закрытом сосуде при постоянной температуре давление газовой
смеси пропорционально общему количеству составляющих ее газов.
Следовательно, давление (Р) при равновесии определится из пропорции: 12 : 8,8 = 300 : Р, откуда Р = 8,8 ∙ 300/12 = 220 кПа.
Задача 6. При некоторой температуре константа диссоциации иодоводорода на простые вещества равна 6,25∙10-2. Какой процент HI диссоциирует при этой температуре?
Решение: уравнение реакции диссоциации HI: 2HI↔H2+ I2 . Обозначим начальную концентрацию HI через С (моль/л). Если к моменту
наступления равновесия из каждых С молей иодоводорода диссоциировало
х молей, то при этом, согласно уравнению реакции, образовалось 0,5х
моля H2 и 0,5х моля I2.
Таким образом, равновесные концентрации составляют:
[HI] = (С – х) моль/л;
[Н2] = [I 2] = 0,5х моль/л.
Подставим эти значения в выражение константы равновесия реакции:
К= [Н2] ·[I 2] / [HI]2 ; 6,25 ∙ 10-2 =0,5х ∙ 0,5х/(С – х)2.
Извлекая из обеих частей уравнения квадратный корень, получим:
0,25 = 0,5х/(С – х), откуда х =0, 333С, т.о. к моменту наступления равновесия диссоциировало 33,3% исходного количества иодоводорода.
Задачи и упражнения для самостоятельной работы
1.
2.
3.
4.
5.
Тема: Химическая кинетика
Вычислите, во сколько раз уменьшится скорость реакции, протекающей в газовой фазе, если понизить температуру от 120 до 80оС.
Температурный коэффициент скорости реакции 3.
Как изменится скорость реакции, протекающей в газовой фазе, при
повышении температуры на 60оС, если температурный коэффициент
скорости данной реакции 2?
При повышении температуры на 50°С скорость реакции возросла в
1200 раз. Вычислите температурный коэффициент скорости реакции.
Вычислите температурный коэффициент Вант-Гоффа, если константа скорости реакций при 120°С составляет 5,9 · 10-4, а при 170°С
равна 6,7 · 10-2.
При 150°С некоторая реакция заканчивается за 16 мин. Принимая
температурный коэффициент скорости реакции равным 2,5, рассчи-
82
тайте, через какое время закончится эта реакция, если проводить ее:
а) при 200 °С; б) при 80 °С.
6. Напишите кинетические уравнения реакций, протекающих по схеме:
А + 2В = С, если: а) А и В – газообразные вещества; б) А – газ, В –
твердое вещество.
7. Во сколько раз следует увеличить концентрацию вещества В2 в системе 2А2(г) + В2(г) =2А2В(г), чтобы, при уменьшении концентрации
вещества А в 4 раза, скорость прямой реакции не изменилась?
8. Окисление серы и ее диоксида протекает по уравнениям:
а) S(к) + О2 = SO2(к); б) 2SO2(г) + О2 = 2SO3(г). Как изменятся скорости
реакций, если объемы каждой из систем уменьшить в четыре раза?
9. Напишите кинетическое уравнение реакции: 2NO + Н2 = N2O + Н2О.
Укажите общий порядок реакции и порядок реакции по каждому
веществу.
10.Взаимодействие между оксидом углерода (II) и хлором протекает по
уравнению СО + С12 = СОС12. Концентрация оксида углерода (II)
равна 0,3 моль/дм3, хлора – 0,2 моль/дм3. Как изменится скорость
прямой реакции, если увеличить концентрацию оксида углерода (II)
до 1,2 моль/дм3, а концентрацию хлора – до 0,6 моль/дм3?
11.Реакция идет согласно уравнению: 2NO(г) + Cl2(г) = 2NOCl(г).
Концентрации исходных веществ до начала реакции составляли
[NO] = 0,8 моль/л, [Cl2] = 0,6 моль/л. Во сколько раз изменится
скорость реакции по сравнению с первоначальной в тот момент,
когда успеет прореагировать половина NO?
12.Как изменится скорость реакции 2NO(г) + O2(г) =2NO2(г) , если:
а) увеличить давление в системе в 3 раза;
б) уменьшить объем системы в 3 раза;
в) повысить концентрацию NO в 3 раза?
13.Две реакции протекают при 25°С с одинаковой скоростью. Температурный коэффициент скорости первой реакции равен 2,0, а второй –
2,5. Найдите отношение скоростей этих реакций при 95 °С.
14.Реакция 2Н2(г) + О2(г) =2Н2О(г) протекает с выделением теплоты.
Однако для того, чтобы реакция началась, исходную смесь газов
надо нагреть. Как это объяснить?
15.В два сосуда одной и той же вместимости введены: в первый – 1
моль газа А и 2 моля газа В, во второй – 2 моля газа А и 1 моль газа
В. Температура в обоих сосудах одинакова. Будет ли различаться
скорость реакции между газами А и В в этих сосудах, если скорость
реакции выражается уравнением:
а) υ1=k[А]; б) υ1=k [А]2[В] ?
Тема: Обратимость химических реакций.
Химическое равновесие
83
1. Напишите выражение константы равновесия системы:
4 H2O(пар) + 3 Fe ↔ Fe3O4 +4H2 , ∆H0 < 0.
Какие изменения концентраций, давления могут вызвать смещение
равновесия этой системы вправо?
2. Как следует изменить температуру, концентрацию и давление компонентов в данной равновесной системе, чтобы повысить выход
хлора: 4HCl + O2 ↔ 2Cl2 + 2H2O , ∆H0 < 0 .
3. В каком направлении сместится равновесие реакции:
А2(г) + В2(г)↔ 2АВ(г) , если давление в системе увеличить в 2 раза и
одновременно повысить температуру на 10 градусов? Температурные коэффициенты скорости прямой и обратной реакций равны соответственно 2 и 3. Каков знак ∆H0 этой реакции?
4. В каком направлении сместится равновесие реакции:
1) 2СО(г) + О2(г) ↔ 2СО2 (г) , ∆H0 = – 566 кДж;
2) N2(г) + О2(г) ↔ 2NO(г) ,
∆H0 = 180 кДж;
а) при понижении температуры? б) при повышении давления?
5. Укажите, какими изменениями концентраций реагирующих веществ можно сместить вправо равновесие реакции:
СО2(г) +С (графит) ↔2СО(г) .
6. Укажите реакцию, в которой изменение давления не влияет на положение равновесия:
а) SO2(г) + Cl2(г) ↔ SO2Cl2(г) ;
б) 2NO(г) + О2(г) ↔ 2NO2(г) ;
в) SO2(г) + Н2О(ж)↔H2SO3(p-p) ;
г) 3Fe2O3(к) + СО(г) ↔2Fe3O4(к+ СО2(г) .
7. Укажите все возможные способы смещения равновесия в системе:
2SO2(г) + О2(г) ↔2SO3(г), ∆H0 < 0, в сторону образования оксида cepы
(VI).
8. Вычислите процент разложения молекулярного хлора на атомы, если
константа равновесия составляет 4,2 · 10-4, а исходная концентрация
хлора 0,04 моль/л.
9. Константа равновесия реакции СО(г) + Сl2(г) = СОСl2(г) при некоторой
температуре равна 6. Найдите равновесные концентрации всех веществ, если их исходные концентрации равны [СО] =2 моль/л,
[Сl2]=0,25 моль/л, [СOCl2] = 3 моль/л.
10.Константа равновесия реакции СО(г) + Сl2(г) = СOCl2 (г) при некоторой
температуре равна 6. В каком направлении будет протекать реакция
при следующих исходных концентрациях веществ: 2 моль СО, 2
моль Cl2, 8 моль СOCl2? Рассчитайте равновесные концентрации
всех веществ.
11.Вычислите константу равновесия системы: СО(г) + Сl2 (г) = СOCl2 (г),
если [СО]исх = [Cl2]исх = 4 моль/л, [СОСl2]исх = 0, а [COCl2] = 3,27
моль/л.
84
12.Определите равновесную концентрацию водорода в системе H2S →
H2 + S, если [H2S]=0,05 моль/л, а Кравн. = 9· 10-2.
13.Сколько молей вещества АВ образуется в системе А2 + В2 = 2АВ из 1
моля В2 и 2 молей А2, если Кравн. = 4 · 10-4.
14.Реакция СО(г) + Сl2(г) ↔ СОСl2(г) протекает в закрытом сосуде при
постоянной температуре, исходные вещества взяты в эквивалентных
количествах. К моменту наступления равновесия остается 50%
начального количества СО. Определите давление равновесной
газовой смеси, если первоначальное давление равнялось 100 кПа
(750 мм рт. ст.).
15.Константа равновесия реакции А + В ↔ С + D равна единице.
Начальная концентрация [А0]= 0,02 моль/л. Сколько процентов
вещества А подвергается превращению, если начальные
концентрации [В0] равны 0,02; 0,1 и 0,2 моль/л?
Контрольная работа №3
по разделам «Термодинамика» и «Химическая кинетика»
Задание 1. Рассчитайте тепловой эффект химической реакции по теплотам
образования участников реакции (табл.9).
Задание 2. Рассчитайте H0298 и S0298 реакции (табл.9).
Таблица 9
вариант
1
2
3
4
5
6
7
6
7
8
9
10
11
12
13
Задание 1
Задание 2
CaCO3(т)+ 2C(графит)= CaO(т)+ CO(г)
Fe2O3+2Al(к)=2Fe(к) + Al2O3
SiH4(г)+4N2O(г)=SiO2(к)+4N2(г)+O2(г)
CaO(к)+H2O(ж)=Ca(OH)2(к)
4HCl(г)+O2(г) =2H2O(г)+2Cl2(г)
N2(г)+3H2(г)=2NH3(г)
2CO(г)+O2(г)=2CO2(г)
SiCl4(г)+2H2O(г)=SiO2(к)+4HCl(г)
SiH4(г)=Si(к)+2H2(г)
NH3(г)+HCl(г)= NH4Cl(к)
CH4(г)+2O2(г)=CO2(г)+2H2O(г)
SiHCl3(г)+H2(г) = 3HCl(г)+Si(к)
CS2(тв)+3H2O(ж)=2H2S(г)+CO2(г)+
H2O(ж)
2B(кр)+N2(г)=2BN(тв)
CaCO3(т)=CaO(т)+CO2(г)
85
Fe2O3 (к)+3H2 (г) = 2Fe (к)+3H2O (г)
CaO(к) + H2O (ж) = Ca(OH)2 (к)
C(графит) + 0,5O2 (г) = CO (г)
BaO(к) + CO2 (г) = BaCO3 (к)
3Fe2O3(к)+H2 (г)=2Fe3O4(к)+ H2O(г)
8Al(к)+3Fe3O4(к)=9Fe(к)+4Al2O3(к)
SrO(к) + CO2 (г) = SrCO3 (к)
CO(г) + 2H2 (г) = СH3OH (ж)
C2H4(г)+3O2(г)=2CO2(г) +2H2O(ж)
Sn(к) + 0,5O2(г) = SnO(к)
FeO(к) + H2(г) =Fe(к) + H2O(г)
Fe(к) + H2O(г) = FeO(к) + H2(г)
3Fe2O3(к)+H2(г)=2Fe3O4(к) + H2O(г)
C2H4(г)+3O2(г)=2CO2(г)+2H2O(ж)
S(к) + 1,5O2(г) = SO3 (г)
14
15
16
17
18
19
20
CH4(г)+2H2O(г)=CO2(г)+4H2(г)
SiH4(г)+2CO2(г)=SiO2(к)+CH4(г)
Fe2O3+3CO(г)=2 Fe(к) +3CO2(г)
2PH3(г)+ 4O2(г)= P2O5(к) + 3H2O(ж)
4NH3(г)+ 3O2(г )= 6H2O(г) + 2N2(г)
CaO(т)+ H2O(ж)=Ca(OH)2 (т)
C(графит)+2N2O(г)= CO2(г)+ 2N2(г)
CO(г) + 3H2(г) = СH4(г) + H2O(г)
FeO(к) + CO(г) = Fe (к) + CO2 (г)
Fe2O3(к)+ CO (г) = 2FeO(к) + CO2 (г)
FeO(к) + C(графит) = Fe(к) + CO(г)
CaCO3 (к) = CaO(к) + CO2(г)
Pb(к) + 0,5O2 (г) = PbO(к)
C(графит) + 0,5O2 (г) = CO(г)
Задание 3.
Вычислите стандартное изменение изобарно-изотермического потенциала при стандартной температуре по изменению энтальпии и энтропии реакции. Определите возможность самопроизвольного протекания химической реакции (табл. 10).
Таблица 10
0
№
H 298,
S0298,
задаУравнение реакций
кДж
Дж/К
чи
1 2B2O3(к)+3Si(к)=3SiO2(к)+4B(к)
-49,90
-18,11
2
2P2O5(к)+5Si(к)=5SiO2(к)+4P(к)
1895,50
-269,75
3
2PCl3(г)+3H2(г)=2P(к)+6HCl(г)
0,2
193,7
4
2BBr3(г)+3H2(г)=2B(к)+6HBr(г)
17,66
161,19
5
SiHCl3(г)+H2(г)=Si(к)+3HCl(г)
67,03
134,91
6
SiCl4(г)+2H2(г)=Si(к)+4HCl(г)
-840,31
173,55
7
SiBr4(г)+2H2(г)=Si(к)+4HBr(г)
146,41
-81,89
6
2PCl3(г)+5/2O2(г)=P2O5(к)+3Cl2(г)
-2542,00
-184,98
7
B2H6(г)+3O2(г)=B2O3(к)+3H2O(г)
-2184,24
-226,078
8
GeCl4(г)+2H2(г)=Ge(к)+4HCl(г)
20,19
-282,11
9
4BN(к)+3O2(г)=2B2O3(к)+2N2(г)
-1517,6
-179,59
10
Si(к)+2H2O(г)=SiO2(к)+H2(г)
-375,62
-223,60
11
3SiBr4(г)+2N2(г)=Si3N4(к)+6Br2(г)
437,67
52,2
12
SiH4(г)+2O2(г)=SiO2(к)+2H2O(г)
-1229,74
-195
13
SiH4(г)+2CO2(г)=SiO2(к)+CH4(г)
-33,89
-403,43
14
3SiH4(г)+4NH3(г)=Si3N4(к)+12H2(г)
-94,36
279,06
15
AlF3(к)+3/2H2(г)=Al(к)+3HF(г)
682,17
286,46
16
3SiCl4(г)+4NH3(г)=Si3N4(к)+12HCl(г)
-2955,01
571,68
86
17
SiH4(г)+2O2(г)=SiO2(к)+2H2O(г)
-1229,74
-195,00
18
SiCl4(г)+Si(к)=2SiCl2(г)
-545,51
-215,95
19
AsH3(г)+3/2Cl2(г)=As(к)+3HCl(г)
-412,17
38,08
20
2BBr3(г)+3/2O2(г)=B2O5(к)+3Br2(г)
-538,96
53,01
Задание 4.
Как изменится скорость реакции:
а) при изменении объема газовой смеси;
б) при изменении давления в системе (табл. 11).
№ задачи
1
Таблица 11
Изменение
Изменение
объема га- давления в
зовой смеси системе
увеличить в уменьшить в
2 раза
4 раза
Уравнение реакции
2SO2(г)+O2(г)=2SO3(г)
2
2Аl(к) + 3Сl2(г) = 2АlСl3(к)
увеличить в
2 раза
уменьшить в
3 раза
3
Н2(г) + J2(г) = 2НJ (г)
увеличить в
3 раза
уменьшить в
2 раза
4
2H2(г) + O2(г) = 2H2O(г)
уменьшить
в 4 раза
увеличить в
2 раза
5
H2(г) + Cl2(г) = 2HCl(г)
уменьшить
в 3 раза
уменьшить в
2 раза
увеличить
в 10 раз
уменьшить
в 4 раза
уменьшить
в 4 раза
увеличить
в 10 раз
увеличить
в 2 раза
уменьшить
в 4 раза
уменьшить
в 4 раза
увеличить
в 10 раз
увеличить в
4 раза
уменьшить в
4 раза
увеличить в
2 раза
увеличить
в 2 раза
уменьшить
в 4 раза
уменьшить в
5 раз
6
7
8
9
10
11
12
2NO(г) +O2(г) (г) =2NO2
4BN(к)+3O2(г)=2B2O3(к)+2N2(г)
SiH4(г)+2O2(г)=SiO2(к)+2H2O(г)
H2(г)+Cl2(г)=2HCl(г)
CaCO3(т)=CaO(т)+CO2(г)
CH4(г)+2H2O(г)=CO2(г)+4H2(г)
S(г) + O2(г) = 2SO2(г)
87
13
2NO2(г) =N2O4(г)
14
3H2(г) +2N2(г) =2NH3(г)
15
CaCO3(т)=CaO(т)+CO2(г)
16
2SO2(г) + O2(г) = 2SO3(г)
17
CO2(г) + C2(г) = 2CO(г)
18
H2(г) + Br2(г) = 2H2Br(г)
19
SiCl4(г)+ H2(г) =SiHCl3(г) + HCl(г)
20
2NO(г) +O2(г) =2NO2(г)
уменьшить
в 5 раз
уменьшить в
3 раза
увеличить в
4 раза
уменьшить
в 4 раза
увеличить
в 3 раза
уменьшить
в 3 раза
увеличить
в 10 раз
уменьшить
в 3 раза
уменьшить
в 4 раза
уменьшить в
3 раза
увеличить
в 2 раза
уменьшить
в 2 раза
увеличить
в 4 раза
уменьшить
в 5 раз
увеличить
в 3 раза
увеличить
в 5 раз
Задание 5.
Таблица 12
№ варианта
1
2
3
4
5
Задание 5
Константа скорости реакции разложения N2О, протекающей по
уравнению 2N2О=2N2+O2, равна 5∙10-4. Начальная концентрация
N2О = 6,0 моль/л. Вычислите начальную скорость реакции и ее
скорость, когда разложится 50% N2О.
Напишите выражение для константы равновесия гетерогенной
системы СО2 + С=2СО. Как изменится скорость прямой
реакции образования СО, если концентрацию СО2 уменьшить в
четыре раза? Как следует изменить давление, чтобы повысить
выход СО?
Напишите выражение для константы равновесия гетерогенной
системы С+Н2О(г)=СО+ Н2. Как следует изменить концентрацию
и давление, чтобы сместить равновесие в сторону обратной
реакции образования водяных паров?
Равновесие гомогенной системы 4НСl(г) + О2 = 2Н2О(г) + 2Сl2(г)
установилось при следующих концентрациях реагирующих
веществ: [Н2О]р=0,14 моль/л; [Сl2]р=0,14 моль/л; [НСl]р=0,20
моль/л; [О2]р=0,32 моль/л. Вычислите исходные концентрации
хлороводорода и кислорода.
Напишите выражение для константы равновесия гомогенной
системы: N2+3Н2=2NH3. Как изменится скорость прямой
88
6
7
8
9
10
11
12
13
14
15
реакции образования аммиака, если увеличить концентрацию
водорода в три раза?
Реакция идет по уравнению: N2 + О2 = 2NO. Концентрации
исходных веществ до начала реакции были: [N2]=0,049 моль/л;
[О2] = 0,01 моль/л. Вычислите концентрацию этих веществ в
момент, когда [NO] =0,005 моль/л.
Реакция идет по уравнению: N2 + 3H2 = 2NH3. Концентрации
участвующих в ней веществ были: [N2]=0,80 моль/л; [Н2] =1,5
моль/л; [NH3]=0,10 моль/л. Вычислите концентрацию водорода
и аммиака, когда [N2] =0,5 моль/л.
Реакция идет по уравнению: Н2 + I2 = 2НI. Константа скорости
этой реакции при некоторой температуре равна 0,16. Исходные
концентрации реагирующих веществ: [Н2] =0,04 моль/л; [I2]
=0,05 моль/л. Вычислите начальную скорость реакции и ее
скорость, когда [Н2] =0,03 моль/л.
Константа равновесия реакции СО(г)+Сl2(г)=СOCl2(г) при
некоторой температуре равна 4. В каком направлении будет
протекать реакция при следующих исходных концентрациях
веществ: 1 моль СО, 1 моль Cl2, 4 моль СOCl2? Рассчитайте
равновесные концентрации.
Напишите выражение константы равновесия системы, предварительно расставив коэффициенты: NH3+O2=NO+H2O(nap)
Каким образом можно сместить равновесие этой системы
вправо?
В
гомогенной
системе
СО+Сl2=СОСl2
равновесные
концентрации реагирующих веществ: [СО]=0,2 моль/л; [Cl2]=0,3
моль/л, [COCl2]=1,2 моль/л. Вычислите константу равновесия
системы и исходные концентрации Сl2 и СО.
В гомогенной системе А + 2В = С равновесные концентрации
реагирующих газов: [А] =0,06 моль/л; [В] =0,12 моль/л; [С]
=0,216 моль/л. Вычислите константу равновесия системы и
исходные концентрации веществ А и В.
В гомогенной газовой системе А + В = С + D равновесие
установилось при концентрациях: [В] =0,05 моль/л и [С] =0,02
моль/л. Константа равновесия системы равна 0,04. Вычислите
исходные концентрации веществ А и В.
Вычислите константу равновесия для гомогенной системы:
СО(г) + Н2О(г) = СО2(г) + Н2(г) , если равновесные концентрации
реагирующих веществ: [СО]р=0,004 моль/л; [Н2О]р=0,064
моль/л; [СО2]р=0,016 моль/л; [Н2]р=0,016 моль/л. Чему равны
исходные концентрации Н2О и СО?
Константа равновесия гомогенной системы
СО(г) + Н2О(г) = СО2 + Н2(г) при некоторой температуре равна 1.
89
16
17
18
19
20
Вычислите равновесные концентрации всех реагирующих
веществ, если исходные концентрации: [СО]исх=0,10 моль/л;
[H2O]исх =0,40 моль/л.
Константа равновесия гомогенной системы N2 + 3H2 = 2NH3
при некоторой температуре равна 0,1. Равновесные
концентрации водорода и аммиака соответственно равны 0,2 и
0,08
моль/л.
Вычислите
равновесную
и
исходную
концентрацию азота.
При некоторой температуре равновесие гомогенной системы
2NO + О2 = 2NO2 установилось при следующих концентрациях
реагирующих веществ: [NO]р =0,2 моль/л; [О2]р =0,1 моль/л;
[NO2]р =0,1 моль/л. Вычислите константу равновесия и
исходную концентрацию NO и О2.
Почему при изменении давления смещается равновесие
системы N2 + 3Н2 = 2NH3 и не смещается равновесие системы
N2 + О2 = 2NO? Ответ мотивируйте на основании расчета
скорости прямой и обратной реакции в этих системах до и
после изменения давления. Напишите выражения для констант
равновесия каждой из данных систем.
Исходные концентрации [NO]0 и [Сl2]0 в гомогенной системе
2NO + Сl2 = 2NOCl составляют соответственно 0,5 и 0,2 моль/л.
Вычислите константу равновесия, если к моменту наступления
равновесия прореагировало 20% NO.
Напишите выражение константы равновесия системы:
2NO + О2 = 2NO2. При некоторой температуре равновесие
данной гомогенной системы установилось при следующих
концентрациях реагирующих веществ: [NO]р=0,4 моль/л;
[О2]р=0,2 моль/л; [NO2]р=0,2моль/л. Вычислите константу
равновесия и исходную концентрацию NO и О2.
90
ИТОГОВЫЙ ТЕСТ
№ задания
1.
2.
3.
4.
5.
6.
Вариант 1
Условие задачи
Выберите правильное утверждение: у изотопов элемента одинаково:
а) число нейтронов;
б) число протонов;
в) число электронов;
г) массовое число;
д) атомная масса;
е) порядковый номер.
1) б, в, е; 2) а, б; 3) в, г, д; 4) а, в, е.
Электронная конфигурация 1s22s22p63s23p3 соответствует атому:
1) алюминия; 2) азота; 3) фосфора; 4) серы.
Для энергетического уровня с главным квантовым числом п соотнесите:
характеристика:
формула для расчета:
1) число подуровней;
а) 2п2
2) число орбиталей;
б) п
3) максимальное число электронов;
в) п2
Каково строение внешнего энергетического уровня p-элементов?
1) ns1-2;
2) np1-6;
3) ns2 np1-6;
4) (n-1)dp1-10ns2 np1-6.
Назовите элемент, атом которого имеет в основном состоянии 3
неспаренных электрона на 4р-подуровне:
1)V; 2) As; 3) Ge; 4) Nb.
Энергия отрыва электрона от атома элемента с образованием ка91
7.
8.
9.
10.
11.
12.
13.
тиона — это:
1) энергия ионизации;
2) энергия сродства к электрону;
3) электроотрицательность абсолютная;
4) электроотрицательность относительная.
С увеличением порядкового номера элемента периодически повторяются:
1) заряд ядра атома;
2) строение внешних электронных уровней;
3) химические свойства элементов;
4) общее число электронов.
Какую химическую связь называют ковалентной:
1) связь между атомами, осуществляемая общей для этих атомов
парой электронов;
2) связь за счет взаимодействия противоположно заряженных
ионов;
3) связь за счет совокупности электронов, свободно перемещающихся между положительно заряженными ионами в кристалле;
4) связь за счет электростатического взаимодействия молекулярных диполей.
Выберите ряд соединений, в которых есть только ковалентная
полярная связь:
1) N2, HC1, NaCl;
2) СО, СН4, КМnО4;
3) NaHCO3, KNO3, HBr;
4) НС1, NH3, SiH4.
Выберите неправильное утверждение:
1) ионная связь образуется только в соединениях типичных металлов с типичными неметаллами;
2) степень ионности связи тем выше, чем меньше разность ОЭО
между соединенными атомами;
3) ассоциация — характерная особенность соединений с ионной
связью — способствует образованию ионной кристаллической
решетки в твердом состоянии;
4) для ионных соединений характерны высоки температуры
плавления и кипенияю.
Для какого вещества характерно образование атомной решетки?
1) нитрат аммония; 2) оксид углерода (IV); 3) иод; 4) алмаз.
Какие элементы: С, Si, N, Se, P, V образуют летучие водородные
соединения с формулой ЭН3?
1) N; 2) С, Р; 3) N, Р; 4) Se, Si, С; 5) все.
Как изменяются кислотно-основные свойства в ряду:
МnО → МnО2 → МnО3 → Мn2О7?
92
14.
15.
16.
17.
18.
19.
20.
1) усиливаются основные свойства;
2) не изменяются;
3) усиливаются кислотные свойства;
4) ослабевают кислотные свойства.
Соли двухвалентного железа в водных растворах:
1) обладают окислительно-восстановительной двойственностью;
2) окислители;
3) окислительно-восстановительными свойствами не обладают;
4) восстановители.
С какими веществами может взаимодействовать Са при комнатной температуре: а) Н2О; б) HС1(р-р); в) О2; г) С12; д) Вr2?
1) а, в, г;
2) а, б, в;
3) в, г, д;
4) со всеми.
Элементы II-А группы являются:
1) сильными окислителями;
3) сильными восстановителями;
2) слабыми восстановителями;
4) слабыми окислителями.
Определите роль водорода в реакции: Н2 + КМnО4 +H2SO4→ ..
1) восстановитель;
2) реакция не идет;
3) окислитель;
4) реакция не относится к ОВР.
В соответствии с законом Гесса тепловой эффект реакции равен:
1) сумме теплот образования продуктов реакции;
2) сумме теплот образования исходных веществ;
3) сумме теплот образования продуктов реакции за вычетом
суммы теплот образования исходных веществ;
4) сумме теплот образования исходных веществ за вычетом
суммы теплот образования продуктов реакции.
Какие из следующих утверждений верны для реакций, протекающих в стандартных условиях?
1) эндотермические реакции не могут протекать самопроизвольно;
2) эндотермические реакции могут протекать при достаточно
низких температурах;
3) эндотермические реакции могут протекать при высоких температурах, если ∆S>0;
4) эндотермические реакции могут протекать при высоких температурах, если ∆S<0.
Тепловой эффект реакции SO2(г) + 2H2S(г)= 3S(тв) + 2Н2О(ж) равен
93
21.
22.
23.
24.
25.
№ за-
-234,50 кДж.
Определите стандартную теплоту (энтальпию) образования
H2S(г), если ∆H°298(SO2) =-296,9 кДж/моль; ∆Н0298(Н2Ож) = -285,8
кДж/моль.
1) +20,1 кДж; 2) - 20,1 кДж; 3) + 66,4 кДж; 4) - 66,4 кДж.
Укажите гомогенную систему:
1) 3Fe + 4Н2О(пар) = Fe3O4 + 4Н2(г) ;
2) С(тв) + О2(г) = СО2(г);
3) СО(г) + 2Н2(г) =СН3ОН(г);
4) 2Сr + 3С12(г) =2СrС13.
При действии ферментов:
а) снижается энергия активации;
б) увеличивается скорость прямой и обратной реакций;
в) изменяется тепловой эффект реакций;
г) увеличивается скорость только прямой реакции.
1) а, б, в;
2) а, б;
3) б, в;
4) а, г.
Как изменится скорость реакции СО(г) + С12 (г) = СОС12(г), если
давление в системе увеличить в 4 раза?
1) увеличится в 8 раз;
2) увеличится в 16 раз;
3) уменьшится в 8 раз;
4) уменьшится в 16 раз.
2Н2(г)+О2(г) ↔2Н2О(ж): Δ Н=-120,89 кДж. Соотнесите:
воздействие:
смещение равновесия:
1) увеличили давление;
а) сместится вправо;
2) повысили температуру;
б) сместится влево;
3) понизили температуру;
в) не сместится.
4) уменьшили давление.
Рассчитайте изменение энергии Гиббса, энтальпии и энтропии
для стандартного состояния реакции горения метана:
СН4 (г) + 2О2(г) → СО2(г) + 2Н2О(г).
Какой фактор, энтальпийный или энтропийный, является движущей силой этого процесса?
1) ∆H0 = -803,3 кДж/моль; ∆S0 = -4 Дж/моль·К; ∆G 0 = = -801,4
кДж/моль. Движущая сила процесса — изменение энтальпии;
2) ∆H0 = +803,3 кДж/моль; ∆S0 = +4 Дж/моль·К; ∆G = = +801,4
кДж/моль. Движущая сила процесса — изменение энтропии.
Вариант 2
Условие задачи
94
дания
1.
2.
3.
4.
5.
6.
7.
8.
Принцип наименьшей энергии справедлив для атома:
а) в основном состоянии;
б) в возбужденном состоянии;
в) в основном и возбужденном состоянии;
г) имеющего минимальную энергию.
1) а; 2) а, г; 3) б; 4) в, г; 5) в.
Атом азота имеет электронную конфигурацию:
1) 1s22s22p2; 2) 1s22s22p5; 3) 1s22s22p4; 4) 2s22p3.
Четыре квантовых числа полностью описывают состояние
электрона в атоме. Соотнесите:
квантовое число: 1) п; 2) l; 3) ml ; 4) ms ;
физический смысл:
а) характеризует собственный момент количества движения
электрона;
б) определяет размеры электронного облака;
в) определяет форму электронного облака;
г) определяет ориентацию в пространстве электронного облака.
Какова электронная формула валентных электронов d-элементов?
1) nd1-10;
2) (n-1)d1-10ns1-2;
3) np1-6 nd1-10;
4) ns2 nd1-10.
Назовите элемент 4-го периода с максимальным числом неспаренных электронов.
1) Сr; 2) Мn; 3)Fe; 4) As.
Энергия присоединения электрона атомом элемента с образованием аниона — это:
1) энергия ионизации;
2) энергия сродства к электрону;
3) электроотрицательность абсолютная;
4) электроотрицательность относительная.
Выберите правильную последовательность уменьшения атомных
радиусов S, C1 и Аr:
1) S > С1 > Аr;
2) S > Аr > С1;
3) Аr > Cl > S;
4) Аr > Cl > S.
Какую химическую связь называют ионной:
1) связь между атомами, осуществляемая общей для этих атомов
парой электронов;
2) связь за счет взаимодействия противоположно заряженных
ионов;
95
9.
10.
11.
12.
13.
14.
15.
3) связь за счет совокупности электронов, свободно перемещающихся между положительно заряженными ионами в кристалле;
4) связь за счет электростатического взаимодействия молекулярных диполей.
Какая химическая связь возникает между атомами элементов с
порядковыми номерами 8 и 16?
1) ионная;
2) ковалентная неполярная;
3) ковалентная полярная;
4) водородная.
Число общих электронных пар между соединяемыми атомами –
это:
1) насыщаемость связи;
2) сопряжение связи;
3) кратность связи;
4) направленность связи.
Для большинства солей характерна:
1) ионная решетка;
2) атомная ковалентная решетка;
3) молекулярная решетка;
4) любая решетка из названных;
Выберите элементы, высший оксид которых имеет формулу ЭО3:
Al, Se, Sn, Cr, Cl, S, As.
1) Se, Cr, S;
2) Se, Sn, As;
3) Cr, S, As;
4) Al, Cl, S.
В ряду оксидов и гидроксидов As (III) – Sb (III) –Bi (III):
1) кислотные свойства увеличиваются;
2) кислотные свойства уменьшаются;
3) кислотные свойства не изменяются;
4) основные свойства уменьшаются.
Веществом А в схеме реакции FeCl3 + А → Fe(OH)3 является:
1) H2SO4;
2) Н2О;
3) NaOH;
4) SO3.
Элементы Са, Sr, Ва взаимодействуют с рядом соединений при
комнатной температуре:
1) Н2О, Hal2, NaOH;
2) N2, H2O, разб. кислоты;
3) F2, O2, Hal2;
4) N2, H2, H2O.
96
16.
17.
18.
19.
20.
21.
22.
23.
Какой из ионов Cr3+, Fe2+, Ni2+, Pb2+ проявляет более сильные
окислительные свойства?
1) Сr3+;
2) Fe2+;
3) Pb2+;
4) Ni2+.
Определите роль пероксида водорода в реакции:
Н2О2 + КМnО4+ H2SO4→…
1) восстановитель;
2) реакция не идет;
3) окислитель;
4) реакция не относится к ОВР.
Соотнесите:
термодинамическая характеристика реакции:
a) ∆G>0; б) ∆G=0; в) ∆G <O.
возможность самопроизвольного протекания реакции:
1) реакция возможна; 2) реакция невозможна; 3) в системе
установилось равновесие.
При каких значениях по знаку ∆H и ∆S в системе возможны
только экзотермические процессы?
1) ∆AH<0, ∆S>0;
2) ∆H<0, ∆S<0;
3) ∆H>0, ∆S>0;
4) ∆H=0, ∆S=0.
Окисление аммиака протекает по уравнению:
4NH3(г) + 3О2(г) = 2N2(г) + 6Н2О(ж); ∆Н° =-1528кДж.
Определите стандартную теплоту образования NH3(г), если
∆Н0298(Н2Ож) = -285,8 кДж/моль.
1) +220,4 кДж; 2) - 220,4 кДж; 3) + 46,7 кДж; 4) - 46,7 кДж.
Чем объяснить повышение скорости реакции при введении в систему катализатора?
а) уменьшением энергии активации;
б) увеличением средней кинетической энергии молекул;
в) возрастанием числа столкновений;
г) ростом числа активных молекул.
1) а, г;
2) а, в, г;
3) а, б, в, г;
4) б, в, г.
Какой реакции соответствует выражение скорости реакции
υ =k[O2]:
1) С + О2 = СО2; 2) N2 + O2 = 2NO; 3) 2Н2 + О2 = 2Н2О.
В реакции 2А(г) + В(г) = С + D, концентрацию вещества А увели97
24.
25.
№ задания
1.
2.
3.
4.
чили в 2 раза, а вещества В - в 3 раза. Как изменилась скорость
реакции?
1) увеличится в 6 раз;
2) увеличится в 12 раз;
3) уменьшится в 6 раз;
4) уменьшится в 12 раз.
С(тв) + Н2О(пар) ↔ СО + Н2; Δ Н=129,89 кДж. Соотнесите:
воздействие:
смещение равновесия:
1) увеличили давление;
а) сместится вправо;
2) повысили температуру;
б) сместится влево;
3) понизили температуру;
в) не сместится.
4) уменьшили давление.
Константа равновесия реакции N2O4 (г) ↔ 2NO2 (г) при 25 °С равна
0,1. В каком направлении будет идти реакция при следующих
концентрациях веществ: С(NO2) = С (N2O4) = 0,02 моль/л?
1) ∆G = -3,98 кДж, в прямом;
2) ∆G = 0, равновесие;
3) ∆G = -3,98 кДж, в обратном;
4) ∆G = +3,98 кДж, в прямом.
Вариант 3
Условие задачи
Предложенная формулировка:
электроны в невозбужденном атоме распределяются по энергетическим уровням и подуровням так, чтобы их суммарная энергия
была минимальна — это:
1) принцип Паули;
2) правило Гунда;
3) принцип минимума энергии;
4) постулат Н. Бора.
Атом серы имеет электронную конфигурацию:
1) 1s22s22p4;
2) 1s22s22p5;
3) 2s22p4;
4) 3s23p4.
Соотнесите:
квантовое число:
обозначение:
а) магнитное квантовое число;
1) п
б) главное квантовое число;
2) l
в) спиновое квантовое число;
3) ml
г) орбитальное квантовое число
4) ms
Какова формула валентных электронов атома Fe?
98
5.
6.
7.
8.
9.
10.
11.
12.
13.
14.
1) ... 4s23d6;
2) ... 4s2;
3) ... 3d6;
4) ... 4s03d8.
Сколько неспаренных электронов имеет ион Со3+?
1) 3; 2) 4; 3) 5; 4) 6.
Чем больше энергия ионизации атома, тем:
1) металлические свойства выражены сильнее;
2) металлические свойства выражены слабее;
3) неметаллические свойства выражены слабее;
4) атомный радиус меньше.
В периоде с увеличением заряда ядра атома у химических элементов наблюдается:
1) увеличение атомного радиуса;
2) возрастание электроотрицательности;
3) уменьшение атомного радиуса;
4) возрастание окислительная активность.
Ковалентные связи обычно образуются:
1) между атомами двух неметаллов;
2) между атомами типичного металла и типичного неметалла;
3) между атомами в молекулах только газообразных веществ;
4) только в молекулах простых веществ.
Ковалентная связь, образующаяся по донорно-акцепторному механизму, присутствует в соединении:
1) NaCl; 2)СС14; 3) Н2О; 4) [Ag(NH3)2]Cl.
Способность электронной плотности химической связи смещаться под действием внешнего электрического поля или других воздействий — это:
1) полярность связи;
2) направленность связи;
3) поляризуемость связи;
4) гибридизация.
Для какого вещества характерно образование ионной решетки?
1) иодид цезия; 2) глюкоза; 3) аммиак; 4) оксид кремния (IV).
Выберите элемент, высший гидроксид которого является самой
сильной кислотой:
1) С1; 2) F; 3) S; 4) Ва; 5) I.
В ряду NaOH, Mg(ОН) 2, А1(ОН)3:
1) основные свойства увеличиваются;
2) основные свойства ослабевают;
3) растворимость увеличивается;
4) сила электролита увеличивается.
С какими веществами взаимодействует Zn(OH)2:
99
15.
16.
17.
18.
19.
20.
21.
22.
а) НС1; б) NaOH; в) S; г) NH3; д) Na.
1) со всеми; 2) б, в, г; 3) а, б; 4) в, г, д; 5) а, б, г.
Назовите реагенты А, В, С в следующей схеме превращений:
СаС12→ Са→ Са(NO3)2 → СаС2О4:
1) электролиз, HNO3(p), СО2;
2) Zn, HNO3(к), Na2C2O4;
3) электролиз, HNO3(к), Na2C2O4;
4) Na, HNO3(p), CO2.
Как меняется восстановительная активность в ряду Ве – Ва?
1) увеличивается; 2) уменьшается; 3) не меняется.
Какие продукты реакции образуются при взаимодействии NaNO2,
KI в присутствии H2SO4?
1) NaNO3, NaI, K2SO4, H2O;
2) NaNO3, I2, K2SO4, H2O;
3) NaI, KNO2, K2SO4, H2O;
4) NaNO3, K2SO4, NaI, H2O.
Для обратимых процессов изменение свободной энергии Гиббса...
1) всегда положительно;
2) всегда отрицательно;
3) всегда равно нулю;
4) положительно или отрицательно в зависимости от обстоятельств.
Если ∆H < 0 и ∆S < 0, то в каком случае реакция может протекать
самопроизвольно?
1) [∆Н] > [T·∆S];
2) при любых соотношениях ∆Н и T·∆S;
3) [∆H] < [T·∆S];
4) [∆H] = [T·AS].
Определите тепловой эффект реакции ( ∆Н°298):
Fe2O3 + 2A1 = 2Fe + А12О3, если
∆Н0298 (А12О3)= -1676,0 кДж/моль;
∆H0298 (Fe2O3)= -822,2 кДж/моль.
1) +120,4 кДж; 2) - 120,4 кДж; 3) + 853,8 кДж; 4) - 853,8 кДж.
Действие ферментов:
а) изменяет тепловой эффект реакции;
б) снижает энергию активации;
в) увеличивает скорости прямой и обратной реакции;
г) является избирательным.
1) а, б, в, г; 2) б, в, г; 3) б, г; 4) а, б, г.
Кинетическое уравнение реакции СО(г) + 2Н2(г) =СН3ОН(г) имеет
вид:
l ) υ = [CO][H2]2;
2) υ = k[CO][H2]2;
100
23.
24.
25.
№ задания
1.
2.
3.
4.
3) υ = k[CO] +[H2]2;
4) υ = k[CO][H2].
В реакции 2А(г) + В(тв) = С + D, концентрацию вещества А увеличили в 3 раза, а вещества В увеличили в 2 раза. Как изменилась скорость реакции?
1) увеличится в 18 раз;
2) не измениться;
3) увеличится в 9 раз;
4) увеличится в 6 раз.
В реакции 4NH3(г) + 3О2(г) ↔ 2N2 + 6Н2О(ж); ∆Н° = -528кДж для
смещения равновесия влево необходимо:
1) повысить температуру;
2) понизить температуру;
3) увеличить давление;
4) увеличить концентрацию азота.
Чему равна скорость химической реакции, если концентрация
одного из реагирующих веществ в начальный момент была равна
1,2 моль/л, а через 50 мин стала равна 0,3 моль/л?
1) 0,024 моль/л·мин;
3) 0,006 моль/л·мин;
2) 0,018 моль/л·мин;
4) 0,012 моль/л·мин.
Вариант 4
Условие задачи
Расположите частицы по увеличению уровня химической организации вещества:
а) e; 714N; SO4 2-; NaClкрист ;
б) атом, молекула, молекулярный кристалл;
в) молекула, атом, элементарная частица;
г) SO4 2-; NaClкрист; атом азота; протон.
1) а, б; 2) в; 3)г; 4) все.
Укажите элемент, у которого конфигурация внешнего энергетического уровня 5s2 :
1) золото; 2) барий; 3) астат; 4) стронций.
Соотнесите:
квантовое число:
возможные значения:
1) п;
а) +1/2 и -1/2
2) l;
6) 1,2,3,4,....
3) ml
в) 0,1,2, ....( п -1)
4) ms
г) - l…0+ l
34. Какова формула валентных электронов атома Сu?
101
5.
6.
7.
8.
9.
10.
11.
12.
1) ...3d10;
2) ... 4s23d9;
3) ... 4s13d10;
4) ... 4s2.
У атома какого элемента 4-го периода на d -подуровне наибольшее число неспаренных электронов?
1) Сu; 2) Мn; 3) Ni; 4) Fe.
Чем меньше энергия ионизации, тем:
а) атом легче отдает e;
б) атом труднее отдает е;
в) сильнее металлические свойства;
г) слабее металлические свойства.
1) б, в; 2) а, г; 3) а, в; 4) б, г.
В группах s- и р-элементов сверху вниз:
1) возрастает восстановительная активность;
2) уменьшается энергия ионизации;
3) возрастает окислительная активность;
4) возрастает энергия сродства к электрону.
Химическая связь, образуемая за счет одной или нескольких
электронных пар, существенно взаимодействующих с ядрами
обоих соединяемых атомов, называется:
1) ковалентной;
2) ионной;
3) металлической;
4) водородной.
Укажите соединения, в которых есть ионная связь:
a) HF; б) СаВr2; в) BF3; г) K[BF4].
1) а, б, в, г; 2) б; 3) в, г; 4) б, г.
Ковалентная связь тем прочнее, чем:
а) энергия связи больше;
б) длина связи меньше;
в) энергия связи меньше;
г) длина связи больше.
1) а, г; 2) в, г; 3) б, в; 4) а, б.
Веществами молекулярного строения является:
1) NaCl (тв), Na2CO3 (тв), графит;
2) НС1 (тв), S8, SiO2;
3) CuCl2 (тв), алмаз, графит;
4) А1,Сu, Р4.
Оксиды, гидроксиды Ва и Са проявляют характерные свойства:
1) кислотные;
2) основные;
3) амфотерные;
102
13.
14.
15.
16.
17.
18.
19.
20.
4) восстановительные.
Основные свойства гидроксидов в ряду Be – Ва:
1) ослабевают;
2) усиливаются;
3) не меняются;
4) основные свойства не характерны.
Назовите реагенты А и В в следующей схеме превращений: а)
Сu(ОН)2 → СuSO4 ; б) СuSO4 → Сu(ОН)2:
1) А – H2SO4, В – Na2O;
2) А – H2SO4, B – H2O;
3) А – SO2, В – КОН;
4) А – H2SO4, В – NaOH.
Веществом А в схеме реакции СuО + А → Сu является:
1) H2SO4;
2) Н2;
3) NaOH;
4) СO2.
Какой из металлов в ряду Сr – Fe – Со – Ni проявляет более сильные восстановительные свойства?
l) Fe; 2) Сr; 3) Со; 4) Ni.
Какие продукты реакции образуются при пропускании NO2 через
раствор КОН?
1) KNO3, H2O;
2) KNO2, KNO3, H2O;
3) KNO2, H2O;
4) KNO3, NO, H2O.
Самопроизвольный характер процесса лучше определять путем
оценки:
1) энтропии;
3) энтальпии;
2) свободной энергии Гиббса;
4) температуры.
В каком из следующих случаев реакция неосуществима при любых температурах?
1) ∆H>0, ∆S<0;
2) ∆H=0, ∆S=0;
3) ∆H>0, ∆S>0;
4) ∆H<0, ∆S>0.
Определите тепловой эффект реакции:
NaH(к) + Н2О(ж) = NaOH(p) + Н2(г) по стандартным теплотам образования веществ, участвующих в реакции, если
∆H0298(NaHк) = -56,94 кДж/моль,
∆Н0298 (NaOHp) = -469,47 кДж/моль,
103
21.
22.
23.
24.
25.
№ задания
1.
∆Н0298(Н2Ож) =-285,8 кДж/моль.
1) +126,7 кДж; 2) - 126,7 кДж; 3) + 863,8 кДж; 4) - 863,8 кДж.
Действие катализатора на скорость химической реакции объясняется:
1) возникновением активированных комплексов;
2) увеличением числа столкновений;
3) возникновением активированных комплексов и изменением
энергии активации;
4) изменением энергии активации.
Какое выражение соответствует константе равновесия реакции
3Fe + 4Н2О(пар) ↔ Fe3O4 + 4Н2(г)?
1) К=[Н2]4/[Н2О]4;
2) К=[Н2]/[Н2О];
3) К=[Fe3O4][H2]4/[H2O]4[Fe]3;
4) К=[Fe3O4][H2]/[H2O][Fe].
В реакции А(г) + 3В(г) = С + D, концентрацию вещества А увеличили в 2 раза, а вещества В увеличили в 3 раза. Как изменилась
скорость реакции?
1) увеличится в 18 раз;
2) увеличится в 3 раза;
3) увеличится в 9 раз;
4) увеличится в 6 раз.
Взаимодействие хлора с водой протекает по уравнению:
С12(г) + Н2О(ж) ↔ HCl(p-p) + НСlО(р-р); Δ Н <0.
Соотнесите:
воздействие:
смещения равновесия
1) в систему добавили HCl;
а) сместится вправо;
2) в систему добавили NaOH;
б) сместится влево;
3) увеличили давление;
в) не сместится.
4) повысили температуру;
Определить температурный коэффициент скорости реакции, если
при понижении температуры на 30 °С реакция замедлилась в 8
раз?
1) 3,00; 2) 2,67; 3) 2,00; 4) 4,50.
Вариант 5
Условие задачи
Ядро атома 15 Р содержит:
1) 15 протонов и 31 нейтронов;
2) 16 протонов и 15нейтронов;
3) 15 протонов и 16 нейтронов;
4) 31 протон и 15 нейтронов.
104
2.
3.
4.
5.
6.
7.
8.
9.
Атом хлора имеет электронную конфигурацию:
1) 122s22p5;
2) 1s22s22p63s23p5;
3) 2s22p5;
4) 1s22s22p63s13p5.
Соотнесите:
подуровень:
1) s; 2) р; 3) d; 4) f.
максимальное число электронов в подуровне:
а) 2; б) 10; в) 6; г) 14.
Какова формула валентных электронов атома Ag?
1) ... 5s2;
2) ... 5s24d9;
3) ... 5s14d10;
4) ... 5s1.
Выберите ряд элементов, состоящих только из:
А) d-элементов: Б) р-элементов:
a) Ge, Ti, Zn, Sn, Si;
б) Si, As, S, Те, Кr;
в) La, Ge, Zr, V, Co;
г) La, Zn, Zr, V, Co.
1) А – г, Б – б; 2) А – г, Б – в; 3) А – б, Б – г; 4) А – в, Б – а.
Энергия ионизации - это энергия, необходимая для отрыва одного электрона от нейтрального атома. 1) да; 2) нет.
Выберите правильное утверждение.
В главных подгруппах ПСЭ сверху вниз:
а) радиус атома увеличивается;
б) сродство к электрону уменьшается;
в) электроотрицательность увеличивается;
г) энергия ионизации увеличивается.
1) все; 2) б, в; 3) в, г; 4) а, б.
Какую химическую связь называют металлической:
1) связь между атомами, осуществляемая общей для этих атомов
парой электронов;
2) связь за счет взаимодействия противоположно заряженных
ионов;
3) связь за счет совокупности электронов, свободно перемещающихся между положительно заряженными ионами в кристалле;
4) связь за счет электростатического взаимодействия молекулярных диполей.
Ионный характер связи в ряду соединений:
NaCl – MgCl2 – AlCl3 – SiCl4:
1) уменьшается;
105
10.
11.
12.
13.
14.
15.
16.
2) увеличивается;
3) не изменяется;
4) сначала увеличивается, потом уменьшается.
Чем выше электроотрицательность элементов, тем:
1) сильнее выражены неметаллические свойства;
2) сильнее выражены окислительные свойства;
3) слабее выражены неметаллические свойства;
4) сильнее выражены восстановительные свойства.
Какие вещества имеют молекулярную решетку:
а) I2; б) СО2; в) NaCl; г) КОН; д) Si; e) SiO2.
1) а, б, г; 2) а, б, е; 3) в, г, д; 4) все.
Оксид и гидроксид бериллия обладают свойствами:
1) кислотными;
2) основными;
3) окислительными;
4) амфотерными.
Выберите последовательность элементов, записанных в порядке
увеличения основного характера их высших оксидов и гидроксидов:
а) Ва, Sr, Ca, Mg, Bе;
б) Na, Mg, Al, Si, P;
в) P, Si, Al, Mg, Na;
г) Be, Mg, Ca, Sr, Ba.
1) г ; 2) a; 3) б, в; 4) в, г.
Назовите реагенты А и В в следующей схеме превращений:
Zn(NO3) 2 + А → Zn(ОН)2 + В → K2[Zn(ОН)4]:
1) А – Сu(ОН)2, В – КОН (нед.);
2) А – Н2О, В – КОН;
3) А – КОН (изб.), В – КОН (нед.);
4) А – КОН (нед.), В – КОН (изб.).
С какими веществами может взаимодействовать SО2 при комнатной температуре?
а) Н2О; б) HC1; в) О2; г) Na2O; г) Вr2 .
1) а, в, г;
2) а, б, в;
3) в, г, д;
4) со всеми.
Какой из оксидов СrО3, МоО3 или WO3 проявляет более сильные
окислительные свойства?
1) СrО3;
2) МоО3;
3) WO3;
4) проявляют в равной степени.
106
17.
18.
19.
20.
21.
22.
23.
24.
Какой продукт восстановления образуется в результате реакции
KMnO4 + Na2SO4 + H2SO4 →…
1) K2MnO4;
2) MnSO4;
3) MnO2;
4) MnO.
Мерой неупорядоченности расположения частиц в системе
является:
1) внутренняя энергия;
2) энтальпия;
3) энтропия;
4) энергия Гиббса.
При каких постоянных термодинамических параметрах изменение энтальпии может служить критерием направления самопроизвольного процесса? Какой знак ∆H в этих условиях указывает
на самопроизвольный процесс?
1) при постоянных S и Р, ∆H < 0;
3) при постоянных P и T, ∆H < 0;
2) при постоянных S и Р, ∆Н > 0;
4) при постоянных V и T, ∆H > 0.
Вычислить тепловой эффект реакции:
2С12(г) + 2Н2О(г) = 4НС1(г) + О2, если
∆Н°298(Н2Ог)= -241,8 кДж/моль;
∆Н°298(НС1г)= -92,3 кДж/моль.
1) +144,4 кДж; 2) - 144,4 кДж; 3) + 463,8 кДж; 4) - 463,8 кДж.
Если прямая реакция экзотермична, а обратная эндотермична, то
какая из них характеризуется более высоким значением энергии
активации?
1) прямая;
2) обратная.
Как влияет катализатор на смещение химического равновесия:
1) смещает влево;
2) смещает вправо;
3) не влияет.
В реакции 2А(г) + 3В(г) = С + D, концентрацию вещества А
уменьшили в 2 раза, а концентрацию вещества В уменьшили в 3
раза. Как изменилась скорость реакции?
1) увеличится в 6 раз;
2) увеличится в 18 раз;
3) уменьшится в 18 раз;
4) уменьшится в 36 раз.
В реакции N2 + 3Н2 ↔ 2NH3; ΔН= -9268 кДж для смещения
равновесия вправо необходимо:
107
25.
а) повысить температуру;
б) понизить температуру;
в) увеличить давление;
г) увеличить концентрацию водорода.
Чему равна скорость реакции гидролиза дипептида, если его концентрация в начальный момент была равна 0,1 моль/л, а через 30
с стала равна 0,05 моль/л?
1) 0,2 моль/л·мин;
2) 0,1 моль/л·мин;
3) 0,00167 моль/л·мин.
Ответы
№
вопро
са
Ва
риант
1
Ва
риант
2
Ва
риант
3
Ва
риант
4
Ва
риант
5
1 2 3 4 5 6 7 8 9 1 1 1 1 1 1 1 1 1 1 2 2 2 2 2 2
0 1 2 3 4 5 6 7 8 9 0 1 2 3 4 5
1 3 б 3 2 1 2 1 4 2 4 3 3 2 4 3 1 3 3 2 3 2 2 а
в
3
б
а
а
б
2 4 б 2 1 2 1 2 3 3 1 1 2 3 3 3 1 2 2 4 1 1 2 б
в
3
а
г
1
б
а
а
3 4 б 1 2 2 2 1 4 3 1 1 2 5 3 1 2 3 1 4 2 2 3 а
а
3
в
г
4
г
б
1 4 б 1 2 3 1 1 4 4 2 2 2 4 2 1 2 2 1 2 3 1 1 б
в
б
г
а
а
б
3 2 а 3 1 1 4 3 1 1 3 4 4 4 1 3 3 3 1 1 2 3 2 б
в
2
в
б
г
г
108
1
1
2
3
2
ЛИТЕРАТУРА
1. Абкин Г.Л. Задачи и упражнения по общей химии. – М.: Высшая
школа, 1988. – 264 с.
2. Ахметов Н.С., Азизова М.К., Бадыгина Л.И. Лабораторные и семинарские занятия по общей и неорганической химии. – М.: Высшая
школа, 1999. – 368 с.
3. Витинг Л.М., Резницкий Л.А. Задачи и упражнения по общей химии.
– М.: Изд-во Моск. ун-та., 1995. – 219 с.
4. Глинка Н.Л. Общая химия. – М.: Интеграл-Пресс, 2001. – 704 с.
5. Глинка Н.Л. Задачи и упражнения по общей химии – Л. Химия, 1985.
– 264 с.
6. Гольбрайх З.Е. Сборник задач и упражнений по химии. – М.: Высшая школа, 2007. – 285 с.
7. Гузей Л.С., Кузнецов В.Н. Общая химия. – М.: Изд-во Моск. ун-та,
1999. – 332 с.
8. Дикерсон Р., Грей Г., Хейт Дж. Основные законы химии. В 2 томах /
Пер. с англ. – М.: Мир, 1982. – 652 с., – 620 с.
9. Задачи и упражнения по общей химии: Учеб. пособие / Б. И. Адамсон, О.Н. Гончарук, В.И. Камышова и др.; Под ред. Н.В. Коровина. –
М.: Высшая школа, 2003. – 55 с.
10.Карапетьянц М.Х., Дракин С.И. Общая и неорганическая химия. –
М.: Химия, 1993. – 558 с.
11.Карапетьянц М.Х., Дракин С.И. Строение вещества. – М.: Высшая
школа, 1976. – 304 с.
12.Кемпбел Д. Современная общая химия. Т. 1-3. – М: Мир, 1989.
13.Коровин В.Н. Общая химия. – М.: Высшая школа, 2000. – 558 с.
14.Коровин Н.В., Масленникова Г.Н. и др. Курс общей химии. – М.:
Высшая школа, 1998. – 445 с.
15.Краткий справочник физико-химических величин / Под ред. А.А.
Равделя и А.М. Пономаревой. – СПб.: Специальная литература, 1998.
– 232 с.
109
16.Литвинова Т.Н. Сборник задач по общей химии. – М.: Мир и образование, 2007. – 224 с.
17.Некрасов Б.В. Основы общей химии, т. 1-2. – М.: Химия, 1973. – 518
с., – 415 с.
18.Николаев Л.А. Современная химия. – М.: Просвещение, 1970. – 208
с.
19.Пузаков С.А., Попков В.А., Филиппова А. Л. Сборник задач и
упражнений по общей химии: Учеб. пособие. – М.: Высшая школа,
2004. – 255 с.
20.Полинг А. Общая химия. – М.: Мир, 1974. – 848 с.
21.Практикум по общей химии / Е.М.Соколовская, О.С.Зайцев. – М.:
Изд-во Моск. ун-та, 1981. – 400 с.
22.Сборник задач и упражнений по общей химии: Учеб. пособие для
нехим. спец. Вузов /Л. М. Романцева, 3.Л. Лещинская, В.А. Суханова. – М.: Высшая школа, 1991. – 288 с.
23.Семенов И.Н., Перфилова И.Л. Химия: Учеб. для вузов. – СПб: Химиздат, 2000. – 656 с.
24.Слета Л.А., Черный А.В., Холин Ю.В. 1001 задача по химии с ответами, указаниями, решениями. – М: Илекса, 2004. – 368 с.
25.Суворов А.В., Никольский А.Б. Общая химия. – СПб: Химиздат,
2001. – 512 с.
26.Угай Я.А. Общая и неорганическая химия. – М.: Высшая школа,
1997. – 527 с.
27.Хомченко И.Г. Сборник задач и упражнений по химии. – М.: Высшая школа, 1989. – 253 с.
110
Приложение 1
Некоторые константы и величины
Число Авогадро А= 6,022·1023 частиц / моль
Постоянная Планка h =6,62·10-34 Дж·К-1
Постоянная Больцмана k б =1,3805·10-23 Дж·К-1
Универсальная газовая постоянная R=8,3143 Дж·К-1·моль-1
Масса электрона mе =9,110·10-31 кг =5,48580·10-4 а.е.м.
Масса протона mр =1,673·10-27кг=1,007276 а.е.м.
Боровский радиус а0 =5,292·10-11 м.
Атомная единица массы (а.е.м.)=1,66-27 кг
Скорость света с =2,998·108 м·с-1
Давление (стандартные условия) Р =1,013·105 Н·м-2 (1,013·10-5 Па)=760 мм
рт. ст.
Абсолютный нуль Т0 =-273,15 0С
Элементарный электрический заряд е =1,602189·10-19 Кл
Число Фарадея F =9,64846·104 Кл·моль-1
Соотношение между некоторыми единицами
в различных системах.
Величина
Энергия, работа
Длина
Давление
Единица измерения
Электроновольт (эВ)
Калория (кал)
Эгр
Микрон или микрометр
(мкм)
Нанометр
Ангстрем
Миллиметр ртутного
столба (мм рт. ст.)
111
Эквивалент в СИ
1,60219·10-19Дж
4,1868 Дж
10-7 Дж
1·10-6м
1·10-9м
1·10-10м
133,322 Па
Значения газовой постоянной в
различных единицах измерения
Едини- Дж/моль ·К
ца
R
8,3143
эрг/моль·К л·атм/моль кал/моль·К кгс·м/моль
·К
·К
7
8,3143·10
0,082057
1,98725
0,8478
Приложение 2
Стандартные энтальпии образования ∆Н0298, энтропии
∆ S0298 и энергии Гиббса ∆G0298 К (25 0С)
Вещество
Al (к)
AlCl3 (к)
AlCl3·6H2O(к)
Al2O3(к)
BaCO3(к)
ВаО (к)
Вr2(ж)
С (графит)
3СаО·Аl2O3 (к)
3СаО·Аl2O3·6H2O (к)
СН4 (г)
С2Н4 (г)
C2H4 (г)
С3Н8
н-С4Н10 (г)
изо-С4Н10 (г)
С5Н12 (г)
С6Н14
С6Н14 (ж)
С7Н16 (г)
С7Н16(ж)
С8Н18 (г)
С6Н12О6 (к)
СН3ОН (ж)
С2Н5ОН (ж)
н-С3Н7ОН (ж)
изо-С3Н7ОН (ж)
∆Н0298
кДж/моль
0
-704,2
-2693
-1676,0
-1219,0
-558,1
0
0
-3556,4
-5550,6
-74,9
52,3
-89,7
-103,8
-126,2
-131,6
-146,4
-167,2
-189,8
-187,8
-224,4
-208,5
-1273,0
-238,6
-277,6
-307,0
-302,3
112
∆S0298,
Дж/(моль·К)
28,3
109,3
318,2
50,9
112,0
70,3
152,3
5,74
205,5
404,8
186,2
219,4
229,5
269,9
310,1
348,9
388,4
295,9
427,8
328,5
466,7
126,8
160,7
192,9
179,9
∆G0298,
кДж/моль
0
-628,6
-2262,8
-1582,0
-1139,0
-528,4
0
0
-3376,5
-5015,6
-50,8
68,1
-32,9
-23,5
-17,2
-8,4
-0,3
-4,3
8,1
1,1
16,5
-919,5
-166,2
-174,8
-173,1
182,5
Вещество
н-С5Н13ОН (ж)
СаВr2 (к)
СаВr2·6Н2О (к)
СаСl2 (к)
СаСl2·6Н2О (к)
СаСО3 (к)
СаНРО4 (к)
СаНРО4·2Н2О (к)
СаО
Са(ОН)2 (к)
СаSO4·0,5Н2О (к)
СаSO4·2Н2О (к)
Сl2 (г)
Сl2O (г)
СlО2 (г)
СО (г)
СО2 (г)
СS2 (г)
2СаО·SiO2 (к)
2CaO·SiO2·2H2O (к)
3CaO·SiO2 (к)
Cu (к)
СuCl2 (к)
СuCl2·2H2O (к)
СuSO4 (к)
СuSO4·5H2O (к)
СuO (к)
F2 (г)
Fe (к)
FeO (к)
Fe2O3 (к)
Fe3O4 (к)
FeSO4 (к)
FeSO4·7H2O (к)
Н2 (г)
HBr (г)
HCl (г)
HF (г)
HI (г)
H2O (г)
Н2О (ж)
∆Н0298
кДж/моль
-306,1
-674,9
-2507,9
-795,0
-2599,6
-1207,0
-1815,6
-2405,2
-635,5
-986,6
-1577,8
-2024,0
0
76,6
105,0
-110,5
-393,5
115,3
-2308,5
-3140,8
-2968,3
0
-215,6
-818,6
-770,9
-2279,4
-162,0
0
0
-264,8
-822,2
-1117,1
-929,5
-3016,0
0
-34,1
-91,8
-270,9
26,6
-241,8
-285,8
113
∆S0298,
Дж/(моль·К)
254,8
130
304,7
113,6
285,1
88,7
111,5
189,6
39,7
76,1
130,6
194,3
222,9
266,2
257,0
197,5
213,7
237,8
167,4
171,2
168,7
33,2
108,1
190,6
109,0
300,0
42,6
202
27,2
60,8
87,4
146,2
121,0
409,1
130,5
198,6
186,8
173,7
206,5
188,7
70,1
∆G0298,
кДж/моль
-656,1
-2118,9
-750,2
-2198,9
-1127,7
-1682,4
-2156,2
-604,2
-896,8
-1437,8
-1798,7
0
94,2
122,3
-137,1
-394,4
65,1
-2193,2
-3335,9
2784,3
0
-171,4
-660,1
-661,8
-1879,9
-129,9
0
0
-244,3
-704,3
-1014,2
-825,6
-2512,0
0
-51,2
-94,8
272,8
1,8
-228,6
-237,3
Вещество
Н3РО4 (р)
Н2S (г)
I2 (г)
MgCO3 (к)
MgO (к)
MnCl2м (к)
MnCl2·4H2O (к)
N2 (к)
Na2CO3
Na2CO3·7H2O (к)
Na2SO4 (к)
Na2SO4·10H2O (к)
NH3 (ж)
Ni (к)
NiO (к)
NiSO4 (к)
NiSO4·7H2O (к)
N2O (г)
NO (г)
N2O3 (г)
NO2 (г)
N2O4 (г)
O2 (г)
OF2 (г)
Pb (к)
Р (к)
PBr3 (г)
PBr5 (ж)
PCl3 (г)
PCl5 (г)
PF3 (г)
PF5 (г)
PbO (к)
PbO2 (к)
S (к)
Sn (к)
SnO (к)
SnO2 (к)
SO2 (г)
SO3 (г)
SnCO3 (к)
∆Н0298
кДж/моль
-1288,3
-21,0
62,4
-1113,0
-601,8
-481,2
-1687,4
0
-1131,0
-4083,5
-1384,6
-4324,2
294,0
0
-239,7
-873,5
-2977,4
82,0
90,3
83,3
33,5
9,6
0
25,1
0
0
-132,0
-184,0
-287,0
-366,0
-956,5
-1593,0
-219,3
-276,6
0
0
-286,0
-580,8
-296,9
-395,8
-1216,4
114
∆S0298,
Дж/(моль·К)
-158,1
205,7
260,6
65,7
26,9
118,2
311,5
191,5
136,4
564,7
149,5
591,9
328,0
29,9
38,0
103,6
378,9
219,9
210,6
307,0
240,2
303,8
205,0
247,0
64,8
41,1
438,0
240,2
311,7
364,5
272,6
296,0
66,2
74,9
31,9
51,6
56,5
52,3
248,1
256,7
97,1
∆G0298,
кДж/моль
-1142,6
-33,8
19,4
-1029,3
-569,6
-440,4
-1426,0
0
-1047,5
-3424,3
-1266,8
-3642,9
238,8
0
-211,6
-763,8
-2463,3
104,1
86,6
140,5
51,5
98,4
0
42,5
0
0
-155,7
-175,7
-260,5
-305,4
-935,7
-1517,2
-189,1
-218,3
0
0
-256,9
-519,3
-300,2
-371,2
-1137,6
Вещество
∆Н0298
∆S0298,
кДж/моль Дж/(моль·К)
-590,4
54,4
0
41,6
-350,6
43,6
SnO (к)
Zn (к)
ZnO (к)
∆G0298,
кДж/моль
-559,8
0
-320,7
Приложение 3
Электродные потенциалы
Уравнение электродного процесса
Li++е-=Li
Rb++e-=Rb
K++e-=K
Cs++e-=Cs
Ca2++2e-=Ca
Na++e-=Na
Mg2++2e-=Mg
Al3++3e-=Al
Ti2++2e-=Ti
Mn2++2e-=Mn
Cr2++2e-=Cr
Zn2++2e-=Zn
Cr3++3e-=Cr
Fe2++2e-= Fe
Cd2++2e-Cd
2H2O+2e=H2+2OHNO3-+10H++8e-=
NH4++3H2O
NO3-+2H++e-=
NO2++H2O
NO3-+4H++3e-=
NO++2H2O
Стандартный потенциал при 250
С, В
-3,045
-2,925
-2,924
-2,923
-2,866
-2,714
-2.363
-1,663
-1,630
-1,179
-0,913
-0,763
-0,744
-0,440
-0,403
-0,828
0,870
0,780
0,957
Уравнение электродного процесса
2+
Co +2e-=Co
Ni2++2e-=Ni
Sn2++2e-=Sn
Pb2++2e-=Pb
Fe3++3e-=Fe
2H++2e-=H2
Bi3++3e-=Bi
Cu2++2e-=Cu
Cu++e-=Cu
Hg22++2e-=2Hg
Ag++e-=Ag
Hg2++2e-=Hg
Pt2++2e-=Pt
Au3++3e-= Au
Au++e-= Au
O2+2H2O+4e=4OHO2+4H++4e=2H2O
SO42-+8H++6e=S+4H2O
SO42-+10H++8e=H2S+4H2O
115
Стандартны потенциал при 250
С, В
-0.277
-0,250
-0,136
-0,126
-0,037
-0
0,215
0,337
0,520
0,788
0,799
0,850
1,188
1,498
1,692
0,401
1,228
0,357
0,300
СОДЕРЖАНИЕ
Предисловие
Глава 1. Основные понятия химии. Современная номенклатура неорганических веществ. Химические свойства и способы получения
неорганических соединений. Химические реакции
3
5
Основные понятия общей химии
Важнейшие химические свойства и способы получения оксидов, оснований, кислот, амфотерных гидроксидов и солей
Химические реакции. Классификация химических реакций
Примеры решения типовых задач
Задачи и упражнения для самостоятельной работы
Контрольная работа № 1
Глава 2. Строение атома. Периодическая система и Периодический
закон Д.И. Менделеева в свете современных представлений о строении атома. Химическая связь. Комплексные соединения
Строение атома
Периодический закон и периодическая система Д. И. Менделеева в свете
современных представлений о строении атома
Химическая связь
Комплексные соединения
Примеры решения типовых задач
Задачи и упражнения для самостоятельной работы
Контрольная работа № 2
Глава 3. Термодинамика химических процессов
Основные понятия термодинамики
Первый закон термодинамики
Второй закон термодинамики
Третий закон термодинамики
Примеры решения типовых задач
Задачи и упражнения для самостоятельной работы
Глава 4. Химическая кинетика. Обратимость химических реакций.
Химическое равновесие
Основные понятия химической кинетики
Факторы, влияющие на скорость химических реакций
5
11
116
13
20
23
27
35
35
38
40
45
49
54
59
63
63
65
67
68
69
71
73
73
73
Обратимость химических реакций. Химическое равновесие
Смещение химического равновесия
Примеры решения типовых задач
Задачи и упражнения для самостоятельной работы
Контрольная работа № 3 по разделам «Термодинамика» и «Химическая
кинетика»
Итоговый тест
Литература
Приложение
Для заметок
117
77
78
79
82
85
91
109
111
Download